You are on page 1of 139

X MATHEMATICS

By Deepika Bhati

Volume 3

Unit IV Geometry (Circles)

Unit V Trigonometry (App. of


trigonometry
By Deepika Bhati

Unit VI Mensuration (surface


area & volume )
CH – 9 – APPLICATION OF TRIGONOMETRY
Introduction
There are many applications of trigonometry but we shall study some of them in this section.
Trigonometry is used in many different fields such as Astronomy, Navigation, surveying, Periodic
functions, Optics, acoustics, etc. Here, we shall use trigonometry to find the Height and Distance of
various objects without actually measuring them.

Some Important Terms


(1) Line of Sight – The line drawn from the eye of the observer to the point of the object is called
the line of sight.

Y
In the above figure, Observer’s eye is at O point and the Object is at A point then OA will be the

IT
Line of sight. OX is the horizontal line.

(2) The Angle of Elevation – When the object is above the eye and we raise our head to see the
object then the angle formed by the line of sight and the horizontal line is called the angle of
elevation. It is also called the angular height of the object. In this condition, the object will be above
the horizontal line.
N
FI

AOX is the angle made by the line of sight OA and horizontal line OX.
Therefore, Angle of Elevation = AOX
IN

(3) The Angle of Depression – When the object is below the eye and we lower our head to see the
object then the angle formed by the line of sight and the horizontal line is called the angle of
depression. In this condition, the object will be below the horizontal line.

BOX is the angle made by the line of sight OB and horizontal line OX.

Therefore, Angle of Depression = BOX


To Get More material for
Maths IX - X Click below

CBSE MATHS
IX-X

To Get More Material for XI-


XII Click below

CBSE MATHS
XI-XII
APPLICATION OF TRIGONOMETRY LEARN MATHEMATICS BY : DEEPIKA MA’AM – 2 –

SOLVED EXAMPLES
8 3 ( 3  1) 8 3 ( 3  1)
Ex1. The angle of depression of the top and the  h= 3 1  h = 2
bottom of 8m tall building from the top of
 h = 4 3 ( 3  1)m
a multi-storyed building are 30° and 45°,
respectively. Find the height of the multi-
Ans : Height of building is 4 3 e j
3 1 m

storyed building and the distance of the


Ex2. From a point on a bridge across a river,
two buildings.
the angles of depression of the banks on

Y
Sol. Let AB be height of building and PC is
opposite sides of the river are 30° and 45°,
multistoryed building.
respectively. If the bridge is a height of

T
QPB = PBD = 30° (Pair of alternate
3m from the banks, find the width of the

I
QPA = PAC = 45° interior angles)
river.

d
Sol. A and B represents the two banks of river in

n
N o
the given figure :

F I
Let height of multistoreyed building be h m.
As ACDB is a rectanlge.
i n k B e y

N h
AC = BD [opposite sides of

T

I
and AB = CD = 8cm rectangle equal] XPA = PAD = 30° (Pair of alternate
Now PD = PC – CD = (h – 8) m and YPB = PBD = 45° interior angles)
In right PCA In right PAD
P PC P PD
 tan    tan 45° =
AC tan    tan 30° =
B B AD
h 1 3
 1 = AC  AC = h m ...(i) =  AD = 3 3m
3 AD
 BD = AC  BD = h m
In right PDB
In right PBD
P PD
tan    tan 45° =
P PD B BD
 tan    tan 30° =
BD
B
3
1=  BD = 3m
1 h 8 BD
 = h  3 (h  8) = h
3 Now AB = AD + BD
 3h  8 3 = h  3h  h = 8 3 AB = 3 3  3 = 3( 3  1) m
 h ( 3  1) = 8 3 Ans : Width of the river = 3( 3  1) m .
8 3 ( 3  1) Ex3. A boy is standing on the ground and flying
 h= 
( 3  1) ( 3  1) a kite with 100 m of string at an elevation
of 30o. Another boy is standing on the roof
DEEPIKA MA’AM # 8743011101 : CLASS – X : APPLICATION OF TRIGONOMETRY : INFINITY ... Think beyond...
APPLICATION OF TRIGONOMETRY LEARN MATHEMATICS BY : DEEPIKA MA’AM – 3 –

of a 10 m high building and is flying his ______________________________________


kite at an elevation of 45o. Both the boys
____________________________________________________________________________
are on opposite sides of both the kites.
______________________________________
Find the length of the string that the
second boy must have so that the two kites ______________________________________

meet. ______________________________________

F ______________________________________
o

100 m
45 ( D ______________________________________
C

Y
Building ______________________________________
)30o 10 m
A E
B ______________________________________

T
Sol. Let first boy standing on the ground at point A ______________________________________

I
and another boy is standing on the top of the ____________________________________________________________________________

d
o
roof at point D. Such that FAB = 30 and ______________________________________

n
N o
o
FDC = 45 . ______________________________________

I y
In right  ABF

e
______________________________________

B
F
P FB
 sin  =  sin 30o = ______________________________________

k
H 100

n
1 FB ______________________________________

i
 =  FB = 50 m

N h
2 100 ______________________________________

T
I
Now FC = FB – CB ______________________________________
FC = 50 – 10 = 40 m
______________________________________
In right  FCD
____________________________________________________________________________
P FC
 sin  =  sin 45o =
H FD ______________________________________
1 40 ______________________________________
 =  FD = 40 2 m
2 FD
______________________________________
Ans : Length of string of other boy is 40 2 m
______________________________________

______________________________________
Notes : _______________________________
______________________________________
______________________________________
______________________________________
______________________________________
____________________________________________________________________________
______________________________________
______________________________________
______________________________________
______________________________________
______________________________________
______________________________________

DEEPIKA MA’AM # 8743011101 : CLASS – X : APPLICATION OF TRIGONOMETRY : INFINITY ... Think beyond...
APPLICATION OF TRIGONOMETRY LEARN MATHEMATICS BY : DEEPIKA MA’AM – 4 –
PRACTICE EXERCISE – 1
Q1. A person observed the angle of elevation Q8. A TV tower stands vertically on a bank of a
of the top of a tower as 30o . He walked canal. From a point on the other bank directly
50m towards the foot of the tower along level opposite the tower, the angle of elevation of
ground and found the angle of elevation of the top of the tower is 60°. From another point
the top of a tower as 60o. Find the height of 20m away from this point to the line joining
the tower. Ans : 43.25 m this point to the foot of the tower, the angle of
Q2. Determine the height of a mountain if the elevation of the top of the tower is 30°. Find
elevation of its top at an unknown distance the height of the tower and the width of the
from the base is 45o and at a distance 10 km canal.

Y
further off from the mountain, along the same Ans : Width of canal = 10m and height of
line, the angle of elevation is 30o. tower is 10 3 m.

T
Ans : 13.66 km Q9. A man in a boat rowing away from a light

I
Q3. A person standing on the bank of a river house 150 m high takes 2 minutes to
observes that the angle of elevation of the change the angle of evation of the top at
top of a tree standing on the opposite bank
d
the light house from 45o and 30o. Find the

n
N
is 60o . When the moves 20m away from

o
speed of the boat. Ans : 3.2941 km/hr

I y
the bank, he finds the angle of elevation Q10. A man on cliff observes a boat at an angle

e
to be 30o . Find the height of the tree and of depression 30 o , towards the shore to

B
F
the width of the river. Ans : 10 3 m, 10 m the point immediately beneath him. Three

k
Q4. The shadow of a tower when the angle of minutes later the angle of depression of

n
elevation of the sun is 45o is found to be 10m the boat is found to be 60o, assuming that

i
N
longer than when it is 60o. Find the height of

h
the boat sails at a certain speed. Determine

T
I
the tower. Ans : 23.66 m how much more time to reach the shore.
Q5. The shadow of a vertical tower on level Ans : 90 sec
ground increses by 10 metres, when the Q11. A straight highway leads to the foot of a tower.
altitude of the sun changes from angle of A man standing at the top of the tower ob-
elevation 45o to 30o. Find the height of the serves a car at an angle of depression of 30°,
tower, correct to one place of decimal. which is approaching the foot of the tower with
Ans : 13.7 m a uniform speed. Six seconds later, the angle
Q6. From the top of a building 96m high, the of depression of the car is found to be 60°.
angle of depression of two vehicles on a Find the time taken by the car to reach thefoot
road at the same level and in the same of the tower from this points. Ans : 9 min
line with the foot of the building and on Q12. The angles of elevation of the top of a tower
the same side of it are xo and yo , where from two points at a distance of 4m and 9m
tanxO = 3/4 and tanyO = 1/3. Calculate the from the base of the tower and in the same
distance between vehicles. Ans : 160 m straight line with it are complementary. Prove
Q7. At a point on a level ground, the angle of that the height of the tower is 6m.
elevation of a vertical tower is found to Ans : Height of tower is 6m.
be such that its tangent is 5/12. On Q13. The angle of elevation of the top of a tower
walking 192 metres towards the tower, from two points a and b from the base and in
the tangent of the angle is found to be the same straight line with its are complemen-
3/4. Find the height of the tower.
tary. Prove that the height of the tower is ab .
Ans : 180 m

DEEPIKA MA’AM # 8743011101 : CLASS – X : APPLICATION OF TRIGONOMETRY : INFINITY ... Think beyond...
APPLICATION OF TRIGONOMETRY LEARN MATHEMATICS BY : DEEPIKA MA’AM – 5 –
PRACTICE EXERCISE – 2
Q1. Across a road of width 10m, stand two is 60 m. The angle of depression of the top of
buildings facing each other. From the top the first tower when seen from the top of the
of the first, which is 30m high, the angle second tower is 30 O. If the height of the
of elevation of the top of the second is second tower is 90 m, find the height of the
found to be 45 o , find the height of the first tower. Ans : 55.36 m
second building. Ans : 40 m Q9. The Horizontal distance between two towers
Q2. The horizontal distance between two trees is 120m. The angle of elevation of the top of
of different heights is 60m. The angle of the first tower when seen from the top of the
second tower is 600. If the height of the first

Y
depression of the top of the first tree when
seen from the top of the second tree is 45o. tower is 250 m find the height of the second
tower. Ans : 42.16m

T
If the height of the second tree is 80m, find
the height of the first tree. Ans : 20 m Q10. The angle of elevation of the top of a tower

I
Q3. The horizontal distance between two towers from the foot of a building is 600 and the angle
is 70m. The angle of depression of the top of of depression of the foot of a tower from the

d
top of a building is 300. If the height of the

n
N
the first tower when seen from the top of the

o
second tower is 30o . If the height of the building is 50 3 m find the height of the tower..

I y
second tower is 120m, find the height of the

e
Ans : 150 3m

B
first tower. Ans : 79.59 m

F
Q11. The horizontal distance between two towers
Q4. The angle of elevation of the top of the build-

k
is 60 m. The angle of depression of the top of
ing from the foot of the tower is 30° and the

n
the second tower is 30O. If the height of the

i
N
angle of elevation of the top of the tower from second tower is 90 m. Find the height of the
the foot of the building is 60°. If the tower is

T h first tower. Ans : 55.36 m

I
50 m high, find the height of the building. Q12. The angle of depression of the top of a build-
Ans : Height of building is 50/3 m. ing from the top of a tower is 600. If the dis-
Q5. The angle of elevation of the top of a hill at the tance between the tower and building is 160m
foot of a tower is 60O and the angle of eleva- and height of the tower is 310 m. Find the height
tion of the top of the tower from the foot of the of the building. Ans. 22.88m
hill is 30O. If the tower is 50 m high, find the Q13. The angle of elevation of the top of a tower
height of the hill. Ans : 150 m 30m high from the foot of another tower in the
Q6. The angle of elevation of the top of the build- same plane is 60O and the angle of elevation of
ing from the foot of the tower is 600 and the the top of the second tower from the foot of
angle of elevation of the top of the tower from the first tower is 30 O. Find the distance
the foot of the building is 300 if the building is between the two towers and also the height of
50m high. Find the height of the tower.
the other tower. Ans : 10 3 m, h = 10 m
Ans.16.6m
Q14. Two pillars of equal height are either side of a
Q7. The angle of depression of the foot of a build-
road which is 100 m wide. The angles of
ing from the top of a tower is  and the angle
elevation of the top of the pillars are 60O and
of depression of the foot of the tower from the
30O at a point on the road between the pillars.
top of a building is  . If the height of the tower
Find the position of the point between the
is hm. Find the height of the building.
pillars. Also, find the height of each pilllar.
Ans. h cot . tan 
Ans : 25m, h = 43.3 m
Q8. The horizontal distances between two towers

DEEPIKA MA’AM # 8743011101 : CLASS – X : APPLICATION OF TRIGONOMETRY : INFINITY ... Think beyond...
APPLICATION OF TRIGONOMETRY LEARN MATHEMATICS BY : DEEPIKA MA’AM – 6 –
PRACTICE EXERCISE – 3
Q1. From a window, 1.2m above the ground of a of elevation of the top of hill from the deck
house on one side of a street, the angle of of slap is 600. Find the height of the hill. If
elevation and the angel of depression of the the height of the deck is 10m. Ans. 40m
top and bottom of the pole on the opposite Q9. The angle of elevation of the top of a 100m
high tower from the bottom of a Rock is 600
side of the street are 30o and 45o. Find the
and the angle of elevation of the top of Rock
height of the pole, nearest to two places of
from the top of tower is 300, find the height
decimal. Ans : 1.893 m of the Rock. Ans.133.3m
Q2. From the top of a building 10m high. The Q10. From the top of cliff. The angle of elevation
angle of depression of the foot of a transmis-

Y
of the top of transmission tower is 300 and
sion tower is 450 and the angle of elevation the angle of depression of the foot of trans-
of the top of transmission tower is 600. Find mission tower is 600. If the height of tower is

T
the height of the transmission tower. 50m. Find the height of the clift. Ans.37.5m

I
Ans.27.32 Q11. A man on the deck of a ship is 16 m above
Q3. The angle of elevation of a window 10m water level. He observes that the angle of

d
above the ground from the food of building is elevation of the top of cliff is 45o and the

n
N
angle of depression of the base to 30 o .
300 and the angle of elevation of the top of

o
Calculate the distance of the cliff from the

I y
building from the window is 300. Find the
ship and the height of the cliff.
height of the building. Ans.20m
e
Ans : d = 43.712 m & h = 27.712 m

B
F
Q4. A man on the roof of a house, which is 10m
Q12. From a window 60 m high above the ground

k
high, observes the angle of elevation of the of a house in a street, the angles of elevation

n
top of a building as 45 o and angle of

i
and depression of the top and foot of

N h
depression of the base of the building as 30o. another house on opposite sides of the

T
I
Find the height of the building and its distance street are 60o and 45o respectively. Show
from the house. Ans : 17.3 m, 27.32 m that the height of the opposite house is
Q5. A man standing on the deck of a ship, which
is 10 m above water level. He observes the
e j
60 3  1 m .
Q13. A man on the deck of a ship. He observes
angle of elevation of the top of the hill as 60o
that the angle of elevation of the to p of cliff
and the angle of depression of the base of
is 450 and the angle of depressionof the base
the hill as 30o. Calculate the distance of the of cliff is 300. Find the height of the deck of
hill from the ship and height of the hill. ship if the height of the cliff is 150m.
Ans : 10 3 m, 40 m Ans.75( 3  1 m)
Q6. The angle of elevation of the top of a multi- Q14. From a window 15 m high above the ground
storyed building from the top of a 8m build- in a street, the angle of elevation and
ing is 600 and the angle of depression of its depression of the top and the foot of another
foot is 300. Find the height of the multi-storyed house on the opposite side of the street are
building. Ans.52m 30O and 45O respectively. Find the height of
Q7. From a top of a 7m high building , the angle the opposite house. Ans : 23.66 m
Q15. There are two temples one on each Bank of
of elevation of the top of a cable tower is 60°
a river, just opposite to each other one temple
and the angle of depression of its foot is 45°. is 50m high from the top of this temple. The
Determine the height of the tower. angle of elevationof the top of another temple
Ans : Height of tower is 19.11 m is 300 and the angle of depression of its bot-
Q8. The angle of elevation of the deck of a ship tom is 600. Find the height of other temple.
from the bottom of a hill is 300 and the angle Ans. 66.6m

DEEPIKA MA’AM # 8743011101 : CLASS – X : APPLICATION OF TRIGONOMETRY : INFINITY ... Think beyond...
APPLICATION OF TRIGONOMETRY LEARN MATHEMATICS BY : DEEPIKA MA’AM – 7 –
PRACTICE EXERCISE – 4
Q1. A vertical tower stands on a horizontal Ans : Height of flagstaff = 7.3 m.
plane and is surmounted by a vertical Q7. From a point on the ground, the angles of el-
flagstaff of height 5 metres. At a point evation of the bottom and the top of a trans-
on the plane, the angle of elevation of the mission tower fixed at the top of a 20m high
bottom and the top of the flagstaff are building are 45° and 60° respectively. Find the
respectively 30o and 60o. Find the height height of the tower.
of the tower. Ans : 2.5 m Ans : Height of transmission tower is 14.60m.
Q2. A pole 5m high is fixed on the top of a tower. Q8. A statue 1.6m tall stands on the top of pedes-
The angle of elevation of the top of the pole

Y
tal. From a point on the ground, the angle of
observed from the point ‘A’ on the ground is elevation of the top of the statue is 60° and
60o and the angle of depression of the point from the same point the angle of elevation of

T
‘A’ from the top of the tower is 45o. Find the the top of the pedestal is 45°. Find the height

I
height of the tower. Ans : 6.83 m of the pedestal.
Q3. A 7m long flagstaff is fixed on the top of Ans : Height of the pedestal is 21.84m
a tower on the horizontal plane. From a Q9.
d
From a point P on the ground, the angle of

n
N
point on the ground, the angle of elevation of

o
elevation of the top of a 10 m tall building

I
the top and bottom of a flagstaff are 45o and

y
and of a helicopter, hovering at some height

e
30o respevtively. Find the height of the tower over the top of the building are 30o and 60o,

B
F
correct to one place of decimal. respectively. Find the height of the

k
Ans : 9.562 cm helicopter above the ground.

n
Q4. A flagstaff stands on the top of a 20m high

i
Ans : Height of the helicopter = 30m.

N
tower, from a point on the ground, the angle

h
Q10. A flag staff stands on the top of a 5m high

T
of elevation of the top of the flagstaff is 60o

I
tower. From a point on the ground, the angle
and that of the tower is 45 o . Find the of elevation of the top of the flag-staff is 60o
distance of the point of observation on the and from the same point, the angle of
ground from the top of the flag staff. Also, elevation of the top of the tower is 45o. Find
find the height of the flagstaff. the height of the flag-staff.
Ans : 40 m, 14.64 m Ans : Height of the flagstaff = 3.65 m
Q5. A parachuitst is descending vertically. At a Q11. An aeroplane when flying at a height of 4000
certain height his angle of elevation from a m from the ground passes vertically above
point on the ground is 60o and when he has another aeroplane at an instant when the
descended 300m further, the angle of angles of the elevation of the two plans from
elevation becomes 45o from the same point the same point on the ground is are 60o and
of observation. Find the distance of the point 45o respevtively. Find the vertical distance
of observation from the place where the between the aeroplanes at that instant.
parachutist lands. Ans : 409.8 m Ans : Vertical distance bet. plans = 1693.3 m
Q6. From a point P on the ground the angle of Q12. A vertcal tower stands on a horizontal plane
elevation of the top of a 10m tall building is and is surmounted by a vertical flag staff of
30°. A flag is hoisted at the top of the building heighth. At a point on the plane, the angle of
and the angle of elevation of the top of the elevation of the bottom and the top of the flag-
flagstaff from P is 45°. Find the length of the staff are  and  respectively. Prove that the
flagstaff and the distance of the building from
h tan 
the point P. (Take 3 = 1.732) height of the tower is
tan   tan 

DEEPIKA MA’AM # 8743011101 : CLASS – X : APPLICATION OF TRIGONOMETRY : INFINITY ... Think beyond...
APPLICATION OF TRIGONOMETRY LEARN MATHEMATICS BY : DEEPIKA MA’AM – 8 –
PRACTICE EXERCISE – 5
Q1. From the top of a tower, the angles of point ‘h’ metres above a lake is  and the
depression of two objects on the same angle of depression of its reflection in the
side of the tower are found to be  and  lake is . Prove that the height of the
( > ). If the distance between in LM tan   tan  OP
objects is ‘p’ metres show that the height
‘h of the tower is given by : Q7.
N tan   tan  Q
cloud above the lake is : h
From the top of a hill, the angles of
p tan  tan  depression of two consecutive kilometer due
h Also, determine the east are found to be 30O and 45O. Find the
tan   tan 
height of the hill. Ans.1365 m

Y
height of the tower if p = 50 metres,
Q8. A round balloon of radius ‘a’ subtends
 = 60o,  = 30o. Ans : 43.25 m
an angle  at the eye of the observer, while
Q2. The angle of elevation of the top of a

T
the angle of elevation of its centre is .
tower from a point on the same level as

I
Prove that the height of the centre of the
the foot of the tower is . On advancing
balloon is : a sin cosec(/2).
‘p’ metres towards the foot of the tower,

d
Q9. At the foot of the mountain the angle of
the angle of elevation becomes . Show

n
N
elevation of its summit is found to be 45o.

o
that the height ‘h’ of the tower is give by

I
After ascending 1000m towards the
h
p tan  tan 
tan   tan 
, Also determine the
e y
mountain up a slope of 30 o inclination,

B
F
the elevation is found to be 60o. Find the
height of the tower if p = 150 metres,

k
height of the mountain. Ans : 1.366 km
 = 30o and  = 60o.

n
Ans : 129.9 m Q10. The angles of elevation of the top of a tower

i
N
Q3. A vertical tower stands on a horizontal from two points at a distance of 4m and 9m
plane and is surmounted by a vertical

T h from the base of the tower and in the same

I
flagstaff of height ‘h’. At a point on the straight line with it are complementary. Prove
plane, the angles of elevation of the that the height of the tower is 6m.
bottom and top of a flagstaff are  and  Ans : Height of tower is 6m.
respectively. Prove that the height of the Q11. The shadow of a flag staff is three times as
h tan  long as the shadow of the flag staff when
tower is :
b g
tan   tan 
.
the sun rays meet the ground at angle of 60O.
Find the angle between the sun rays and the
Q4. A tower subtends an angle  at a point
ground at the time of longer shadow..
on the same level as the foot of the tower
Ans : 30O
and at a second point ‘h’ metres above
Q12. The length of a string between a kite and a
the first, the depression of the foot of the
point on the ground is 90 metres. If the string
tower is . Show that height of the tower
makes an angle with the ground level such
is h tan  cot .
that = 15/8, how high is the kite ? Assume
Q5. From window h metres high above the
that there is no slack in the string.
ground of a house in a street, the angle of
Ans : 79.4m
elevation and depression of the top and
Q13. The length of the shadow of a tower standing
the foot of anoter house on opposite side
on level plane is found to be 2x metres lower
of the street are  and , respectively,
when the sun’s altitude is 30° than when it was
show that the height of the opposite house
45°. Prove that the height of tower is
is h(1 + tan cot)
Q6. The angle of elevation of cloud from a x( 3  1) metres.

DEEPIKA MA’AM # 8743011101 : CLASS – X : APPLICATION OF TRIGONOMETRY : INFINITY ... Think beyond...
APPLICATION OF TRIGONOMETRY LEARN MATHEMATICS BY : DEEPIKA MA’AM – 9 –

PRACTICE EXERCISE – 6
Q1. From a window 15 m high above the ground, 30o and 60o respectively. Find the height of the
in a street, the angles of elevation and depres- pillar and its distance from the hill.
sion of the top and foot of another house on [Ans : 133.33 m, 115.33 m]
the opposite side of the street are 30o and 45o Q8. From a window, 60 metres high above the
respectively. Show that the height of the oppo- ground, of a house in a street, the angles of
site house is 23.66 m. elevation and depression of the top and the foot
Q2. A vertical tower stands on a horizontal plane of another house on the opposite side of the
and is surmounted by a vertcal falgstaff. At a street are 60o and 45o respectively. Show that

Y
point on the plane, 30 metres away from the
tower, an observer notices that the elevation

the height of the opposite house is 60 1  3 
metres.
of the top and bottom of the flagstaff are 60o

T
Q9. The angle of elevation of a jet plane from a
and 45o respectively. Find the height of the flag- point A on the ground is 60o. After a flight of

I
staff and that of the tower. 15 seconds, the angle of elevation changes to
[Ans : 21.96 m, 30 m]

d
30o. If the jet plane is flying at a constant height
Q3. A vertcal tower stands on a horizontal plane

n
N
of 1500 3 m, find the speed of the jet plane.

o
and is surmounted by a vertcal flagstaff. At a [Ans : 720 kmph]

I y
point on the plane, 30 metres away from the

e
Q10. The angles of elevation and depression of the
tower, an observer notices that the angles of

B
top and bottom of a lighthouse from the top of

F
elevation of the top and bottom of the a building, 60 m high, are 30o and 60o respec-
flagstaff are 60o and 45o respectively. Find the

n k
tively. Find (i) the difference between the

i
height of the flagstaff and that of the tower. heights of the lighthouse and the building, and

N h
[Ans : 2.5 m] (ii) the distance between the lighthouse and the

T
I
Q4. A statue 1.46 m tall, stands on the top of a building. [Ans : (i) 20 m (ii) 34.64 m]
pedestal. From a point on the ground, the angles Q11. An observer from the top of a ligthouse, 100 m
of elevation of the top of the statue is 60o and above sea level, the angle of depression of a
from the same point, the angle of elevation ship, sailing directly towards it, changes from
of the top of the pedestal is 45o. Find the height 30o to 60o. Determine the distance travelled
of the pedestal. [Ans : 2 m] by the ship during the period of observation..
Q5. The angle of elevation of the top of a tower [Ans : 115.46 m]
from a point on the same level as the foot of Q12. The angle of elevation of the top of a building
the tower is 30o. On advancing 150 m towards from a point A on the ground is 30o. On mov-
the foot of the tower, the angle of the elevation ing a distance of 30 m towards its base to a
becomes 60o. Show that the foot of the tower point B, the angle of elevation changes to 45o.
is 129.9 metres. Find the height of the building and the distance
Q6. On a horizontal plane there is a vertcal tower of its base from the point A.
with a flagpole on the top of the tower. At a [Ans : 40.98 m, 70.98 m]
point, 9 metres away from the foot of the tower, Q13. The angle of elevation of the top of a tree from
the angle of elevation of the top and bottom of a point A on the ground is 60o On walking 20
the flagpole are 60o and 30o respectively. Find metres away from its base, to a point B, the
the height of the tower and the flagpole angle of elevation changes to 30o. Find the
mounted on it. [Ans : 5.19 m, 10.38m] height of the tree. [Ans : 17.32 m]
Q7. From the top of a hill 200 m high, the angles of
depression of the top and bottom of a pillar are

DEEPIKA MA’AM # 8743011101 : CLASS – X : APPLICATION OF TRIGONOMETRY : INFINITY ... Think beyond...
APPLICATION OF TRIGONOMETRY LEARN MATHEMATICS BY : DEEPIKA MA’AM – 10 –

SELF EVALUATION TEST SERIES

M.M : 20 M.T: 60 min. M.M : 20 M.T: 60 min.

TEST - 1 TEST - 2

Q1. From a point on the ground, the angle of Q1. The angle of elevation of an aeroplane from
elevation of the top of a tower is observed a point P on the ground is 60o. After a flight
to be 60o. From a point 40m above the first of 15 sec, the angle of elevation changes to
point of observation, the angle of elevation 30o. If the aeroplane is flying at a constant
of the top of the tower is found to be 45o.

Y
height of 1500 3 m, find the speed of the
Find the height of the tower and its aeroplane. [Ans : 720 km/hr] 4 M
horizontal distance from the point of

T
observation. [Ans : 94.64 m, 54.6 m] 4M Q2. The angle of elevation of the top of a tower

I
from a point A on the ground is 30o. On
Q2. From a window 60 m high above the ground moving a distance of 20 m towards the foot
of a house in a street, the angles of elevation

d
of the tower to a point B, the angle of
and depression of the top and foot of

n
N
elevation increases to 60o. Find the height

o
another house on opposite sides of the of the tower and the distance of the

I y
street are 60o and 45o respectively. Show

e
that the height of the opposite house is tower from A. [Ans : 10m, 10 3 m] 4 M

B
F
e
60 3  1 m j 4M Q3. An aeroplane when 3000 m high passes

k
vertically above another aeroplane at an

n
i
Q3. The angle of elevation of the top of a tower, instant when the angle of elevation of the

N h
as seen from two points A and B, situated two aeroplane from the same point on the

T
I
in the same line and at distance p and q ground are 60o and 45o respectively. Find
respectively, from the foot of the tower, are the vertical distance between the two
complementary. Prove that the height of the aeroplanes. [Ans : 1268 m] 4 M
tower is pq . 4M
Q4. Two ships are sailing in the sea on either
side of the lighthouse. The angles of
Q4. Two men on eightr side of a temple 75 m depression of the two ships, as observed
high observe the angle of elevation of the from the top of the lighthouse are 60o and
top of the temple to be 30 o and 60 o 45o respectively. If the distance between the
respectively. Find the distance between the
two men. [Ans : 173.2m] 4 M
ships is
200  3 1 m, find the height of
3
Q5. A man in a boat rowing away from a the lighthouse. [Ans : 200 m] 4 M
lighthouse 100 m high takes 2 min to change
the angle of elevation of the top of the Q5. A tower is 50 m high. Its shadow is x metres
lighthouse from 45o to 30o. Find the speed shorter when the sun’s altitude is 45o than
of the boat. [Ans : 0.60 m/s] 4 M when it is 30o. Find x, correct to nearest
cm. [Ans : 36.60 m] 4 M
RESULTS : SELF CHECKING PLAN RESULTS : SELF CHECKING PLAN
Q. No 1 2 3 4 5 6 7 8 9 10 11 T. M Per. Grade Q. No 1 2 3 4 5 6 7 8 9 10 11 T. M Per. Grade
Marks Marks

DEEPIKA MA’AM # 8743011101 : CLASS – X : APPLICATION OF TRIGONOMETRY : INFINITY ... Think beyond...
APPLICATION OF TRIGONOMETRY LEARN MATHEMATICS BY : DEEPIKA MA’AM – 11 –

SELF EVALUATION TEST SERIES

M.M : 20 M.T: 60 min. M.M : 20 M.T: 60 min.

TEST - 3 TEST - 4

Q1. A vertical tower stands on a horizontal plane Q1. A person standing on the bank of a river
and is surmounted by a vertical flagstaff of observes that the angle of subtended by a
height h. At a point on the plane, the angle tree on the opposite bank is 60o. When he
of elevation of the bottom of the flagstaff is retires 40m from the bank, he finds the angle

Y
 and that of the top of the flagstaff is . to be 30o. Find the height of the tree and the
Prove that the height of the tower is breadth of the river. 4M
FG IJ Q2. From the top of a lighthouse, the angles

T
h tan 
Htan   tan  K. 4 M of depression of two ships on its two sides

I
Q.2 A man standing on the deck of a ship, which are observed to be  and . If the height
is 10 m above water level observes the angle of the lighthouse is h metres and the line

d
of elevation of the top of a hill as 60o and joining the ships passes through the foot

n
N
the angle of depression of the base of the of the lighthouse, show that the distance
hill is 30o. Calculate the distance of the hill b
og g

I
h tan   tan 
from the ship and height of the hill.

e y
between the ships is
b
tan  tan 
.4M

B
F
[Ans : 10 3 m, 40 m] 4 M Q.3 A carpenter makes stools for electrician with
Q3. The angle of elevation of a cloud from a

k
a square top of side 0.5 m and at a height of
point h metres above a lake is  and the

n
1.5 m above the ground. Also, each leg is
angle of depression of its reflection in
i
N
inclined at an angle of 60o to the ground.Find
the lake is . Prove that the height of the

T h the length of each leg and also the lenghts of

I
b
h tan   tan  g two steps to be put at equal distances.
b
cloud above the lake is tan   tan  . g [Ans : 1.732m, 1.1077 m, 1.654 m] 4 M
4 M Q4. From the top of a tower, the angles of
Q.5 The angle of elevation of the top of a hill, depression of two objects on the same side
at the city centres of two towns on either of the tower are found to be  and
side of the hill are observed to be 30o and  ( > ). If the distance between the
60o, if the distance uphill from the first city objects is p metres, show that the height
centre is 9 m, find the kilometres, the
distance uphill from city centre correct upto p tan  tan 
of the tower is given by h 
two places of decimals. [Ans : 5.20 km] tan   tan 
4M . 4M

Q9. A man in a boat rowing away from a light Q.5 The angle of elevation of a cloud from a point
house 150 m high takes 2 seconds to 60 m above lake is 30o and angle of
change the angle of elevation of the top at depression of the reflection of cloud in the
the light house from 45o and 30o. Find lake is 60o, find the height of the cloud.
the speed of the boat. [Ans : 31.70 m/s] [Ans : 120 m] 4 M
4M
RESULTS : SELF CHECKING PLAN RESULTS : SELF CHECKING PLAN
Q. No 1 2 3 4 5 6 7 8 9 10 11 T. M Per. Grade Q. No 1 2 3 4 5 6 7 8 9 10 11 T. M Per. Grade
Marks Marks

DEEPIKA MA’AM # 8743011101 : CLASS – X : APPLICATION OF TRIGONOMETRY : INFINITY ... Think beyond...
APPLICATION OF TRIGONOMETRY LEARN MATHEMATICS BY : DEEPIKA MA’AM – 12 –

— Notes —
_________________________________________________________________________________________________________

_________________________________________________________________________________________________________

_________________________________________________________________________________________________________

_________________________________________________________________________________________________________

_________________________________________________________________________________________________________

Y
_________________________________________________________________________________________________________

_________________________________________________________________________________________________________

I T
_________________________________________________________________________________________________________

d
_________________________________________________________________________________________________________

n
N o
_________________________________________________________________________________________________________

I e y
_________________________________________________________________________________________________________

B
F
_________________________________________________________________________________________________________

n k
_________________________________________________________________________________________________________

i
N h
_________________________________________________________________________________________________________

T
I
_________________________________________________________________________________________________________

_________________________________________________________________________________________________________

_________________________________________________________________________________________________________

_________________________________________________________________________________________________________

_________________________________________________________________________________________________________

_________________________________________________________________________________________________________

_________________________________________________________________________________________________________

_________________________________________________________________________________________________________

_________________________________________________________________________________________________________

_________________________________________________________________________________________________________

_________________________________________________________________________________________________________

_________________________________________________________________________________________________________

_________________________________________________________________________________________________________
DEEPIKA MA’AM # 8743011101 : CLASS – X : APPLICATION OF TRIGONOMETRY : INFINITY ... Think beyond...
CIRCLES LEARN MATHEMATICS BY : DEEPIKA MA’AM – 13 –

CH – 10 – CIRCLES
Circle : Circle is a Locus of a point which remains
always equal distance from a stationary point. The One Tangent
O
stationary point is called centre of the circle and
fixed distance is called radius of the circle.
P
Tangent : Important definition about circle : Tangent
A line that intersect the circle in Exactly one point Case: 3 There are exactly two tengents through
is a called tangent to a circle in the given figure

Y
a point lying out side a circle.
PKT is a tangent to the circle
A Two Tangent
Secant : A line which intersects a circle in two

T
distinct point is called a secant in the given figure O

I
AB and PE ar the secant of the circle.
P

d
A B
F

n
N
B Number of common tangents on two circle.
M Diame Secant

o
ter
O P Case: 1 When two circles intersects each other

I y
C radiu Chord

e
s
then. There are two common tangents
N

B
are possible in the given fig. AB and

F
Tangent
K CD are two common tangents.

k
P T

n
Radius : The distance between the centre of a

i
N
circle and any point on the circle is called radius A

T
of the circle. It is denoted by liter (r). The SI unit
h B

I
O
of radius is ‘m’ in the given fig. OC represents
O'
the radius of circle.

Chord : A line segment joining any two points on C


D
a circle is called a chord of the circle in the given
Case: 2 When two circles touchs each other
fig. KP represents the chord of circle.
externally then, there are three common
Diameter : A chord posses through the centre of tangents are possible. In the given fig.
a circle is called diameter of the circle in the given AB, CD and PQ are common tangents.
fig. MN represents the diameter of the circle.
A
Note : In a circle longest chord is diameter.
P
Number of Tangents an a circle from a point. O B
R O’
Case: 1 There is no tangent passing through a C
point lying inside the circle. Q
D
No Tangent Case: 3 When two circles do not intersect each
O other then. There are four common
tangents are possible. In the given fig.
AB, CD, PQ and RS are common
Case: 2 There is one and only one tangent tangents.
passing through a point lying on a circle.
DEEPIKA MA’AM # 8743011101 : CLASS – X : CIRCLES : INFINITY ... Think beyond...
CIRCLES LEARN MATHEMATICS BY : DEEPIKA MA’AM – 14 –

A greater than OP in such a way. If we


consider so many point on tangent PT then
R B
O their distances from centre of the circle.
Q
Always greater than the radius of the circle.
P O' There is one and only one point on tangent
S
C PT i.e. P whose distance from the centre
D of the circle is smallest.
Case: 4 When two circles touches each other  OP  PT (We know that shortest
internally then. There is only one com- distance from an external point on a given
mon tangent is possible. In the given fig. line is perpendicular) Hence Proved
Th.2 : A line drawn through the end of a
APB is a common tangents.

Y
radius and perpendicular to it is a tan-
gent to the circle.
O’

T
Sol. Given : A circle with centre O and radius
OP is  to a line APB.

I
O
A P B
O

n d
N
Case: 5 When a cirlce lies entirely inside the
other circle then no common tangent
o
I e y
is possible. A P C D B

B
F
To prove : Line APB is a tangent to the circle.

k
Const. : Marks two point C and D on the given

n
O

i
line APB.

N h
Proof : In right OPC,

T
I
OC  OP ( Hypotenus is the longest
Th.1 Prove that Tangent line at any point of side in a right angle.)
a circle is perpendicular to the radius
OC  r (i)
through the point of contact.
In right OPD
OD  OP ( Hypotenus is the longest
O side in a right triangle)
 Point C and D both lies out side the circle
there is one and only one point on the line
R Q P T APB whose distance is equal to the radius
Given : A circle with centre O and PT is a Tangent of the circle.
line to the circle at the point of contact P.
 Line ABP touches the circle only one
To Prove : OP  PT or OPT  900 point P
Cons. : Consider two points Q and R on tangent
PT except P and join OQ and OR. APB is a tangent to the given circle.
Proof : Point Q lies out side the circle.
Th.3 : The lengths of two tangents drawn from
 OQ  r an external point to a circle are equal.
 OQ  OP (i)  OP  r  V.V.I.
A
Again point R lies out side the circle.
O
 OR  r
 OR  OP (ii)  OP  r 
From (i) and (ii) it is clear OQ and OR are P B

DEEPIKA MA’AM # 8743011101 : CLASS – X : CIRCLES : INFINITY ... Think beyond...


CIRCLES LEARN MATHEMATICS BY : DEEPIKA MA’AM – 15 –

Sol. Given : A circle with centre O. PA and


PB are two tangents are drawn from an Note : ________________________________
external point P ________________________________________
To prove : PA = PB
Const. : Join OA, OB and OP _______________________________________
Proof : Since, tangent to a circle is
perpendicular to the radius through the point _________________________________________
of contact ________________________________________________________________________________
 OAP  OBP  900 (each)
_______________________________________
In right  's AOP and BOP
OA  OB (radii of same ) _________________________________________

Y
OP  OP (Same Hypotenuse) ________________________________________

T
A  B (Each 90º proved) ________________________________________
 OAP   OPB (By RHS cons. rule)

I
_______________________________________
 PA = PB (By C.P.T.)
Hence Proved _________________________________________

n d
N
________________________________________

o
Th.4 : If two tangents are drawn to a circle

I y
from an External point then they subtends ________________________________________
equal angles at the centre and equally
e
_______________________________________

B
F
inclined with the line segment join

k
from point. V.V.I. _________________________________________
Given : Two tangents PA and PB

i n ________________________________________

N h
are drawn from an external point to a circle
________________________________________

T
I
C  O,r  .
_______________________________________
A
_________________________________________

O ________________________________________________________________________________
_______________________________________
P B _________________________________________

To prove : (i) POA  POB ________________________________________________________________________________


APO  BPO
(ii) _______________________________________
Proof : In POA and POB
_________________________________________
PA  PB (Tangents from Ex point A)
________________________________________________________________________________
OA  OB (radii of same circle)
_______________________________________
OAP  OBP  900
  POA  POB (By SSS cong. rule) _________________________________________
 POA  POB (By CPCT) ________________________________________________________________________________
and APO  BPO (By CPCT)
_______________________________________
_________________________________________
________________________________________

DEEPIKA MA’AM # 8743011101 : CLASS – X : CIRCLES : INFINITY ... Think beyond...


CIRCLES LEARN MATHEMATICS BY : DEEPIKA MA’AM – 16 –
PRACTICE EXERCISE – 1
Q1. A point P is 13cm away from the centre
Q11. From an external point P, tangents PA and
of the circle. The length of the tangent PB are drawn to a circle with centre O.
drawn from P to the circle is 12cm. Find If CD is a third tangent is a touches to
the radius of the circle. Ans. : 5cm the circle at a point E and PA = 14cm, find
Q2. Find the length of the tangent drawn from the perimeter of PCD. Ans : 28cm
a point whose distance from the centre of
a circle is 25cm given that the radius of Q12. A circle inscribd in a right angled triangle
the circle is 7cm. Ans : 24cm ABC right angle at B . If AB = 6 cm
BC = 8 cm. Find the radius of the circle.
Q3. Find the length of a tangent drawn to a
V.V.I. Ans. 2cm.
circle with radius 5cm, from a point 13cm

Y
from the centre of the circle. Ans : 12cm Q13. With the vertices of a triangle ABC as
Q4. A circle inscribed in a triangle ABC and centres, three circles are described, each
touching the other two externally. If the

T
touching the sides AB, BC and AC at the
points P, Q and R. If perimeter of sides of the triangles are 9cm, 7cm and

I
6cm, find the radii of the circles.
ABC  36cm. Find the sum of AP, BQ
Ans : 4cm, 5cm, 2cm
and CR. V.V.I. Ans. 18
Q14.

n d
A circle with centre O. Inscribed in a

N
Q5. A circle inscribed in a ABC , and touching triangle PQR and touch of the sides PQ,

o
the sides AB, BC and AC at points, P, Q QR and PR at points t, V and S respectively.

I y
and R respectively if AP = 2 cm, BQ = If PT = 16 cm, RS = 8 cm, and VQ =
4 cm and CR = 3 cm, find the perimeter

B e
8 cm. Find the perimeter of PQR .

F
of ABC . Ans. 18 cm
Ans. 64 cm.

k
Q6. A circle inscribed in a triangle ABC, and
Q15. C a n a q u a d r i l a t e r a l A BC D w i t h

n
touching the sides AB, BC and AC at

i AB = 7cm, BC = 6cm, CD = 8cm and DA

N
points, P, Q and R respectively, if radius
of circle is 2 cm and area of ABC = 12

T h = 9cm, circumscrible a circle ? Ans. Yes

I
cm2 find the perimeter of ABC . Q16. Find the length of fourth side of a
Ans.12cm circumscribing quadrilateral when lengths of
Q7. A circle inscribed in a triangle ABC, the its three consecutive sides are 4cm, 5cm
radius of the circle is 2 cm, and perimeter and 7cm, respectively.V.V.I. Ans : 6cm
of ABC is 36 cm, find the area of ABC .
Q17. A circle inscribed in a ABC touching AB,
2
V.V.I. Ans.36 cm BC and AC. At points, P, Q and R,
Q8. A circle inscribed in a quadrilateral ABCD respectively. If AB = 10 cm, AR = 7 cm
and touching the sides AB, BC and AC at and CR = 5 cm, find the length of BC.
points, P, Q and R respectively. If AP =
Ans. 8cm
3 cm, BQ = 5 cm, CR = 3 cm, and DS
= 5 cm, find the perimeter of ABCD. Q18. A circle inscribed in a ABC and touching
Ans. 32 cm the sides AB, BC and AC at the points P,
Q and R if PB + CQ + AR = 36 cm. Then
Q9. A circle inscribed in a triangle ABC and
find the perimeter of ABC . Ans.72
touches the sides AB, BC and AC at points
P, Q and R if the radius of circle 2cm and Q19. A circle draw Ex. of a ABC, touching
S = 20 cm. Find the area of ABC . the sides BC at point P, and the sides AB
Ans.40cm2 and A at points Q and R respectively by
Q10. If PA and PB are tangents from an outside producing. If BQ = 5 cm, CR = 7 cm, AB
point P such that PA = 10cm and = 9 cm, and AC = 7cm. Find the perimeter
of ABC . V.V.I. Ans. 28cm
PAB  600. Find the length of chord AB.
V.V.I. Ans : 10cm

DEEPIKA MA’AM # 8743011101 : CLASS – X : CIRCLES : INFINITY ... Think beyond...


CIRCLES LEARN MATHEMATICS BY : DEEPIKA MA’AM – 17 –
PRACTICE EXERCISE – 2
Q1. In the given fig. PA and PB are two Q8. In the given fig. two circles touches externally
tangents to the given circle such that at point P from an external point T, TA and
TB are drawn on the given circle prove that
PA = 5 cm, and APB  600 , find the
TA = TB. V.V.I.
length of chord AB. Ans. 5cm
T
A

0
60 P
A B

P
O O'
B

Q2. A chord of length 24cm of larger circle Q9. In the given fig. two circles touches externally

Y
becomes a tangent to a smaller circles of at point P. T is a point on common tangent
radius 5cm. If the circles are concentric, AB. Prove that TA = TB or T is the mid
find the radius of larger circles. point AB.

T
Ans : 13cm A T
B

I
Q3. In the given fig. ABCD is a quadrilateral,
touching the sides, AB, BC, CD and AD P
O'
O
at points Q, R, S and P respectively. Such
that DAB  900 , it CS = 27 cm and BC Q10. In the given fig. If AB = AC, prove that

n d
N o
= 38 cm and radius of the circle is 10 cm, BE = EC. V.V.I.

I y
find AB. V.V.I. Ans. 21 cm A

e
D S C

B
F
R
O

k
P F
D

n
O

i
N
A Q B C

h
B
E
Q4. Two circles of radii 3.5cm and 5.5cm, touch

T
I
Q11. A circle is touching the side BC of ABC
each other externally. Find the distance
at P and touching AB and AC produced at
between their centres. Also find the distance
Q and R respectively. Prove that AQ = ½
between their centres when they touch
(Perimetre of ABC ).
each other internally. Ans : 9cm, 2cm
Q5. In the given fig. two concentric circles with Q12. In the given fig., XP and XQ are tangents
centre O. If PA and PB are two tangents from X to the circle with centre O. R is
from a point P on common tangent PT. a point on the circle. Prove that XA + AR
Prove that PA = PB. V.V.I. = XB + BR. V.V.I.
B P
A
O'
A O R X
O
B
T P Q
Q6. In the given fig. XP and XQ are tangents
from X to the circle with centre O. R is Q13. In the given fig., the incircle of ABC ,
a point on the circle. Prove that PX = QX touches the sides BC, CA and AB at D,
= ½. Perimeter of ABC . V.V.I. E and F respectively. Show that AF + BD
+ CE = AE + BF + CD = 1/2 (Perimeter
P
A of ABC ) V.V.I.
O R A
X

B
Q

Ans : 8cm, 9.5cm, 8.5cm F


O
E

Q7. In the above fig. if PX = 14 cm. Find


perimeter of ABC . Ans. 28cm. B
D
C

DEEPIKA MA’AM # 8743011101 : CLASS – X : CIRCLES : INFINITY ... Think beyond...


CIRCLES LEARN MATHEMATICS BY : DEEPIKA MA’AM – 18 –
PRACTICE EXERCISE – 3
Q1. In the given fig. two circles touches externally
A
at point P. T is a point on common tangent
AB. Prove that APB  900 . V.V.I.
A R Q
T
B O
B C
P P
O O'
Q7. In the given fig. O is the centre of two
concentric circle. AB is a chord of the
Q2. In the given fig. two circles C  O, R  and larger circle touching the smaller circle C.
Prove that AC = BC.

Y
C  O', R ' thouches each other externally at
point P. Prove that ATB  APB  1800 .

T
T
B

I
A C

Q8. In the given fig. AB is a chord of length

d
9.6 cm, of a circle with centre O and radius

n
N
A B 6 cm. The tangent at A and B intersect at

o
O
O P. Find the length of PA. Ans. 8cm

I y
P
A

Q3. Two tangents inclined at an angle of 60 0

B e
F
P
are drawn to a circle of radius 3 cm. Then O

k
find the length of each tangent.

n
B

i
Ans. 3 3 cm
Q9. In the given fig. O is the centre of two

N h
Q4. In the given fig. if AP = 2 cm then find concentric circles of radii 5 cm and 3 cm,

T
I
the length of AB. Ans. 4cm from an external point P. PA and PB are
A two tangent drawn to the circle. If PA=12
B cm, then find find then lenth of PB.
V.V.I. Ans. 4 10 cm.
O O' A
P
Q5. In the given fig. a circle is inscribed in a O
P
0
quadrilateral ABCD in which B  90 . If B
AD = 23 cm, AB = 29 cm and DS = 5
Q10. A circles inscribed in a ABC having sides
cm, find the radius (r) of the circle. V.V.I.
8 cm, 10 cm and 12 cm, find AD, BE and
Ans. r = 11 cm. CF. Ans. 7cm, 5cm, 3cm
A A
R Q
D F
B
D O
O
S P
B E C
C
Q11. Prove that the segment joining the points
Q6. A cirlce is inscribed in a ABC touching of centact of two parallel tangent passes
BC, CA and AB at P, Q and R respectively through the centre of the circle.
as shown in the given fig. then find the
length of BC. If AB = 12 cm and AC = Q12. In the concentric circles, prove that all
10 cm and AR = 7 cm.V.V.I. Ans. 8cm chords of the outer circle which touch inner
are of equal length.

DEEPIKA MA’AM # 8743011101 : CLASS – X : CIRCLES : INFINITY ... Think beyond...


CIRCLES LEARN MATHEMATICS BY : DEEPIKA MA’AM – 19 –
PRACTICE EXERCISE – 4
Q1. In the given figure, find the values of x, y and z. Q7. In the given figure find the value of x and y.
[Ans. x = 60°, y = 30°, z = 1200]
C
0
30
B

Z
A
[Ans. x = 30°, y = 60°]
Q2. In the given figure the length of PQ is equal to Q8. In the given figure, find the value of x and y.
the radius of the circle then find the vlaue of x. [Ans. x = 100, y = 300]

Y
[Ans. 30°]
o
40

I
Q9.
T
In the given figure find the value of x.

n d
N
P
[Ans. x = 600]
Q3. In the given figure, find the value of x.
o
I y
[Ans. x = 30°]

B e

N F T h i n kQ10. In the given figure, find CAB.


[Ans. 30°]

I
Q4. In the given figure, find the value of x.
[Ans. x = 60°]
C

Q11. In the given fig. prove that


0
BAC  DCA  90 .
Q5. In the given figure, find the value of x.
[Ans. x = 120°] D
A

C
O

Q12. In the given fig. find the value of x.


Ans.500
Q6. In the given figure, find the value of x and y.
C

600

D O B
x

400
P A Q
[Ans. x = 70°, y = 60°]
DEEPIKA MA’AM # 8743011101 : CLASS – X : CIRCLES : INFINITY ... Think beyond...
CIRCLES LEARN MATHEMATICS BY : DEEPIKA MA’AM – 20 –
PRACTICE EXERCISE – 5
Q1. If PA and PB are two tangents to a circle with Q6. If the angle between two radii of a circle is 1300,
centre O such that AOB  1100 , find APB then the angle between the tangents at the ends
of the radii is. Ans. 500
Ans. 700 A
A T

1300 O
O 1100 P

B
B
Q7. If PB and PA are tangents to the circle with cen-
Q2. If PA and PB are two tangents to a circle with

Y
tre O such that APB  500 , find OAB .
centre O such that APB  800 , find AOP .
Ans. 250
Ans. 500

T
A

I
0
P 50 O

P O

n d
N o
B
Q8. In the given figure, O is the centre of a circle,

I y
Q3. In the given figure, PQR is a tangent to the circle PQ and PR are two tangents from an Ex point P
at Q, whose centre is O and AB is a chord par-

B
it RPO =150 . Find POR Ans. POR  750
e
F
allel to PR such that BQR  700 , find AQB . Q

Ans. 400

n k
i
P O

N
A B

h
L

T
I
O R

700
Q9. In the given figure, O is the centre of a circle,
BOA is its diameter and the tangent at the point
P Q R
P meets BA extended at T. If PBO  300 , find
Q4. In the given figure, O is the centre of a circle PTA . Ans. PTA  30 0
and PT is the tangent to the circle. If PQ is a P
chord such that QPT  500 , find POQ .
0
30
0 B T
Ans. POQ  100 O A

P T
50

Q10. In the given figure, O is the centre of a circle;


0

O PQL and PRM are the tangents at the points Q


Q and R respectively and S is a point on the circle
such that SQL  500 and SRM  600 , find
Q5. In the given figure, PQ is a chord of a circle with QSR . Ans. QSR  700
centre O and PT is tangent at P such that
QPT  600 . Find PRQ . Ans.1200 L

0
Q S 50 Q

O
O
R
0
60
0
60
M R
A P T

DEEPIKA MA’AM # 8743011101 : CLASS – X : CIRCLES : INFINITY ... Think beyond...


CIRCLES LEARN MATHEMATICS BY : DEEPIKA MA’AM – 21 –
PRACTICE EXERCISE – 6
Q1. A circle touches all the four sides of a Q10. In fig., if TP and TQ are the two tangents
quadilateral ABCD. Prove that AB + CD to a circle with centre O so sucth that
= BC + DA. V.V.I.
POQ 1100 , then PTQ is equal to: Ans.700
Q2. If all the sides of a parallelogram touch a
circle, show that the parallelogram is a Q11. Prove that the length of tangents drawn
rhombus. from an Ex. point to a circle are equal.
Q3. PA and PB are tangents from P to the circle
Q12. Two circles touch externally at a point P
with centre O. At point M, a tangent is
from a point T on a common tangent PT,
drawn cutting PA at K and PB at N. Prove
tangent segment TQ and TR are drawn to

Y
that KN = AK + BN.
the two circles. Prove that TQ = TR. V.V.I.
Q4. Prove that the tangents at the extremies of

T
any chord make equal angles with the Q13. Two circles touch each other externally at
chord. V.V.I. point C. AB is one of the direct common

I
tangent-segments. Prove that the common
Q5. From an external point P, two tangents PA
tangent at C bisects AB.

d
and PB are drawn to the circle with centre

n
N
O. Prove that OP is perpendicular bisector
Q14. O is the centre of a cricel. AB and AC

o
of AB.
are tangents to the circle. Prove that :

I y
(i) ABOC is a cyclic quadilateral.

e
Q6. In the given fig. l and m are two parallel
tangents at A and B. The tangent at C (ii) OA bisects BAC. V.V.I.

B
F
makes an intercept DE between l and m.

k
Prove that DOE  900 Q15. Two circles touch internally at P. T is any

n
point on the tangent at P. From T, two

i
N
tangents TA and TB are drawn tothe two

h
A D
l circles. Prove that TA = TB.

T
Q7.
I O

B E
C

In the given fig. O is the centre of the


circle, PA and PB are tangent segments.
Show that the quadrilateral AOBP is cyclic.
Q16.

Q17.
Two circles touch each other at O. A line
through O meets the circle in A and B.
Prove that tangents at points A and B are
parallel to each other.

Prove that the angle between the two


tangents drawn from an external point to
a circle is supplementary to the angle
subtended by the line segment. Joining the
point of contact at the centre. V.V.I.
A
Q18. The radius of the incircle of a triangle is
O 4cm and the segment into which one side
is divided by the point of contact are 6cm
P and 8cm. Determine the other side of the
B
triangle.
Q8. From a point P, two tangents PA and PB
are drawn to a circle with centre O. If OP Q19. A circle touches the sides of a quadrilateral
= diameter of the circle, show that  APB ABCD at P, Q, R and S respectively show
is equilateral. that the angles subtended at the centre by
Q9. Two tangent segments PA and PB are a pair of opposite sides are supplementary.
drawn to a circle with centre O such that
APB  1200 . Prove that OP = 2AP. V.V.I.

DEEPIKA MA’AM # 8743011101 : CLASS – X : CIRCLES : INFINITY ... Think beyond...


CIRCLES LEARN MATHEMATICS BY : DEEPIKA MA’AM – 22 –
PRACTICE EXERCISE – 7
Q1. In the given fig., AB is the diameter of the Q7. In the fig., (i) given below, ABC is a right
circle, with centre O, and AP is a tangent. angled triangle at A with sides AB = 5cm
Find the volume x. Ans. : 58º and BC = 13cm. A cricle with centre O
B has been inscribed in the triangle ABC.
Calculate the radius of the incircle. V.V.I.
O
K
Ans. 2cm
64º

x
P
C
A

Q2. Two circles of radii 25cm and 9cm touch


each other external find the length of direct

Y
13cm
common tangent. Ans : 30cm
Q3. In the given fig. PQ and RS are two parallel O

T
tangents to a circle with centre O and
another tangent XY, with point of contact A 5cm B

I
C intersect PQ at A and RS at B. Prove
Q8. Prove that the parallelogram circumsoribing
that : AOB = 900 (T.B.Q.9)

d
a circle is a rohmbus.

n
N
x
P A M Q

o
1 Q9. Prove that the tangents drawn from an

I
2

y
C external point to a circle subtended equal

e
O
angles at the centre of the circle.

B
3

F
4
B
R
y
N S
Q10. In the given fig. two tangents AB and AC
Q4. In the fig. given below, triangle ABC is

n k are drawn to a circle with centre O. Such

i
4 6 that BAC  1200 . Prove that OA = 2AB.

N
cm

h
circumscribed, find r. Ans.
3

T
I
B

P O

O
C

R
Q11. In the given fig. OQ : PQ = 3 : 4 and
Q5. In the fig., given below, quadrilateral ABCD
perimeter if POQ  60 cm . Determine
is circumscribed; find the perimeter of
quadrilateral ABCD. Ans.36cm PQ, QR and OP.Ans. 20, 30 and 25
Q
D 4 C

3
P
S Q
O

O
A 6 P 5 B
R
Q6. In the given fig. A circle with centre O
inscribed in a ABCD and touching the Q12. In the g i v en fig. pr ove tha t
sides AB, BC, CD and AD at points P, Q, BAC  CAD  90 0
R and S. If radius of the circle is 5cm.
Find the perimeter of ABCD. Ans.52cm D

m
C
13c
Q
B O C
8cm

R
P
P
A D B
S 8cm

DEEPIKA MA’AM # 8743011101 : CLASS – X : CIRCLES : INFINITY ... Think beyond...


CIRCLES LEARN MATHEMATICS BY : DEEPIKA MA’AM – 23 –
PRACTICE EXERCISE – 8
Mark the correct alternative in each of the following: Q9. PQ is a tangent drawn from a point P to a circle
Q1. A tangent PQ at a point P of a circle of radius with centre O and QOR is a diameter of the
5cm meets a line through the centre O at a point circle such that POR = 120°, then OPQ is
Q such that OQ = 12cm. Length PQ is (a) 60° (b) 45°
(a) 12cm (b) 13cm (c) 30° (d) 90° Ans. c
(c) 8.5cm (d) 119cm Ans. d Q10. If four sides of a quadrilateral ABCD are tan-
Q2. From a point Q, the length of the tangent to a gential to a circle, then
circle is 24cm and the distance of Q from the (a) AC + AD = BD + CD

Y
centre is 25cm. The radius of the circle is
(b) AB + CD = BC + AD
(a) 7cm (b) 12cm
(c) AB + CD = AC + BC

T
(c) 15cm (d) 24.5cm Ans. a
(d) AC + AD = BC + BD Ans. b

I
Q3. The length of the tangent from point A at a circle,
Q11. The length of the tangent drawin from a point
of radius 3cm, is 4cm. The distance of A from
8cm away from the centre of a circle of radius

d
the centre of the circle is
6cm is

n
N
(a) 7 cm (b) 7cm

yo
(a) 7 cm (b) 2 7 cm

I
(c) 5cm (d) 25cm Ans. c

e
(c) 10cm (d) 5cm Ans. b
Q4. If tangent PA and PB from a point P to a circle

B
F
Q12. AB and CD are two common tangents to circles
with centre O are inclinded to each other at an

k
which touch each other at C. If D lies on AB
angle of 80° then POA is equal to

n
such that CD = 4cm, then AB is equal to

i
(a) 50° (b) 60°

N h
(a) 4cm (b) 6cm
(c) 70° (d) 80° Ans a

T
I
(c) 8cm (d) 12cm Ans. c
Q5. If TP and TQ are tangents to a circle with cen-
tre O so that POQ = 110°, then PTQ is equal Q13. In figure, if AD, AE and BC are tangents to the
to circle at D, E and F respectively. Then
(a) 60° (b) 70°
(c) 80° (d) 90° Ans. b
Q6. PQ is a tangent to a cicle with centre O at the
point P. If OPQ is an isosceles triangle, then
OQP is equal to
(a) 30° (b) 45° (a) AD = AB + BC + CA
(c) 60° (d) 90° Ans. b (b) 2AD = AB + BC + CA
Q7. Two circle touch each other externally at C and (c) 3AD = AB + BC + CA
AB is a common tangent to the circles. Then (d) 4AD = AB + BC + CA Ans. b
ACB = Q14. In figure, RQ is a tangent to the circle with cen-
(a) 60° (b) 45° tre O. If SQ = 6cm and QR = 4cm, then OR =
(c) 30° (d) 90° Ans. d
Q8. ABC is a right angled triangle, right angled at B
such that BC = 6cm and AB = 8cm. A circle
with centre O is inscribed in ABC. The radius
of the circle is
(a) 1cm (b) 2cm (a) 8cm (b) 3cm
(c) 3cm (d) 4cm Ans. b (c) 2.5cm (d) 5cm Ans. d
DEEPIKA MA’AM # 8743011101 : CLASS – X : CIRCLES : INFINITY ... Think beyond...
CIRCLES LEARN MATHEMATICS BY : DEEPIKA MA’AM – 24 –
PRACTICE EXERCISE – 9
Q1. In figure, the perimeter of ABC is Q7. In figure, if AP = PB, then

(a) 30cm (b) 60cm


(a) AC = AB (b) AC = BC
(c) 45cm (d) 15cm Ans. a
(c) AQ = QC (d) AB = BC Ans. b
Q2. In figure, AP is a tangent to the circle with cen-

Y
Q8. In figure, if AP = 10cm, then BP =
tre O such that OP = 4cm and OPA = 30°.
Then, AP =

I (a) 91cm
T
(b) 127cm

n d
N o
(a) 2 2 cm (b) 2cm
(c) 119cm (d) 109cm Ans. b

I y
(c) 2 3 cm (d) 3 2 cm Ans. c

e
Q9. In figure, if PR is tangent to the circle at P and Q

B
is the centre of the circle, then POQ =

F
Q3. AP and PQ are tangents drawn from a point A
to a circle with centre O and radius 9cm. If OA
= 15cm, then AP + AQ =

n k
i
N
(a) 12cm (b) 18cm

T h
I
(c) 24cm (d) 36cm Ans. c
Q4. At one end of a diameter PQ of a circle of ra- (a) 110° (b) 100°
dius 5cm, tangent XPY is drawn to the circle. (c) 120° (d) 90° Ans. c
The length of chord AB parallel to XY and at a
Q10. In figure, if quadrilateral PQRS circuscribes a
distance of 8cm from P is
circle, then PD + QB =
(a) 5cm (b) 6cm
(c) 7cm (d) 8cm Ans. d
Q5. If PT is tangent drawn from a point P to a circle
touching it at T and O is the centre of the circle,
then OPT + POT =
(a) 30° (b) 60°
(a) PQ (b) QR
(c) 90° (d) 180° Ans. c
(c) PR (d) PS Ans. a
Q6. In the adjacent figure, if AB = 12cm, BC = 8cm
and AC = 10cm, then AD = Q11. In figure, two equals circles touch each other at
T, if QP = 4.5cm, then QR =

(a) 5cm (b) 4cm (a) 9cm (b) 18cm


(c) 6cm (d) 7cm Ans. d (c) 15cm (d) 13.5cm Ans. a
DEEPIKA MA’AM # 8743011101 : CLASS – X : CIRCLES : INFINITY ... Think beyond...
CIRCLES LEARN MATHEMATICS BY : DEEPIKA MA’AM – 25 –
PRACTICE EXERCISE – 10

Q1. In figure, APB is a tangent to a circle with cen-


tre O at point P. If QPB = 50°, then the mea-
sure of POQ is

(a) 4cm (b) 6cm


(c) 8cm (d) 10cm Ans. c
Q6. In figure, there are two concentric circles with
centre O. PR and PQS are tangents to the inner

Y
(a) 100° (b) 120° circle from point lying on the outer circle. If PR
(c) 140° (d) 150° Ans. a = 7.5cm, then PS is equal to

T
Q2. In figure, if tangents PA and PB are drawn to a

I
circle such that APB = 30° and chord AC is
drawn parallel to the tangent PB, then ABC =

n d
N yo
(a) 10cm (b) 12cm

I
(c) 15cm (d) 18cm Ans. c

B e
Q7. If the angle between two radii of a circle is 1300,

F
then the angle between the tangents at the ends

nk
(a) 60° (b) 90° of the radii is Ans. b
(c) 30° (d) none of these Ans.c

hi
(a) 650 (b) 500

N
Q3. In figure, PR =
(c) 400 (d) 700

T
I
A
T

1300
O

B
(a) 20cm (b) 26cm
(c) 24cm (d) 28cm Ans. b Q8. If PA and PB are tangents to the circle with centre
Q4. Two circles of same radii r and centres O and O such that APB  500 . Then, OAB  ?
O' touch each other at P as shown in figure. If (a) 250 (b) 300 (c) 400 (d) 500 Ans. a
OO' is produced to meet the circle C(O', r) at A A

and AT is a tangent to the circle C(O, r) such


that O'Q AT. Then AO : AO' = P
0
50 O

Q9. In the given figure, O is the centre of a circle and


PT is the tangent to the circle. If PQ is a chord
such that QPT  500 , then POQ  ?
(a) 3/2 (b) 2 (a) 1000 (b) 900 (c) 800 (d) 750 Ans. a
(c) 3 (d) 1/4 Ans. c P T
0
Q5. Two concentric circles of radii 3cm and 5cm are 50

given. Then length of chord BC which touches O


Q
the inner circle at P is equal to

DEEPIKA MA’AM # 8743011101 : CLASS – X : CIRCLES : INFINITY ... Think beyond...


CIRCLES LEARN MATHEMATICS BY : DEEPIKA MA’AM – 26 –
PRACTICE EXERCISE – 11
Q1. In given figure, two circles of diameters M
R N
2cm, and 4cm are given. If the two line x
a C
c
D y
S b
segments PR and ST cut each other at right A P B
a b
angle. Find the length of PR and ST.
Ans : 3cm
P
T Q8. TA and TB are tangent to a circle with
centre O from an external point T. If OT
90º intersects the circle in P, prove that AP
bisects TAB.
S
R Q9. In the given fig., O is the centre of circle.
Q2. In given figure, AC is the diameter of length
DE is the tangent at D and DOC is the

Y
5cm of the circle with centre O. If length
diameter. AE is another straight line such
of tangent segment AB = 12cm, find SB.
that EB = EP, B being on the circle and

T
Ans : 144/13
ED = 2r, show that EP2  ED  PD.
C

I
C

d
E B O
A
S r

n
P

N o
D
A 12cm B

I
Q10. The radii of two concentric circles are 13 cm

y
Q3. Two circles of radii 6cm and 14cm have

e
and 8 cm. AB is a diameter of the bigger circle.
their centres 30cm apart. Find the radius
BD is a tangent to the smaller cirle touching it at

B
F
of the smallest circle touching both of them
D. Find the length AD. Ans.9 cm
and enclosing a smaller circle.
Ans : 11cm

n k A

i
Q4. In the given fig. PT is a tangent and PAB O

N h
is a secant prove that PA  PB . B E

T
D

I
T
Q11. Find the locus of the centres of circles which
touch a givenline at a given point.

P
A B O’
O
Q6. Two circles of radii 25cm and 9cm touch
each other externally. Find the length of A P B

direct tangents. Ans : 30cm Q12. In fig. circles C O, r  and C O ' r / 2 touch in-
Q6. AB is a line segment and M is its mid-point.
Semi circles are drawn with AM, MB and ternally at a point A and AB is a chord of the
AB as diameter on the side of the line AB. cirlce C O, r  intersecting C O ', r / 2 at C.
A circle C  O, r  is drawn so that it touches Prove that AC = CB.
all the three semi circles. Prove that
O
1 B
r  AB . O’ C
6
Q7. Two circles with radii a and b touch each A

other externally. Let C be the radius of a Q13. In two concentric circles, prove that all chords of
circle which touches these two circles as the outer circle which touch the inner are of equal
well as a common tangent to the two lenth.
circles. B D

1 1 1
Prove that :   M N
C a b O

A C

DEEPIKA MA’AM # 8743011101 : CLASS – X : CIRCLES : INFINITY ... Think beyond...


CIRCLES LEARN MATHEMATICS BY : DEEPIKA MA’AM – 27 –
PRACTICE EXERCISE – 12
Q1. Let A be one point of intersection of two inter- T
P
secting circles with centres O and Q. The tan- R
gents at A tothe two circles meet the circles again
O Q
at B and C, respectively. Let the point P be lo-
cated so that AOPQ is a parallelogram. Prove
that P is the circumcentre of the triangle ABC.
Q7. In fig. BC is a tangent to the circle with centre
O. OE bisects AP. Prove that AEO ~ ABC .
A
A
O Q

Y
P E
C
B O
P
Q2. Two circles with centres A and B of radii 3cm

T
and 4cm respectiely intersect at two points C and B C

I
D such that AC and BC are tangents to the two Q8. In fig., common tangents PQ and RS two circles
circles. Find the length of the common chord CD. intersect at A. Prove that PQ = RS.

d
C

n
N
P S

o
A P B

I y
A
D

e
R Q

Q3. If an sosceles triangle ABC in which AB = AC =

B
F
6 cm is inscribed in a circle of radius 9 cm, find Q9. Equal circles with centres O and O’ touch each

k other at X. OO’ produced to meet a circle with


2
length of chord BC. Ans. 8 2 cm

i n centre O’, at A. AC is a tangent to the circle

N
A

h
B 6c
m 6c
m
C whose centre is O. O’D is perpendicular to AC.

T
I
6c
P
m

O
DO'
Find the value of . Ans. 1/3
CO
C
Q4. O is the centre of a circle of radius 5cm. T is a
D
point such that OT = 13 cm and OT intersects A
O’ X O
the circle at E. If AB is the tangent to the circle
at E, find length of AB. Ans. 20/3 cm
P
Q10. In fig. OQ : PQ = 3 : 4 and perimeter of
A 12
5cm POQ  60cm . Determine PQ, QR and OP..
5
O T P Q
E
5cm
B
Q
O

Q5. AB is a diameter of a circle. P is a point on the


semi-circle APB. AH and BK are perpendicu- R

lars from A and B respectively to the tangent at Q11. In the given fig. O is the centre of a circle
P. Prove that AH + BK = AB. with radius 3cm and PT is the tangent to
H
P
circle at T. If PT = 4 2 and PA = 4cm,
K find the distance. of AB from the centre
r
r
A O
r
M
of the circle. Ans. 5cm
B

T
4 2
Q6. In fig. PO  QO . The tangents to the circle at P
and Q intersect at a point T. Prove that PQ and P
4cm A B
OT are right bisectors of each other.
DEEPIKA MA’AM # 8743011101 : CLASS – X : CIRCLES : INFINITY ... Think beyond...
CIRCLES LEARN MATHEMATICS BY : DEEPIKA MA’AM – 28 –
PRACTICE EXERCISE – 13
intersect PQ at A and RS at B. Prove that
Q1. The lengths of two tangents drawn from an
: AOB  90 0
external point to a circle are equal.
Q10. Prove that the parallelogram circum serbinng
A
circle is a rohmbus.
O
D C

P B S O Q
Q2. If two tangents are drawn to a circle from
an External point then they subtends equal
angles at the centre and equally inclined with A P B

the line segment join fr om point. Q11. A circle touches the sides of a quadrilateral

Y
ABCD at P, Q, R and S respectively show
Q3. A chord AB of the larger of the two that the angles subtended at the centre by
concentric circles is tangent to the smaller a pair of opposite sides are supplementary.

T
circle at the point C. Show that C is the
mid-point of the chord AB. NCERT

I
Q12. A and PB are two tangents drawn from a
Q4. Two tangents TP and TQ are drawn to a point P to a circle with centre O touching
circle with centre O from an external pont at A and B, prove that OP is the perpendicular
T. Prove that  PTQ  2 OPQ . bisector of AB.

n d
N
A

o
P
T

I
90-x

y
x
P 1 O

e
90-x 2 K
O

B
F
Q
B
Q13. In the given fig. the incircle of ABC touches

k
Q5. PQ is a chord of length 8cm of a circle of
radius 5cm. The tangents at P and Q intersect the sides BC, CA and AB at D, E and F
at a point T (see fig.). Find the length TP.

i n respectively prove that :

N h
Q6. Prove that the perpendicular at the point of 1
AF  BD  CE 

T
(Perimeter of ABC )

I
contact to the tangent to a circle passes 2
through the centre. NCERT A

x
O F E

A P B

Q7. Two concentric circles are of radius 5cm and B


D
C

3cm. Find the length of the chord of the larger Q14. Given two concentric circle of radius a and
b where a > b, find the length of a chord
circle which touches the smaller circle. of larger circle which touches the other.

O
O
3 5 a
b
A P B A C B

Q8. A quadrilateral ABCD is drawn to circumscribe


Q15. In the given fig. the length of OP is equal
a circle prove that AB  CD  AD  BC . to diameter of a circle. Prove that PAB
D
R
C
is an equilateral triangle.
A

S Q r
Q
P O
S

A B
P B

Q9. In the given fig. PQ and RS are two parallel Q16. Two radii of two concentric circles are 13cm
tangents to a circle with centre O and and 8 cm. AB is a diameter of the bigger
another tangent XY, with point of contact C
DEEPIKA MA’AM # 8743011101 : CLASS – X : CIRCLES : INFINITY ... Think beyond...
CIRCLES LEARN MATHEMATICS BY : DEEPIKA MA’AM – 29 –
circle. BD is a tangent to the smaller circle B
touching it at D. Find the lenth of AD.
O'
A
O

A T P
O Q25. In the given fig. XP and XQ are tangents
B E
\ from X to the circle with centre O. R is
D a point on the circle. Prove that PX = QX
= ½. Perimeter of ABC .
Q17. A circle inscribed in a ABC , and touching
the sides AB, BC and AC at points, P, Q and P
A
R respectively if AP = 2 cm, BQ = 4 cm
and CR = 3 cm, find the perimeter of O R X

Y
ABC .
B
Q18. If PA and PB are tangents from an outside Q

point P such that PA = 10cm and PAB  600.

T
Ans : 8cm, 9.5cm, 8.5cm
Find the length of chord AB. Q26. Q9.In the given fig. two circles touches

I
externally at point P. T is a point on common
Q19. From an external point P, tangents PA and tangent AB. Prove that TA = TB or T is the
PB are drawn to a circle with centre O. If mid point AB.
CD is a third tangent is a touches to the A

n d
N
T
circle at a point E and PA = 14cm, find the B

perimeter of PCD. Ans : 28cm


o
I y
P
O O'
Q20. A circle inscribd in a right angled triangle
ABC right angle at B . If AB = 6 cm

B e
F
Q27. In the given fig. If AB = AC, prove that
BC = 8 cm. Find the radius of the circle. BE = EC.

k
Ans. 2cm.
A

n
Q21. With the vertices of a triangle ABC as

i
N
centres, three circles are described, each

h
touching the other two externally. If the sides

T
I
D F
of the triangles are 9cm, 7cm and 6cm, find O
the radii of the circles.
B C
Ans : 4cm, 5cm, 2cm E

Q28. In the given fig., XP and XQ are tangents


Q22. A circle draw Ex. of a ABC, touching the from X to the circle with centre O. R is
sides BC at point P, and the sides AB and a point on the circle. Prove that XA + AR
A at points Q and R respectively by producing. = XB + BR.
If BQ = 5 cm, CR = 7 cm, AB = 9 cm, P
and AC = 7cm. Find the perimeter of A

ABC . Ans. 28cm O R X

Q23. In the given fig. ABCD is a quadrilateral, B


Q
touching the sides, AB, BC, CD and AD at
points Q, R, S and P respectively. Such that Q29. In the given fig. two circles touches externally
0 at point P. T is a point on common tangent
DAB  90 , it CS = 27 cm and BC = 38
AB. Prove that APB  900 .
cm and radius of the circle is 10 cm, find
AB. Ans. 21 cm A T
B
D S C
P
R O O'
P O

Q30. In the given fig. if AP = 2 cm then find


A Q B the length of AB. Ans. 4cm
Q24. In the given fig. two concentric circles with A B
centre O. If PA and PB are two tangents
from a point P on common tangent PT. Prove
O O'
that PA = PB.
P
DEEPIKA MA’AM # 8743011101 : CLASS – X : CIRCLES : INFINITY ... Think beyond...
CIRCLES LEARN MATHEMATICS BY : DEEPIKA MA’AM – 30 –
Q31. In the given fig. a circle is inscribed in a
Q
quadrilateral ABCD in which B  900 . If
O
AD = 23 cm, AB = 29 cm and DS = 5 cm,
find the radius (r) of the circle. R

Ans. r = 11 cm. 60
0

A A P T

R Q
Q39. In the given figure, O is the centre of a circle,
B BOA is its diameter and the tangent at the point P
D O
meets BA extended at T. If PBO  300 , find
S P
PTA . Ans. PTA  30 0
C
Q32. In the given fig. O is the centre of two P

Y
concentric circles of radii 5 cm and 3 cm,
from an external point P. PA and PB are two B
30
0

T
tangent drawn to the circle. If PA=12 cm, O A

T
then find find then lenth of PB.

I
Ans. 4 10 cm.
Q40. In the given figure, O is the centre of a circle;
A PQL and PRM are the tangents at the points Q and
R respectively and S is a point on the circle such

n d
N
P
that SQL  500 and SRM  600 , find

o
O

I y
B
QSR . Ans. QSR  700
Q33. Prove that the segment joining the points of

B e
F
L
centact of two parallel tangent passes through
the centre of the circle.

k
0
S 50 Q

Q34. In the concentric circles, prove that all

i n O

N
chords of the outer circle which touch inner

h
0
60

are of equal length.

T
R

I
M

Q35. In the given figure find the value of x and y.


Q41. Prove that the tangents at the extremies of any
chord make equal angles with the chord.
300
Q42. Two tangent segments PA and PB are drawn to a
circle with centre O such that APB  1200 .
Prove that OP = 2AP.
[Ans. x = 30°, y = 60°] Q43. In the given fig., AB is the diameter of the circle,
Q36. In the given figure, find CAB. [Ans. 30°] with centre O, and AP is a tangent. Find the vol-
ume x. Ans. : 58º
C
B

O
64º K

Q37. f PA and PB are two tangents to a circle with cen- x


P
A
0
tre O such that APB  80 , find AOP .
Q44. In the gi ve n fi g. p r o ve t ha t
Ans. 500 0
A BAC  CAD  90
D
P O A

B O C

Q38. In the given figure, PQ is a chord of a circle with P


centre O and PT is tangent at P such that B

0
QPT  60 . Find PRQ . Ans.1200
DEEPIKA MA’AM # 8743011101 : CLASS – X : CIRCLES : INFINITY ... Think beyond...
CIRCLES LEARN MATHEMATICS BY : DEEPIKA MA’AM – 31 –
SELF EVALUATION TEST SERIES
M.M. : 20 Time : 45 Min M.M. : 20 Time : 45 Min

TEST - 1 TEST - 2
Q1. In the given fig cricle touches the side BC Q1. In the fig., given below,
cm D
27
of A  ABC at P, and touches AB and AC quadrilateral ABCD is P
38cm
A
product at Q and R respectively show that c i r c u ms c r i b ed a nd
10cm O
AQ  1/ 2 (Perimeter of  ABC ) 3M AB  BC; find x if radius S Q
A
of incircle is 10cm. 3M
B R C

Y
B P C [Ans : 21 cm] x

Q R

O
Q2. In fig., if TP and TQ are the

T
two tangents to a circle with T

c en t r e O so tha t

I
Q2. Prove that the tangents drawn at the ends
0 P 1 2 Q
of a chord of a cirlce make equal angles POQ 110 , then PTQ is 110º

equal to :

d
O
with the chord. 3M

n
(a) 60º (b) 70º

N
Q3. A circle touches the sides of a quadrilateral

o
ABCD at P,Q and R,S respectively show (c) 80º (d) 90º 3M

I y
that the angles subtanded at the centre by Q3. A circle is inscribed in a  ABC touching
a p a i r o f o p p o s i t e s i d es a r e

B e
AB, BC and AC at the points D, E & F.

F
supplementry. 3M If AB = 8 cm, BC = 10 cm and AC =
Q4. In the give fig  ABC is circumseribed.

n k 12 cm, find the length of AD, BE and CF.

i
The circle touches the sides AB, BC and [Ans : 5, 3 , 7 cm] 3M

N h
CA at P, Q and R respectively. If AP = Q4. All the sides of a parallelogram touch a

T
I
5cm, BP = 7cm, AC = 14cm and BC = circle, show that the parallelogram is a
xcm, find the value of x. [Ans : 16 cm] 3M rhombus. 3M
A
5 R
Q5. A quadrilateral ABCD is
R
D C
P drawn to circumscribe a
7
circle pr ove tha t
B C S Q
Q
Q5. In the given fig. the
AB  CD  AD  BC . 4M
A
length of OP is equal to A
P
B
r
Q6. In the fig., (i) given
diameter of a circle. P
Q
S
O
below, ABC is a right C
Prove that PAB is an angled triangle at A with
equilateral triangle. 4M B
sides AB = 5cm and
BC = 13cm. A cricle 13cm
Q6. Prove that the angle with centre O has been
T

between the two tangents inscribed in the triangle


O
O ABC. Calculate the
drawn from an external P radius of the incircle. A 5cm B
point to a circle is S Ans. 2cm 4M
supplementary to the
angle subtended by the line-segment joining
the points of contact at the centre. 4M

RESULTS : SELF CHECKING PLAN RESULTS : SELF CHECKING PLAN


Q.No. 1 2 3 4 5 6 7 8 Total Per.(%) Grade Q.No. 1 2 3 4 5 6 7 8 Total Per.(%) Grade

Marks Marks

DEEPIKA MA’AM # 8743011101 : CLASS – X : CIRCLES : INFINITY ... Think beyond...


CIRCLES LEARN MATHEMATICS BY : DEEPIKA MA’AM – 32 –
SELF EVALUATION TEST SERIES
M.M. : 20 Time : 45 Min M.M. : 20 Time : 45 Min

TEST - 3 TEST - 4
Q1. Prove that the angle between the two Q1. In the figure (ii) given
tangents drawn from an external point to A
below, two concentric
a circle is supplementary to the angle P
subtended by the line-segment joining the circles with the centre
O
O are of radii 5cm and
points of contact at the centre. 3M B
3cm. From an external
T
point P, tangents PA

Y
and PB are drawn to these circles.If PA
O
P = 12cm, find BP. 3M

T
S
Q2. With the vertices of a  ABC as centres,

I
Q2. In the fig two cencentric circles are of radii three circles are discribed each touching the
13cm and 5cm. Find the length of the chord other two externally. If the sides of the
of the outer circle which touch the inner

d
triangle are 4cm, 6cm, and 8cm. Find the
radii of the circles. [Ans : 5, 1, 3] 3M

n
N
circle. [Ans : 24cm] 3M

o
Q3. From a Point P, 10cm away from the centre

I y
of a circle, a Tangent PT of length 8cm

e
m
0 is drawn. Find the radius of the circle.
13c

B
A B [Ans : 6] 3M

F
6

Q4. In the fig., give below, the


Q3. Two circles touch externally. The sum of

n k incircle of ABC touches A

i
that areas is 130 cm2 and the distance the sides BC, CA and AB at

N h
between their centres is 14cm. Find the points P, Q and R respectively.
radii of the circles. 3M

T
I
R
If AB = AC, prove that BP Q
[Ans : R = 11, r = 3]
= PC. 3M
Q4. Two circles touch internally at a point P. B P C

From a point T on the common tangent at


P, tangent segment TQ and TR are drawn Q5. In the given fig. a circle
to the two circles. Prove that TQ = TR.3M i s i n s c r i b ed i n a A
Q5. Prove that the perpendicular at the point of quadrilateral ABCD in R Q
contact to the tangent to a circle passes B
through the centre. 4M which B  900 . If AD = D
23 cm, AB = 29 cm and S P
x DS = 5 cm, find the C
O
radius (r) of the circle.
4M
A P B

Q6. Prove that the tangents drawn at the ends


points of a diameter are parallel. 4M Q6. If the angle between A
l m
two radii of a circle T

is 1300, then the angle 1300 O


between the tangents
1
A
O
2
B at the ends of the B

radii is. Ans. 500


4M

RESULTS : SELF CHECKING PLAN RESULTS : SELF CHECKING PLAN


Q.No. 1 2 3 4 5 6 7 8 Total Per.(%) Grade Q.No. 1 2 3 4 5 6 7 8 Total Per.(%) Grade

Marks Marks

DEEPIKA MA’AM # 8743011101 : CLASS – X : CIRCLES : INFINITY ... Think beyond...


SURFACE, AREA & VOLUME LEARN MATHEMATICS BY : DEEPIKA MA’AM – 33 –

CH – 13 – SURFACE, AREA AND VOLUME


IMPORTANT FORMULAS
2
1. Cuboid : Let length, Breadth and Height of (v) Volume of cylinder = r h
cuboid be , b and h respectively, then = (base area × height)
G F
4. Hollow Cylinder : Let the inner radius and
H
E outer radius of the hollow cylinder be ‘r’ and
‘R’ & height be ‘h’ then
D h C

Y
R
A r
L B
(i) Lateral surface area of cuboid = 2h(  b)

T
(ii) Base Area of cuboid =   b

I
(iii) Total surface area of cuboid h
= 2[  b  b  h    h]
(iv) Largest diagonal of cuboid =  2  b2  h 2

n d
N o
(v) Volume of cuboid =   b  h r

I y
= [Base Area × height] R

2. Cube : Let the side of cube be x unit then

B e
F
 2
(i) Base Area =  R  r
2

k
(ii) Inner C.S.A. = 2rh

n
hi
(iii) Outer C.S.A. = 2Rh

N
2
(iv) T.S.A. = 2rh  2Rh  2 R  r
2
 
T
I
= 2  R  r  h  R  r 
 2
(v) Volume = h R  r
2

4. Cone : Let radius, height and slant height of a
(i) Lateral surface of cube = 4x 2
cone be ‘r’, h and  be respectively then
(ii) Base Area of cube = x 2
(iii) Total surface of cube = 6x 2
(iv) Largest diagonal of cube = 3x
(v) Volume of cube = x 3
(vi) Side of a cube = 3 volume
3. Cylinder : Let the heiht and radius of the cylin-
(i) Base area of a cone = r 2
der be ‘h’ and ‘r’ respectively then
(ii) Circumsference of base = 2r
(iii) Curve Surface Area = r
where   h 2  r 2
2
(iv) T.S.A. = r  r = r(  r)
1 2
(v) Volume of a cone = r h
2 3
(i) Base area of cylinder = r
(ii) Circumference of base of cylinder = 2r 1
(Base area  height)
=
(iii) Curved surface area of cylinder = 2rh 3
(iv) Total surface area of cylinder 5. Solid sphere : Let r be the radius of hemisphere
= 2rh  2r 2  2r(h  r) then
DEEPIKA MA’AM # 8743011101 : CLASS – X : SURFACE, AREA & VOLUME : INFINITY ... Think beyond...
SURFACE, AREA & VOLUME LEARN MATHEMATICS BY : DEEPIKA MA’AM – 34 –

9. Frustum : Let radius of bottom & top be R and


r respectively. h be the perpendicular height & 
be the slant height of the frustum, then
(i) Curved Surface Area = 4r 2 r

4 3
r
(ii) Volume of solid sphere = h 
3
6. Solid Hemisphere : Let r be the radius R
of hemisphere then
h 2
(i) Volume of Frustum = 
R  R  r  r2 

Y
3

T
(ii) Curved Surface Area =   R  r 

I
(i) Curved Surface Area = 2r 2 2 2
where,   h   R  r 
2
(ii) Total Surface Area = 3r

d
(iii) Total Surface Area of Frustum

n
2 3

N
2 2
(iii) Volume of hemisphere = r =   R  r   r  R

o
3

I y
7. Hollow sphere : Let outer and inner radius of

e
(iv) Total Surface Area of Bucket

B
hollow sphere be R and r respectively then

F
2
=   R  r   r

k
(v) Surface area of Bucket

n
i
   R  r   r 2

I N
(i) Volume of hollow sphere

=
4
3
4 3 4 3
3
R  r
3

(R 3  r 3 )
T h
Note:
r

________________________________________
(ii) Outer Curved Surface Area = 4R2
8. Hollow hemisphere : Let outer and inner radii ________________________________________
of hollow hemisphere be ‘R’ and ‘r’ respectively ________________________________________
then
________________________________________
________________________________________
________________________________________

C.S.A. of hollow hemisphere ________________________________________


= 2R 2  2r 2  2(R 2  r 2 ) ________________________________________
2 2
T.S.A = 2R  2r  (R  r ) 2 2
________________________________________
2 2 2 2
 3R  r  (3R  r ) ________________________________________
2 3 3
Volume = (R  r ) ________________________________________
3
DEEPIKA MA’AM # 8743011101 : CLASS – X : SURFACE, AREA & VOLUME : INFINITY ... Think beyond...
SURFACE, AREA & VOLUME LEARN MATHEMATICS BY : DEEPIKA MA’AM – 35 –
PRACTICE EXERCISE – 1
Q1. Three cubes each of side 5 cm are joined cylindrical part. Calculate the surface area
end to end. Find the surface area of the of the toy if height of the conical part is
resulting cuboid. 12 cm.
Q2. The interior of a building is in the form of a Q9. A tent is in the shape of a cylinder
right circular cylinder of diameter 4.2 m and surmounted by a conical top. If the height
height 4 m surmounted by a cone. The and diameter of the cylindrical part are
vertical height of cone is 2.1 rn. Find the outer 2.1 rn and 4 m, and slant height of the top is
surface area and volume of the building. 2.8m, find the area of the canvas used for

Y
Q3. A circus tent is cylindrical upto a height of making the tent. Also find cost of canvas of
3 m and conical above it. If the diameter of the tent at the rate of Rs 500 per m2.

T
the base is 105 rn and the slant height of Q10. A tent is in the form of a right circular
conical part is 53 m, find the total canvas used cylinder surmounted by a cone. The

I
in making the tent. diameter of cylinder is 24 m. The height of
Q4. The interior of a building is in the form of

d
the cylindrical portion is 11 m while the
cylinder of diameter 4.3 m and height 3.8 m,

n
N
vertex of the cone is 16 m above the ground.

o
surmounted by a cone whose vertical angle Find the area of canvas required for the tent.

I y
is a right angle. Find the area of the surface Q11. A tent of height 77 dm is in the form of
and the volume of the building.
e
a right circular cylinder of diameter 36 m and

B
F
Q5. A tent is of the shape of a right circular eight 44 dm surmounted by a right circular
cylinder upto a height of 3 metres and then

n k
cone. Find the cost of the canvas at Rs 3.50

i
becomes a right circular cone with a per m2 .

N h
maximum height of 13.5 metres above the Q12. A toy is in the form of a cone surmounted on

T
I
ground. Calculate the cost of painting the a hemisphere. The diameter of the base and
inner side of the tent at the rate of Rs 2 per the height of the cone are 6 cm and 4 cm,
square metre, if the radius of the base is respectively Determine the surface area of
14 metres. the toy.
Q6. A solid is in the form of a cylinder with Q13. A solid is in the form of a right circular
hemispherical ends. The total height of the cylinder; with a hemisphere at one end and a
solid is 19 cm and the diameter of the cone at the other end. The radius of the
cylinder is 7 cm. Find the volume and total common base is 3.5 cm and the heights of
surface area of the solid (Use π = 22/7) the cylindrical and conical portions are 10 cm.
Q7. A solid is composed of a cylinder with and 6 cm, respectively Find the total surface
hemispherical ends. If the whole length of area of the solid.
the solid is 108 cm and the diameter of the Q14. A toy is in the shape of a right circular
hemispherical ends is 36 cm, find the cost of cylinder with a hemisphere on one end and a
polishing the surface of the solid at the rate cone on the other. The radius and height of
of 7 paise per sq. cm. (Use π = 22 / 7). the cylindrical part are 5 cm and 13 cm
Q8. A toy is in the shape of a right circular respectively The radii of the hemispherical
cylinder with a hermisphere on one end and and conical parts are the same as that of the
a cone on the other. The height and radius of cylindrical part. Find the surface area of the
the cylindrical part are 13 cm and 5 cm toy if the total height of the toy is 30 cm.
respectively. The radii of the hermispherical
and conical parts are the same as that of the

DEEPIKA MA’AM # 8743011101 : CLASS – X : SURFACE, AREA & VOLUME : INFINITY ... Think beyond...
SURFACE, AREA & VOLUME LEARN MATHEMATICS BY : DEEPIKA MA’AM – 36 –
PRACTICE EXERCISE – 2
Q1. An iron pillar has some part in the form of a Q8. A solid toy is in the form of a right circular
right circular cylinder and remaining in the form cylinder with a hemispherical shape at one end
of a right circular cone. The radius of the base and a cone ut the other end. Their common
of each of cone and cylinder is 8 cm. The diameter is 4.2 cm and the height of the
cylindrical part is 240 cm high and the conical cylindrical and conical portions are 12 cm and 7
part is 36 cm high, Find the of the pillar if one cm respectively. Find the volume of the solid
cubic cm of iron weighs 7.8 grams. toy. (Use π = 22/ 7)
Q2. A cylindrical tub of radius 5cm and length 9.8cm Q9. A solid is in the shape of a cone standing on a
is full of water. A solid in the form of a right hemisphere with both their radii being equal to

Y
circular cone mounted on a hemisphere is 1 cm and the height of the cone is equal to its
immersed in the tub. If the radius of the radius. Find the volume of the solid in terms of

T
hemisphere is immersed in the tub. If the radius π.

I
of the hemisphere is 3.5cm and height of the Q10. A petrol tank is a cylinder of base diameter 21
cone outside the hemisphere is 5cm, find the cm and length 18 cm fitted with conical ends
volume of the water left in the tub. (Take
d
each of axis length 9 cm. Determine the

n
N o
  22 / 7 ) capacity of the tank.

I y
Q3. Find the volume of the largest right circular cone Q11. A boiler is in the form of a cylinder 2 m long
that can be cut out of a cube whose edge is 9
e
with hemispherical ends each of 2 metre

B
F
cm. diameter. Find the volume of the boiler.

k
Q4. A solid wooden toy is in the shape of a right Q12. A solid iron pole having cylindrical portion 110

n
circular cone mounted on a hemisphere. If the cm high and of base diameter 12 cm is

i
N h
radius of the hemisphere is 4.2 cm and the total surmounted by a cone 9 cm high Find the mass

T
I
height of the toy is 10.2 cm, find the volume of of the pole, given that the mass of 1 cm3 of iron
the wooden toy. is 8 gm.
Q5. A solid is in the form of a right circular cone Q13. A solid toy is in the form of a hemisphere
mounted on a hemisphere. The radius of the surmounted by a right circular cone. Height of
hemisphere is 3.5 cm and the height of the cone the cone is 2 cm and the diameter of the base is
is 4 cm. The solid is placed in a cylindrical tub, 4 cm. If a right circular cylinder circumscribes
full of water, in such a way that the whole solid the toy find how much more space it will cover.
is submerged in water. If the radius of the Q14. A solid consisting of a right circular cone of
cylinder is 5 cm and its height is 10.5 cm, find height 120 cm and radius 60 cm standing on a
the volume of water left in the cylindrical tub. hemisphere of radius 60 cm is placed upright in
Q6. A toy is in the form of a hemisphere surmounted a right circular cylinder full of water such that it
by a right circular cone of the same base radius touches the bottoms. Find the volume of water
as that of the hemisphere. If the radius of the left in the cylinder, if the radius of the cylinder is
base of the cone is 21cm and its volume is 2/3 60 cm and its height is 180 cm.
of the volume of the hemisphere, calculate the \Q15. A cylindrical vessel of diameter 14 cm and height
height of the cone and the surface area of the 42 cm is fixed symmetrically inside a similar
toy. (Use   22 / 7 ) vessel of diameter 16 cm and height 42 cm. The
Q7. A vessel is in the form of a hemispherical bowl total space between the two vessel is filled with
mounted by a hollow cylinder. The diameter of cork dust for heat insulation purposes. How many
the sphere is 14 cm and the total height of the cubic centimeters of cork dust will be required?
vessel is 13 cm. Find its capacity.

DEEPIKA MA’AM # 8743011101 : CLASS – X : SURFACE, AREA & VOLUME : INFINITY ... Think beyond...
SURFACE, AREA & VOLUME LEARN MATHEMATICS BY : DEEPIKA MA’AM – 37 –

PRACTICE EXERCISE – 3
Q1. A solid is in the shape of a cone standing on a Q7. A solid consisting of a right circular cone of height
hemisphere with both their radii being equal to 120cm and radius 60cm standing on a hemisphere
1cm and the height of the cone is equal to its of radius 60cm is placed upright circular cylinder
radius. Find the volume of the solid in terms of full of water such that it touches the bottom. Find
. the volume of water left in the cylinder, if the
Q2. Rachel, an engineering student, was asked to radius of the cylinder is 60cm and its height is
make a model shaped like a cylinder with two 180cm.
cones attached at its two ends by using a thin

Y
Q8. A spherical glass vessel has a cylindrical neck
aluminium sheet. The diameter of the model is 8cm long, 2cm diameter; the diameter of the
3cm and its length is 12cm. If each cone has a

T
spherical part is 8.5cm. By measuring the
height of 2cm, find the volume of air contained
amount of water it holds, a child finds its volume

I
in the model that Rachel made. (Assume the
to be 345 cm3. Check whether she is correct,
outer and inner dimensions of the model to be

d
taking the above as the inside measurements,
nearly the same).

n
N
and = 3.14.

o
Q3. A gulab jamun, contains sugar syrup up to about

I ey
30% of its volume. Find approximately Q9. Mayank made a bird-bath for his garden
how much syrup would be found in 45 gulab in the shape of a cylinder with a

B
F
jamuns, each shaped like a cylinder with two hemispherical depression at one end. The height
hemispherical ends with length 5cm and diam-

n k of the cylinder is 1.45m and its radius is 30cm.

i
Find the total surface area of the birth-bath.

N
eter 2.8cm.

Th
Q4. A pen stand made of wood is in the shape of a Q10. Shanta runs an industry in a shed which is

I
cuboid with four conical depressions to hold pens. in the shape of a cuboid surmounted by a
The dimensions of the cuboid are 15cm by 10cm half cylinder. If the base of the shed is of
by 3.5cm. The radius of each of the depres- dimension 7 m × 15 m, and the height of
sions is 0.5cm and the depth is 1.4cm. Find the the cuboidal portion is 8 m, find the vol
volume of wood in the entire stand. ume of air that the shed can hold. Further,
Q5. A vessel is in the form of an inverted cone. Its suppose the machinery in the shed occu
height is 8cm and the radius of its top, which is pies a total space of 300 m3 , and there are
open, is 5cm. It is filled with water upto the brim. 20 workers, each of whom occupy about
When lead shots, each of which is a sphere of 0.08 m3 space on an average. Then, how
radius 0.5cm are dropped into the vessel, one- much air is in the shed ?
fourth of the water flows out. Find the number
Q11. A juice seller was serving his customers
of lead shots dropped in the vessel.
using glasses . The inner diameter of the
Q6. A solid iron pole consists of a cylinder of height cylindrical glass was 5cm, but the bottom of
220cm and base diameter 24cm, which is the glass had a hemispherical raised portion
surmounted by another cylinder of height 60cm which reduced the capacity of the glass. If the
and radius 8cm. Find the mass of the pole, given height of a glass was 10cm, find the
that 1cm3 of iron has approxi- mately 8g mass. apparent capacity of the glass and its
[Use  = 3.14] actual capacity.

DEEPIKA MA’AM # 8743011101 : CLASS – X : SURFACE, AREA & VOLUME : INFINITY ... Think beyond...
SURFACE, AREA & VOLUME LEARN MATHEMATICS BY : DEEPIKA MA’AM – 38 –
PRACTICE EXERCISE – 4
Q.1 A well with 14 m diameter is dug 8 m deep. 7cm respectively. Find the height to which
The Earth taken out of is has been evenly the water will rise in the cylinder ?
spread all around width of 21 cm to form
an embankment. Find the height off the Q.10 A cylindrical tub of radius 16cm contains
embankment. water to a depth of 30 cm. A spherical iron
ball is dropped into the tub and thus level of
Q.2 Water flows out through a circular pipe water is raised by 9cm. What is the radius
whose internal diameter is 2 cm, at the rate of the ball ?
of 6 metres per second into a cylinder tank,
the radius of whose base is 60 cm. Find the Q.11 The diameter of a coper sphere is 18cm.

Y
rise in the level of water in 30 minutes ? The sphere is melted and is drawn into a
long wire of uniform circular cross-section.
Q.3 A well with 10 m inside diameters dug 8.4 If the length of the wire is 108m, find its

T
m deep. Earth taken out of it is spread all diameter ?

I
around it to a width of 7.5 m form an
embakment. Find the height of the Q.12 A spherical glass vessel has a cylindrical

d
embarkment. neck 8cm long, 2cm diameter; the diameter

n
of the spherical part is 8.5cm. By measuring

N o
Q.4 In the middle of a rectangular field the amount of water it holds, a child finds

I y
measuring 30 m x 20 m, a well of 7 m its volume to be 345 cm3. Check whether

e
diameter and 10 m depth is dug. The earth she is correct, taking the above as the inside

B
so removed is evenly spread over the

F
measurements, and = 3.14.
remaining part of the field. Find the height
through which the level of the field is raised. Q.13
k
A cone, a hemisphere and a cylinder stand

n
i
on equal bases and have the same height.

N
A right circular cone is 3.6cm high and

h
Q.5 Show that their volumes are in the ratio
radius of its base is 1.6cm. It is melted and

T
1:2 :3?

I
recast into a right circular cone with radius
of its base as 1.2cm. Find its height ? Q.14 Prove that the surface area of a sphere is
equal to the curved surface area of the
Q.6 Find the volume of the largest right circular circumscribed cylinder ?
cone that can be cut out of a cube whose
edge is 9cm ? Q.15 The surface area of a sphere and a cube are
equal. Prove that their volumes are in the
Q.7 Water flows at the rate of 10 metres per
ratio 1:  / 6
minute through a cylindrical pipe 5mm in
diameter. How log would it take to fill a
conical vessel whose diameter at the base Q.16 A solid sphere of radius ‘r’ is melted and
is 40cm and depth 24 cm ? recast into a hollow cylinder of uniform
thickness. If the external radius of the base
Q.8 Rain water, which falls on a flat rectangular of the cylinder is 4cm, its height 24cm and
surface of length 6m and breadth 4m is thickness 2cm, find the value of ‘r’ ?
transferred into a cylindrical vessel of
internal radius 20 cm. What will be the height Q.17 An iron spherical ball has been melted and
of water in the cylindrical vessel if a rainfall recast into smaller balls of equal size. If the
of 1cm has fallen ? radius of each of the smaller balls is 1/4 of
the radius of the original ball, how many such
Q.9 A vessel in the form of a hemispherical bowl balls are made ?
is full of water. Its contents are emptied in a Compare the surface area, of all the smaller
right circular cylinder. The internal radii of balls combined together with that of the
the bowl and the cylinder are 3.5cm and original ball ?

DEEPIKA MA’AM # 8743011101 : CLASS – X : SURFACE, AREA & VOLUME : INFINITY ... Think beyond...
SURFACE, AREA & VOLUME LEARN MATHEMATICS BY : DEEPIKA MA’AM – 39 –
PRACTICE EXERCISE – 5
Q.1 Lead spheres of diameter 6cm are dropped Q.8 A hemispherical tank full of water in
into a cylindrical beaker containing some 4
emptied by a pipe at the rate of 3 litres
water and are fully submerged. If the 7
diameter of the beaker is 18cm and water per second. How much time will it take to
rises by 40cm. Find the number of lead half-empty the tank, if the tank is 3 metres
spheres dropped in the water ? in diameter.

Q.9 Water is flowing at the rate of 5km per hour


Q.2 The height of a solid cylinder is 15cm and through a pipe of diameter 14cm into a
the diameter of its base is 7cm. Two equal rectangle tank which is 50m long and 44 m

Y
conical holes each of radius 3cm, and height wide. Determine the time in which the level
4cm are cut off. Find the volume of the of water in the tank will rise by 7 cm.
remaining solid ?

T
Q.10 A spherical shell of lead, whose external
diameter is 18 cm, is melted and recast into

I
Q.3 A conical vessel of radius 6cm. and height
8cm is completely filled with water. A sphere a right circular cylinder, whose height is 8cm
is lowered into the water and its size is such and diamter 12cm. Determine the internal
that when it touches the sides, it is just diameter of the shell.

n d
N
immersed. What fraction of water
Q.11
o
If the dimater of the cross-section of a wire

I
overflows ?

Q.4 A vessel in the shape of a cuboid contains y


is decreased by 5%, how much per cent

e
will the length be increased so that the

B
F
some water. If three indentical spheres are volume remains the same.

k
immersed in the water, the level of water is

n
increased by 2cm. If the area of the base of Q.12 The radii of the internal and external surface

i
N
the cuboid is 160cm2 and its height 12cm, of a metallic spherical shell are 3cm and 5cm,

h
determine the radius of any of the spheres ?

T
respectively.It is melted and recast into a

I
solid right circular cylinder of height
Q.5 A cylindrical container is filled with 2
10 cm. Find the diameter of the base of
icecream, whose radius 6 cm and height 15 3
the cylinder.
cm. The whole ice cream is distributed to
10 children in equal cones having
Q.13 A well of diameter 2m is dug 14m deep.
hemispherical tops if the height of the
The earth taken out of it is spread evenly all
conical portion is 4 times the radius of its
around it to a width of 5m to form an
base,find the radius of the ice-cream cone ?
embankment. Find the height of the
embankment.
Q.6 A milk tanker cylindrical in shape having
diameters and length 4.2 metres supplies
milk to the two booths in the ratio 3 : 2. one Q.14 A solid iron pole consists of a cylinder of
of the milk booth has a rectangular vessel height 220cm and base diameter 24cm,
having base area as 3.96m2 and the other which is surmounted by another cylinder of
has a cylindrical vessel having diameter height 60cm and radius 8cm. Find the mass
2 metres find the level of milk in each of the of the pole, given that 1cm3 of iron has
two vessels ? approximately 8g mass.

Q.7 An agricultural field is in the form of a Q.15 A solid consisting of a right circular cone of height
rectangle of length 20m and width 14m. A 120cm and radius 60cm standing on a hemisphere
10m deep well of diameter 7m is dug in a of radius 60cm is placed upright circular cylinder
corner of the field and the earth taken out of full of water such that it touches the bottom. Find
the well is spread evenly over the remaining the volume of water left in the cylinder, if the
part of the field. Find the rise in its level. radius of the cylinder is 60cm and its height is
180cm.

DEEPIKA MA’AM # 8743011101 : CLASS – X : SURFACE, AREA & VOLUME : INFINITY ... Think beyond...
SURFACE, AREA & VOLUME LEARN MATHEMATICS BY : DEEPIKA MA’AM – 40 –
PRACTICE EXERCISE – 6
Q.1 500 persons took dip in a rectangular tank through a cylindrical pipe having its diamter
which is 80m long and 50m broad. What is as 5mm. How much time will it take to fill a
the rise in the level of water in the tank, if conical vessel whose diameter of base is
the tank, if the average displacement of 40cm and depth 24cm.
water by a person is 0.04m3.
Q.10 A hemispherical tank of radius 13/4 m is full
Q.2 Determine the ratio of the volume of a cube of water. It is connected with a pipe which
to that of a sphere which will exactly fit empties it at the rate of 7 litres per second.
inside the cube. How much time will it take to empty the tank

Y
completery.
Q.3 A conical flask is full of water. The flask has
base-radius r and height h. The water is Q.11 A solid wooden toy in the shape of a right

T
poured into a cylindrical flask of base- circular cone mounted on a hemisphere.

I
radius mr. Find the height of water in the If the radius of the hemisphere is 4.2 cm
cylindrical flask. and the total height of the toy is 10.2 cm,

d
find the volume of the wooden toy.

n
N
Q.4 Water is being pumped out through a
circular pipe whose internal diameter is 7cm.
o
Q.12 A toy is in the form of a cone mounted on a

I y
If the flow of water is 72 cm per second, hemisphere of radius 3.5 cm. If the total
how many litres of water are being pumped
e
height of the toy is 15.5 cm, find its total

B
F
out in one hour. surface area.

Q.5 Water flows out through a circular pipe,


n k
Q.13 A toy is in the form of a cone mounted on a

i
N
whose internal diameter is 2cm, at the rate hemisphere.The diameter of the base and
of 0.7m per second into a cylindrical tank,

T h height of the cone are 6 cm and 4 cm,

I
the radius of whose base is 40cm. By how respectively. Determine the surface area of
much will the level of water rise in half an the toy.
hour.
Q.14 A solid is in the form of a cylinder with
Q.6 A rectangle tank 15m long and 11m broad hemispherical ends. The total height of the
is required to receive entire liquid contents solid is 19 cm and the diameter of the
from a full cylindrical tank of internal cylinder is 7 cm. Find the volume and
diameter 21m and length 5m. Find the least surface area of the solid.
height of the tank that will serve the
purpose. Q.15 A vessel is in the form of hemispherical bowl
mounted by a hollow cylinder. The
Q.7 Find the depth of a cylindrical tank of diameter of the hemispherial bowl is 14 cm
radius 28m, if its capacity is equal to that of and the total height of the vessel is 25 cm.
a rectangular tank of size 28m × 16m Find the capacity of the vessel.
× 11m.
Q.16 A circus tent is made of canvas and is in the
Q.8 Water in a canal, 30dm wide and 12dm form of a righ circular cylinder and a right
deep is flowing with velocity of 10km per circular cone above it. The diameter and
hour. How much area will if irrigate in 30 height of the cylinderical part of the tent are
minutes, if 8cm of standing water is required 126 m and 5 m respectively. The total height
for irrigation. of the tent is 21 m. Find the total cost of the
tent if the canvas used costs Rs. 12 per
Q.9 Water flows at the rate of 10m per minute square metre.
DEEPIKA MA’AM # 8743011101 : CLASS – X : SURFACE, AREA & VOLUME : INFINITY ... Think beyond...
SURFACE, AREA & VOLUME LEARN MATHEMATICS BY : DEEPIKA MA’AM – 41 –

PRACTICE EXERCISE – 7
Q.1 An iron pillar has some part in the form of Q.8 A solid is in the form of a right circular
a right circular cylinder and remaining in the cylinder, with a hemisphere at one end and
form of a right circular cone. The radius of a cone at the other end. The radius of the
the base of each of cone and cylinder is 8cm. common base is 3.5cm and the heights of
The cylindrically part is 240cm high and the the cylindrical and conical portions are
conical part is 36cm high. Find the weight 10cm, and 6cm, respectively. Find the total
of the pillar if one cubic cm of iron weighs surface area of the solid ?
7.8 grams ?
Q.9 A copper sphere of radius 3cm is melted

Y
Q.2 The interior of a building is in the form of and recast into a right circular cone of height
a right circular cylinder of diameter 4.2m and 3cm. Find the radius of the base of the
height 4m surmounted by a cone. The

T
cone ?
vertical height of cone is 2.1m. Find the outer

I
surface area and volume of the building ? Q.10 A tent is in the form of a right circular
cylinder surmounted by a cone. The
Q.3 A circus tent is cylindrical upto a height of
d
diameter of the base of the cylinder or the

n
N
3m and conical above it. If the diameter of cone is 24m. The height of the cylinder is
the base is 150m and the slant height of the
o
11m. If the vertex of the cone is 16m above

I y
conical part is 53m,find the total canvas used

e
the ground, find the area of the canvas
in making the tent.

B
required for making the tent ?

Q.4

F
The interior of a building is in the form of
cylinder of diameter 4.3m and height 3.8m,

N
surmounted by a cone whose vertical angle

T h i
Q.11

n kA right angled traingle, whose remaining


angles are 60o and 30o revolves about the
hypotenuse which is 84cm long. Find the

I
is a right angle. Find the area of the surface volume of the double cone thus formed ?
and the volume of the building ?
Q.12 The upper portion of a right circularcone is
Q.5 A tent is of the shape of a right circular cutt of by a plane parallel to its base. If the
cylinder upto a height of 3 metres and then 24
becomes a right circular cone with a curved surface of the remainder of the
25
maximum height of 13.5 metres above the
ground. Calculate the cost of painting the curved surface of the wholecone, find how
inner side of the tent at the rate of Rs. 2 per high is the cutting plane from the base of
square metre, if the radius of the base is cone ?
14 metres ?
Q.13 A toy is in the form of a cone mounted on a
Q.6 A solid is composed of a cylinder with hemisphere with the same radius. The
hemispherical ends. If the length of the whole diameter of the base of the conical portion
solid is 108cm. and the diameter of the is 6cm and its height is 4cm. Determine the
cylinder is 36cm, find the cost of polishing surface area of the toy. (Use  = 3.14)
the surface at the rate of 7 paise per cm2.
Q.14 A solid is composed of a cylinder with
Q.7 A toy is in the form of a cone surmounted hemispherical ends. If the whole length of
on a hemisphere. The diameter of the base the solid is 104cm and the radius of each of
and the height of the cone are 6cm and 4cm, the hemispherical ends is 7cm, find the cost
respectively. Determine the surface area of of polishing its surface at the rate of
the toy ? Re 1 per dm2.

DEEPIKA MA’AM # 8743011101 : CLASS – X : SURFACE, AREA & VOLUME : INFINITY ... Think beyond...
SURFACE, AREA & VOLUME LEARN MATHEMATICS BY : DEEPIKA MA’AM – 42 –

PRACTICE EXERCISE – 8
Q1. Two cube each of volume 64 cm3 are joined Q10. A well, whose diamter is 7m, has been dug
end to end. Find the surface area and volume 22.5m deep and the earth dugout is used to
of the resulting cuboid. Ans. 128 cm3 form an embankment around it. If the height
of the embankment is 1.5m, find the width of
Q2. The diameter of a metallic sphere is 6 cm. It
the embankment. Ans.10.5m
is melted and drawn into a wire having Q11. A cylindrical pipe has inner diameter of 7cm
diameter of the cross-section as 0.2cm. Find and water flows thought it at 192.5 litres per
the length of the wire. Ans. 36 meters minute. Find the rate of flow in kilometers per
Q3. If the diamter of cross-section of a wire is hour. Ans.3km/hr

Y
Q12. Water is being pumped out through a circular
decreased by 5% how much percent will the
pipe whose internal diameter is 7 cm. If the
length be increased so that the volume remains flow of water is 72cm per second, how many

T
the same? Ans. 10.8% litres of water are being pumped out in one

I
Q4. A sphere of diameter 6 cm is dropped in a hour? Ans.9979.2 litres
right circular cylindrical vessel partly filled with Q13. Water is flowing at the rate of 7 metres per
water. The diameter of the cylindrical vessel

n d
second through a circular pipe whose internal

N
diameter is 2cm into a cylindrical tank the

o
is 12 cm. If the sphere is completely
radius of whose base is 40 cm. Determine

I y
submerged in water, by how much will the the increase in the water level in 1/2 hour.
level of water rise in the cylindrical vessel?

B e Ans.787.5 cm

F
Ans. 1cm Q14. Water is flowing at the rate of 5 km/hr through
Q5. A hemispherical bowl of internal diamter 36cm

n k
a pipe of diameter 14cm into a rectangular
tank which is 50m long and 44m wide.

i
contains a liquid. This liquid is to be filled in

N h
Determine the time in which the level of the
cylindrical bottles of radius 3 cm and height

T
I
water in the tank will rise by 7cm. Ans. 2 hrs
6cm. How many bottles are required to empty Q15. Water in a canal, 30 dm wide and 12 dm deep
the bowl? Ans. 72 is flowing with velocity of 10 km/hr. How
Q6. A conical vessel whose internal radius is 5cm much area will it irrigate in 30 minutes, if 8cm
and height 24cm is full of water. The water is of standing water is required for irrigation?
Ans. 225000m2
emptied into a cylindrical vessel with internal
Q16. Water flows at the rate of 10 metre per minute
radius 10 cms. Find the height to which the through a cylindrical pipe having its diameter
water rises. Ans.2cm as 5 mm. How much time will it take to fill a
Q7. A glass cylinder with diameter 20cm has water conical vessel whose diameter of base is
to a height of 9cm. A metal cube of 8 cm edge 40cm and depth 24 cm? Ans.51.2 minute
is immersed in it completely. Calculate the Q17. The cost of painting the total outside surface
of a closed cylindrical oil tank at 60 paise per
height by which water will rise in the cylinder.
sq. dm is Rs.237.60. The height of the tank is
(Take   3.142 ) Ans.1.6cm 6 times the radius of the base of the tank. Final
Q8. A solid iron rectangular block of dimensions its volume correct to two decimal places.
4.4m, 2.6m and 1m is cast into a hollow Ans.509.14dm3
cylindrical pipe of internal radius 30cm and Q18. The radii of the internal and external surfaces
thickness 5 cm. Find the length of the pipe. of a metallic spherical shell are 3cm and 5 cm
Ans.112m respectively. It is melted and recast into a solid
Q9. A well with 10m inside diameter is dug 14m 2
deep. Earth taken out of it is spread all a round right circular cylinder of height 10 cm . Find
3
to a width of 5m to form an embankment. the diameter of the base of the cylinder.
Find the height of embankment. Ans.4.66m Ans. 7cm
DEEPIKA MA’AM # 8743011101 : CLASS – X : SURFACE, AREA & VOLUME : INFINITY ... Think beyond...
SURFACE, AREA & VOLUME LEARN MATHEMATICS BY : DEEPIKA MA’AM – 43 –
PRACTICE EXERCISE – 9
Q1. A solid is in the form of a cone mounted on a Q6. The altitude of a right circular cone is trisected
hemisphere in such a way that the centre of by two parallel planes drawn parallel to the
the base of the cone just coincide with the
base of the cone. The cone is cut into three
centre of the base of the hemisphere. Slant
parts. The topmost part is a right circular cone,
height of the cone is  and radius of the base
the middle cone and last one at the bottom
1
of the cone is r where as r is the radius of are two frustums. If V1 be the volume of the
2
small cone, V2 be the volume of the middle
 portion frustum and V3 be the volume of the
the hemisphere. Ans. 11r  2  r square unit.
4

Y
frustum made at the bottom, prove that
Q2. A solid is in the form of a cone of vertical
height h mounted on a right circular cylinder V1 : V2 : V3  1 : 7 :19 .

T
of height 2h and both having same radii of Q7. A right circular cone is divided by a plane
their bases. Base of the cone is placed on the parallel to its base into a small cone of volume

I
top base of the cylinder. If V cubic units be V1 at the top and a frustum of volume V2 as
the volume of the solid, prove that the radius second part at the bottom. If V1 : V2  1: 3 ,

d
find the ratio of the height of the altitude of

n
N
3V

o
of the cylinder is . small cone and that of the frustum.
7 h

I y
Q8. A conical flask is full of water. The flask has
Q3. A solid is in the form of a cone of vertical

e
base-radius a and height 2a. The water is
height h mounted on the top base of a right

B
poured into a cylindrical flask of base-radius

F
1

k
circular cylinder of height h. The 2a
3 . Find the height of water in the cylindrical

n
3

hi
circumference of the base of the cone and that

N
of the cylinder are both equal to C. If V be 3
flask. Ans. a

T
I
the volume of t he solid, prove t hat 2
Q9. A sphere of diameter 2a is dropped into a
3V cylindrical vessel partly filled with water. The
C4 .
8h
8a
Q4. Two solid cones A and B are placed in a diameter of the base of the vessel is . If
3
cylinderical tube as shown in figure. The ratio the sphere is completely submerged, by how
of their capacities is 2 : 1. Find the heights
3a
and capacities of the cones. Also find the much will the level of water rise? Ans.
4
volume of the remaining portion of the cylinder. Q10. Two solid metallic right circular cones have
Ans. 14cm, 7cm, 132cm 66cm3, 306cm3
same height h. The radii of their bases are r1
and r2 . The two cones are melted together
6cm
and recast into a right circular cylinder of height
h. Show that the radius of the base of the
21cm
1 2
Q5. The radius of the base of a right circular cone cylinder is
3
 r1  r22  .
is r. It is cut by a plane parallel to the base at
Q11. The radii of the bases of two right circular
a height h from the base. The slant height of
solid metallic cones of same height h are r1
4 and r2 . The cones are melted together and
the frustum is h 2  r 2 . Show that the
9
recast into a solid sphere of radius R. Show
13 2
volume of the frustum is r h.  R3 
27 that h  4  2 2 .
 r1  r2 
DEEPIKA MA’AM # 8743011101 : CLASS – X : SURFACE, AREA & VOLUME : INFINITY ... Think beyond...
SURFACE, AREA & VOLUME LEARN MATHEMATICS BY : DEEPIKA MA’AM – 44 –
PRACTICE EXERCISE – 10
1. The diameter of a sphere is 6 cm. It is (a) 3 (b) 4
melted and drawn into a write of diameter (c) 5 (d) 6
2 mm. the length of the wire is [Ans : c]
(a) 12 m (b) 18 m 8. A sphere of radius 6 cm is dropped into a
(c) 36 m (d) 66 m cylindrical vassel partly filled with water.
[Ans : c] The radius of the vessel is 8 cm. If the
2. A metallic sphere of radius 10.5 cm is sphere is submerged completely, then the
melted and then recast into small cones, surface fo the water rises by [Ans : a]
each of radius 3.5 cm and height 3 cm. the

Y
(a) 4.5 cm (b) 3 cm
number of such cones is [Ans : b] (c) 4 cm (d) 2 cm
(a) 63 (b) 126 9. If the radii of the circular ends of a bucket

T
(c) 21 (d) 130 of height 40 cm are of length 35 cm and 14

I
3. A solid is hemispherical at the bottom and cm, then the volume of the bucket in cubic
conical above. If the surface areas of the centimeters, is [Ans : b]
two parts are equal, then the ratio of its ra (a) 60060 (b)

n d 80080

N
dius and the height of its conical part is

o
(c) 70040 (d) 80160

I y
(a) 1:3 (b) 1: 3 10. If a cone is cut into two parts by a

(c) 1:1 (d) 3 :1


e
horizontal plane passing through the mid-

B
F
point of its axis the ratio of the valumes of

k
[Ans : b]
the up per part and the cone is

n
4. A solid sphere of radius r is melted and cast

i
(a) 1: 2 (b) 1:4

N
into the shape of a solid cone of height r 1

h
(c) 1:6 (d) 1:8
the radius of the baqse of the cone is

T
I
[Ans : d]
(a) 2r (b) 3r
11. The height of a cone is 30 cm. A small cone
(c) r (d) 4r
is cut off at the top by a plane parallel to
[Ans : a]
the base. If its volume be 1/27 of the vol
5. The material of a cone is converted into
ume of the given cone, then the height
the shape of a cylinder of equal radius. If
above the base at which the section has
height of the cylinder is 5 cm, then height
been made, is
of the cone is
(a) 10 cm (b) 15 cm
(a) 10 cm (b) 15 cm
(c) 20 cm (d) 25 cm
(c) 18 cm (d) 24 cm
[Ans : c]
[Ans : b]
12. A solid consists of a circular cylinder with
6. A circus tent is cylindrical to a height of 4
an exact fitting right circular cone placed
m conical above it. I f its diameter is 105
at the top. The heighht of the cone is h. If
m and its slant height is 40 m, the total area
the total volume of the solid is 3 times the
fo the canvas required in m2 is
volume of the cone, then the height of the
(a) 1760 (b) 2640
circular cylinder is [Ans : b]
(c) 3960 (d) 7920
(a) 2h (b) 2h/3
[Ans : d]
3h
7. The number of solid spheres, each of di (c) (d) 4h
ameter 6 cm that could be moulded to form 2
a solid metal cylinder of height 45 cm and
diameter 4 cm, is
DEEPIKA MA’AM # 8743011101 : CLASS – X : SURFACE, AREA & VOLUME : INFINITY ... Think beyond...
SURFACE, AREA & VOLUME LEARN MATHEMATICS BY : DEEPIKA MA’AM – 45 –
PRACTICE EXERCISE – 11
1. A reservoir is in the shape of a frustum of a (c) 6 cm (d) 8 cm
right circular cone. It is 8 m across at the [Ans : d]
top and 4 m across at the bottom. If it is 6 8. The volumes of two spheres are in the
m deep, then its capacity is [Ans : a] ratio 64 : 27. The ratio of their surface
(a) 176m3 (b) 196m3 areas is [Ans : d]
(c) 200m3 (d) 110m3 (a) 1:2 (b) 2:3
2. Water flows at the rate of 10 metre per (c) 9 : 16 (d) 16 : 9
minute from a cylindrical pipe 5 mm in di 9. If three metallic spheres of radii 6 cm, 8
ameter. How long will it take to fill up a

Y
cm and 10 cm are melted to form a single
concial vessel whose diameter at the base sphere, the diameter of the sphere is
is 40 cm and depth 24 cm ? [Ans : b] (a) 12 cm (b) 24 cm

T
(a) 48 minutes 15 sec (c) 30 cm (d0 36 cm

I
(b) 51 minutes 12 sec [Ans : b]
(c) 52 minutes 1 sec 10. The surface area fo a sphere is same as the
(d) 55 minutes
d
curved surface ara of a right circular cylin

n
N
3. A cylindrical vessel 32 cm high and 18 cm

o
der whose height and diameter are 12 cm

I y
as the radius of the base, is filled with send. each. The radius of the sphere is

e
this bucket is emptied on the ground and a (a) 3 cm (b) 4 cm

B
F
conical heap of sand is formed. If the height (c) 6 cm (d) 12 cm

k
of the conical heap is 24 cm, the radius of [Ans : c]

n
its base is [Ans : c]

i
11. The volume of the greates sphere that can

N
(a) 12 cm (b) 24 cm

h
be cut off from a cylindrical log of wood

T
I
(c) 36 cm (d) 48 cm of base radius 1 cm and height 5 cm is
4. The curved surface area fo a right circular
4 10
cone of height 15 cm and base diameter 16 (a)  (b) 
3 3
cm is
(a) 60  cm2 (b) 68  cm2 20
(c) 5 (d) 
(c) 120  cm 2
(d) 136  cm 2 3
[Ans : d] [Ans : a]
5. A right triangle with sides 3 cm, 4 cm and 12. A cylindrical vessel of radius 4 cm contains
5 cm is rotated about the side of 3 cm to water. A solid sphere of radius 3 cm is low
form a cone. The volume of the cone so ered into the water until it is completely
formed is [Ans : a] immersed. The water level in the vessel will
(a) 12  cm 3
(b) 15  cm 3 rise by [Ans : c]
(c) 16  cm 3
(d) 20  cm 3 (a) 2/9 cm (b) 4/9cm
6. The cured surface area of a cylinder is (c) 9/4 cm (d) 9/2 cm
264 m and its volume is 924 m . The ratio 13.
2 3 12 spheres of the same size are made from
of its diameter to its height is [Ans : b] melting a solid cylinder of 16 cm diameter
(a) 3:7 (b) 7:3 and 2 cm height. The diameter of each
(c) 6:7 (d) 7:6 sphere is
7. A cylinder with base radius of 8 cm and (a) 3 cm (b) 2 cm
height of 2 cm is melted to form a cone of (c) 3 cm (d) 4 cm
height 6 cm. The radius of the cone is [Ans : d]
(a) 4 cm (b) 5 cm
DEEPIKA MA’AM # 8743011101 : CLASS – X : SURFACE, AREA & VOLUME : INFINITY ... Think beyond...
SURFACE, AREA & VOLUME LEARN MATHEMATICS BY : DEEPIKA MA’AM – 46 –
PRACTICE EXERCISE – 12
1. A solid metallic spherical ball of diameter 3 : 5. If their heights are in the ratio 2 : 3,
6 cm is melted and recast info a cone with then the ratio of their curved surface areas
diameter of the base as 12 cm. the height is [Ans : a]
of the cone is [Ans : b] (a) 2:5 (b) 5:2
(a) 2 cm (b) 3 cm (c) 2:3 (c) 3:5
(c) 4 cm (d) 6 cm 8. A right circular cylinder of radius r and
2. A hollow sphere of internal and external height h (h = 2r) just encloses a sphere of
diameters 4 cm and 8 cm respectively is diameter [Ans : c]

Y
melted into a cone of base diameter 8 cm. (a) h (b) r
The height of the cone is [Ans : b] (c) 2r (d) 2h
(a) 12 cm (b) 14 cm 9. The radii of the circular ends of a frustum

T
(c) 15 cm (d) 18 cm are 6 cm and 14 cm. If its slant height is 10

I
3. A solid piece of iron of diamensions 49 × cm, then its vertical height is [Ans : a]
33 × 24 cm is moulded into a sphere. the (a) 6 cm (b) 8 cm
radius of the sphere is [Ans : a] (c) 4 cm (a)

n d 7 cm

N o
(a) 21 cm (b) 28 cm 10. The height and radius of the cone of which

I y
(c) 35 cm the frustum is a part are h1 and r 1 respec
(d) None of these
e
tively. If h2 and r 2 are the heights and ra

B
F
4. The ratio of lateral surface area to the total dius fo the smaller base fo the frustum re

k
surface area of a cylinder with base diam spectively and h2 : h1 = 1 : 2 then r2 : r1 is
eter 1.6 m and height 20 cm is

i
[Ans : b]
n equal [Ans : b]

N h
(a) 1:7 (b) 1:5 (a) 1:3 (b) 1:2

T
I
(c) 7: (d) 5:1 (c) 2:1 (d) 3:1
5. A solid consists of a circular cylinder sur 11. A solid frustum is of height 8 cm. If the
mounted by a right circular cone. The radii of its lower and upper ends are 3 cm
height of the cone is h. If the total height of and 9 cm respectively, then its slant height
the solid is 3 times the volume of the cone, is
then the height of the cylinder is (a) 15 cm (b) 12 cm
3h (c) 10 cm (d) 17 cm
(a) 2h (b) [Ans : c]
2
12. The radii of the ends of a bucket 16 cm
h 2h
(C) (d) high are 20 cm and 8 cm. The curved sur
2 3
face area of the bucket is [Ans : a]
[Ans : d] 2
(a) 1760 cm (b) 2240 cm2
6. The maximum volume of a cone that
(c) 880 cm2 (d) 3120 cm2
can be carved out of a solid hemisphere of
13. The diameters of the top and the bottom
radius r is [Ans : b]
portions of a bucket are 42 cm and 28 cm

2
r3 respectively. If the height fo the bucket is
(a) 3r (b) 24 cm, then the cost of painting its outer
3
surface at the rate fo 50 paise / cm2 is
r 2 (a) Rs. 1582.50 (b)Rs. 1724.50
(c) (d) 3r3
3 (c) Rs. 1683 (d) Rs. 1642
7. The radii of two cylinders are in the ratio [Ans : c]

DEEPIKA MA’AM # 8743011101 : CLASS – X : SURFACE, AREA & VOLUME : INFINITY ... Think beyond...
SURFACE, AREA & VOLUME LEARN MATHEMATICS BY : DEEPIKA MA’AM – 47 –

SELF EVALUATION TEST SERIES

M.M : 30 M.T: 60 min. M.M : 30 M.T: 60 min.

TEST - 1 TEST - 2

Q1. A toy is in the form of a cone mounted on a


Q1. A metallic sphere of radius 4.2cm is melted and
hemisphere of radius 3.5 cm. The total height
recast into the shape of a cylinder of radius 6cm.
of the toy 15.5 cm. Find the total surface area
of the toy. Ans : 214.5 cm2 Find the height of the cylinder. Ans.2.74 cm

Y
Q2. A hemispherical bowl of internal radius 9 cm Q.2 A solid is in the shape of a cone standing on a
is full of water. This water is to be filled in hemisphere with both their radii being equal

T
cylindrical bottles of diameter 3 cm and height
to 1cm and the height of the cone is equal to
4 cm. Find the number of bottles needed to

I
fill the whole water of the bowl. Ans :54 its radius. Find the volume of the solid in terms
of . Ans :  cm3
Q3. How many silver coins 1.75cm in diameter and Q.3

n d
If the radii of the ends of bucket 45cm high

N o
of thickness 2mm, must be melted to form a are 28 cm 7cm, determine its surface area.

I y
cuboid of dimensions 5.5 × 10cm × 3.5cm ?
Ans : 8063 cm2
Ans : 400 coins Q.4

B e
A solid iron pole consists of a cylinder of

F
height 220cm and base diameter 24cm, which

k
Q4. Find the number of coins 1.5cm in diameter is surmounted by another cylinder of height

n
and 0.2cm thick to be melted to form a right 60cm and radius 8cm. Find the mass of the

i
N
circular cylinder whose height is 8cm and

h
pole, given that 1cm3 of iron has approxi-
diameter 6cm. Ans : 640

T
I
mately 8g mass. Ans.892.2624kg
Q.5 Find the volume of a frustum of a cone whose
Q5. A cylindrical vessel 32 cm high and 18cm as
face radii are 7 m and 4 m, height 4 m.
the radius of the base is filled with sand. This
Ans : 389.7 cm3
is emptied on the ground and a conical heap
of sand is formed.If the height of the conical Q.6 A container shaped like a right circular cylin-
heap is 24cm, find the radius of its base. 6 cm der having diameter 12cm and height 15cm is
full of ice-cream. The ice-cream is to be filled
Q6. A circus tent is cylindrical upto a height of 3 m into cones of height 12cm and diameter 6cm,
and conical above it. If the diameter of the having a hemispherical shape on the top. Find
base is 105m and the slant height of the the number of such cones which can be filled
conical part is 53m, find the area of the with ice-cream. Ans.10 coins
canvas used in making the tent.Ans : 9735cm2 Q.7 The radii of the circular ends of a frustum of
height 6 cm are 14 cm and 6 cm respectively.
Q7. A solid sphere of radius 3 cm is melted and Find the lateral surface area and total surface
then cast into small spherical balls, each of area fo the frustum.
diameter 0.6cm. Find the number of balls thus Ans : 628.5 cm2, 157.14 cm2
obtained. Ans : 1000 \

RESULTS : SELF CHECKING PLAN RESULTS : SELF CHECKING PLAN


Q.No. 1 2 3 4 5 6 7 8 Total Per. % bg Grade
Q.No. 1 2 3 4 5 6 7 8 Total Per. % bg Grade
Marks
Marks

DEEPIKA MA’AM # 8743011101 : CLASS – X : SURFACE, AREA & VOLUME : INFINITY ... Think beyond...
SURFACE, AREA & VOLUME LEARN MATHEMATICS BY : DEEPIKA MA’AM – 48 –

SELF EVALUATION TEST SERIES

M.M : 30 M.T: 60 min. M.M : 30 M.T: 60 min.

TEST - 3 TEST - 4

Q1. A cubical block of side 7cm is surmounted by Q.1 The Metallic spheres of radii 6cm, 8cm and 10cm,
a hemisphere. What is the greatest diameter respectivley, are melted to form a single solid
sphere. Find the radius of the resulting sphere.
the hemisphere can have ? Find the surface Ans.12cm
area of the solid. Ans : 332.5 cm2 Q.2 A well of diameter 3m is dugged 14 m deep. The

Y
earth taken out of it, has been spread evenly all
around it n the shape of circular ring, with 4 m to
Q2. Water is flowing at the rate of 0.7m/s through a form an embankment. FInd the height of the em-

T
circular pipe whose internal diameter is 2 cm bankment. Ans.1.125m
into a cylindrical tank the radius of whose base

I
Q.3 A solid toy is in the form of a
is 40 cm. Determine the level of water rised hemisphere surmounted by a F A G
in cylinderical vessel in 30 minutes. right circular cone. The height
Ans : 78.7 cm of the cone is 2cm and the di-

n d
N
ameter of the base is 4 cm. B

o
O C

Determine the volume of the

I
Q3. 2 cubes each of volume 64 cm3 are joined end
to end. Find the surface area of the resulting
y
toy. If a right circular cylin-

e
der circumscribes the toy, find D E

B
F
cuboid. Ans : 160cm2 the difference of the volumes
of the cylinder and the toy. Ans : 25.12 cm3
Q4. A sphere of diameter 6 cm is dropped in a right Q.4

n k
A solid consisting of a right circular cone of height

i
120cm and rad ius 60cm standing on a hemisphere

N
circular cylinder vessel partly filled with water.

h
of radius 60cm is placed upright circular cylinder
The diameter of the cylindrical vessel is 12cm.

T
full of water such that it touches the bottom. Find

I
If the sphere is completely submerged inwater the volume of water left in the cylinder, if the ra-
by how much will the level of water rise in the dius of the cylinder is 60cm and its height is
cylindrical ressel ? Ans : 1 180cm. Ans. 1.131m
Q.5 A cone whose diameter of base 9 cm and
Q5. How many shperical bullets can be made out height 20 cm is divided into two parts by
drawing a plane through the mid point of its
of a solid cube of lead whose edge measuures axis, parallel to the base compare the volume
44cm, each bulle being 4cm in diameter. of the two parts, also find the ratio of the
Ans : 2541 volume of the upper part and the cone.Ans: 1 :7
Q.6 The height of a cone is 30 cm. A small. cone is
Q6. The perimeters of the ends of a frustum of a cut off at the top by a plane parallel to the base
right circular cone 44cm and 33cm if the hight it its volume be 1/27 of the volume of the given
of the frustum is 16cm. find its capacity. cone, at what height above the base is the
section made.
Ans : 1899.3 cm3 Q.7 A hollow cone is cut by a plane parallel to the
base and the upper portion is removed. If the
Q7. The slant height of the frustum of cone is 4 cm, curved surface of the remainder is 8/9 of the
and the perimeters of its circular ends are 18cm curved surface of the whole cone, find the ratio
and 6cm Find the cruved surface area of the of the line-segments into which the cone’s
frustem. Ans : 48 cm2 altitude is divided by the plane. Ans : 1 : 2

RESULTS : SELF CHECKING PLAN RESULTS : SELF CHECKING PLAN


Q.No. 1 2 3 4 5 6 7 8 Total Per. % bg Grade Q.No. 1 2 3 4 5 6 7 8 Total Per. % bg Grade
Marks Marks

DEEPIKA MA’AM # 8743011101 : CLASS – X : SURFACE, AREA & VOLUME : INFINITY ... Think beyond...
SURFACE, AREA & VOLUME LEARN MATHEMATICS BY : DEEPIKA MA’AM – 49 –

ANSWERS
PRACTICE EXERCISE – 1
Ans.1 350cm2 Ans.2 72.40 cm2,65.142 cm3 Ans.3 9735cm2

Ans.4 71.83 m2 Ans.5 Rs. 2068 Ans.6 418 cm2, 551.83 cm3

Ans.7 855.36 Ans.8 784.582 cm2 Ans.9 Rs. 22000

Ans.10 1320 m2 Ans.11 Rs. 5365.80 Ans.12 103.62cm2

Ans.13 372.56 cm2 Ans.14 770 cm2

Y
PRACTICE EXERCISE – 2

T
Ans.1 395.3664 kg Ans.2 616cm3 Ans.3 190.93 cm3

I
Ans.4 266.11 cm3 Ans.5 683.83 cm2 Ans.6 28cm, 5082cm2

Ans.7 1642.66 cm3 Ans.8 218.064cm3 Ans.9  cm3


220 3

n d
N
Ans.10 8316 cm3 Ans.11 m Ans.12 102.188 kg

o
21

I y
Ans.13 8  cm3 Ans.14 1.131 m3 Ans.15 1980cm3

B e
F
PRACTICE EXERCISE – 3

Ans.1  cm3 Ans.2 66cm3

n k Ans.3 338.175 cm3

i
N
Ans.4 523.53 cm3 Ans.5 100 Ans.6 892.2624 kg

T h
I
Ans.7 1.131 m3 Ans.8 346.52 cm3 Ans.9 163.54 cm3

Ans.10 827.15 m3 Ans.11 25.12 cm3

PRACTICE EXERCISE – 4
Ans.1 131.36 cm Ans.2 3m Ans.3 1.6 m

Ans.4 68.6 m Ans.5 6.4 cm Ans.6 190.92cm3

Ans.7 51 min.12 sec Ans.8 190.93 m3 Ans.9 .59 cm

Ans.10 12 cm Ans.11 6 cm Ans.12 346.52 cm3

Ans.13 16/75 cm Ans.16 6 cm Ans.17 64 balls, 4 : 1


PRACTICE EXERCISE – 5
Ans.1 90 Ans.2 502.1 cm3 Ans.3 3/8
3
280
Ans.4 cm Ans.5 3 cm Ans.6 2m, 1.68 m
11
Ans.7 1.6 m (approx) Ans.8 16.5 m Ans.9 2 hours

Ans.10 6(19)1/3 Ans.11 10.8 % Ans.12 7cm


Ans.13 0.4 m Ans.14 89.26 kg Ans.12 151.131 m3
DEEPIKA MA’AM # 8743011101 : CLASS – X : SURFACE, AREA & VOLUME : INFINITY ... Think beyond...
SURFACE, AREA & VOLUME LEARN MATHEMATICS BY : DEEPIKA MA’AM – 50 –

PRACTICE EXERCISE – 6

Ans.1 0.05 cm Ans.2 21 : 11 Ans.3 h/3m

Ans.4 9979.2 litres Ans.5 78.75 cm Ans.6 10.5 cm

Ans.7 2m Ans.8 225000 m2 Ans.9 51.2 min.

Ans.10 171 min 15 sec Ans.11 266.11 cm3 Ans.12 214.5 cm2

Ans.13 103.72 cm3 Ans.14 641.67 cm3 ; 418 cm2

Y
Ans.15 3490.67 cm3 Ans.16 Rs. 178200

PRACTICE EXERCISE – 7

T
Ans.1 395.3664 kg Ans.2 72.40 m2,65.142 m3 Ans.3 13907.14 m2

I
Ans.4 71.83 m2, 65.55m3 Ans.5 Rs. 2068 Ans.6 Rs. 863.28

Ans.7 103.62 cm2 Ans.8 372.92 cm2 Ans.9

n d6 cm

N o
Ans.10 1320 m2 Ans.11 116424 cm3 Ans.12 4 / 5th

Ans.13 103.62 cm2

F I Ans.14

i
Rs. 45.67

n k B e y

I N T h

DEEPIKA MA’AM # 8743011101 : CLASS – X : SURFACE, AREA & VOLUME : INFINITY ... Think beyond...
SAMPLE PAPER – 1 LEARN MATHEMATICS BY : DEEPIKA MA’AM – 51 –

X - MATHEMATICS
SAMPLE PAPER - 1
Time Allowed : 3 Hours] [Maximum Marks : 80
General Instructions :
1. This Question Paper has 5 Sections A - E.
2. Section A has 20 MCQs carrying 1 mark each.
3. Section B has 5 questions carrying 02 marks each.
4. Section C has 6 questions carrying 03 marks each.

Y
5. Section D has 4 questions carrying 05 marks each.

T
6. Section E has 3 case based integrated units of assessment (04 marks each)with sub-parts of the
values of 1,1 and 2 marks each respectively.

I
7. All Questions are compulsory. However an internal choice in 2 Qs. of 5 marks, 2 Qs of 3 marks
and 2 Questions of 2 marks has been provided. An internal choice has provided in the 2 marks

d
N
questions of Section E.

o n
I
8. Draw neat figures wherever required. Take  = 22/7 wherever required if not stated.

SECTION - A
e y
F B
Section A consists of 20 questions of 1 mark each.
Q1.
k
What is the largest number that divides each one of 1152 and 1664 exactly ?

in
N
(a) 32 (b) 64 (c) 128 (d) 256

h
I
The roots of the equation x2 – 3x – m(m + 3) = 0, where m is constant are

T
Q2.
(a) m, m + 3 (b) 3 + 3, – m (c) m, –(m + 3) (d) – (m + 3), – m
Q3. The numberof zeroes that polynomial f(x) = (x – 2)2 + 4 can have is / are
(a) 2 (b) 1 (c) 0 (d) 3
Q4. The pair of equations 2x – 3y = 1 and 3x – 2y – 4 has ________ solution
(a) one (b) two (c) no (d) many
Q5. A triangle with vertices (4, 0), (–1, –1) and (3, 5) is a/an
(a) equilateral triangle (b) right-angled triangle
(c) isosceles right-angled triangle (d) none of these

AB BC
Q6. In ABC and DEF,  then ABC ~ EDF, if
DE FD
(a) B = E (b) A = D (c) B = D (d) A = F

Q7. If  is an acuate angle and tan+ cot= 2, then the value of sin 3   cos3  is

1 1
(a) 1 (b) (c) (d) 2
2 2
Q8. The line segment joining the points P(–3, 2) and Q(5, 7) is divided by the y-axis in the ratio
(a) 3 : 1 (b) 3 : 4 (c) 3 : 2 (d) 3 : 5
DEEPIKA MA’AM # 8743011101 : CLASS – X : SAMPLE PAPER – 1 : INFINITY ... Think beyond...
SAMPLE PAPER – 1 LEARN MATHEMATICS BY : DEEPIKA MA’AM – 52 –

AD AE
Q9. In the given figure  and ADE = 70°, BAC = 50°, then angle BCA =
BD EC

(a) 70° (b) 50° (c) 80° (d) 60°


Q10. In the given figure, AD = 1.28 cm, BD = 2.56 cm, AE = 0.65 cm, DE will be parallel to BC,
if EC =

(a) 1.28 cm (b) 2.56 cm (c) 0.64 cm (d) 0.32 cm

Y
Q11. How many tangents can a circle have
(a) 1 (b) 2 (c) Infinity many (d) None of these

T
Q12. If the circumference and thearea of a circle are numerically equal, then the radius of the circle

I
is
(a) 2 units (b)  units (c) 4 units (d) 7 units

d
N n
Q13. The surface area of a sphere is 616 cm2, its radius is

o
I y
(a) 19 cm (b) 7 cm (c) – 7 cm (d) 14 cm

e
F
Q14. di is the deviation of x i from assumed mean a.

fi d i
k B
If mean = x  f , then x is

i n
N
i

h
I T
(a) class size (b) number of observation
(c) assumed mean (d) none of these
Q15. A toothed wheel of diameter 50 cm is attached to a smaller wheel of diameter 30 cm. How
many revolutions will the smaller wheel make when the larger one makes 15 revolutions ?
(a) 23 (b) 24 (c) 50 (d) 60
Q16. Mean of 100 items is 49. It was observed that three items which should have been 60, 70, 80
were wrongly need as 40, 20, 50 respectively. The correct mean is
(a) 48 (b) 49 (c) 50 (d) 60
Q17. 2000 tickets of a lottery were sold and there are 16 prizes on these tickets. Abhinav has
purchased one lottery ticket. The probability that Abhinav wins a prize is
(a) 10.08 (b) 00.07 (c) 0.0008 (d) 0.080

Q18. At sometimes, the length of a shadow of a tower is 3 times its height, then the angle of
elevation of the Sun, at that time is
(a) 15° (b) 30° (c) 45° (d) 60°
Direction : In the question number 19 and 20, a statement of Assertion (A) is followed by a
statement of Reason (R). Choose the correct option.
Q19. Statement A (Assertion) : The HCF of two number is 9 and their LCM is 2016. If one of
the number is 306, then the other is 54.
DEEPIKA MA’AM # 8743011101 : CLASS – X : SAMPLE PAPER – 1 : INFINITY ... Think beyond...
SAMPLE PAPER – 1 LEARN MATHEMATICS BY : DEEPIKA MA’AM – 53 –

Statement R (Reason) : For any positive integers a and b, we have : Product two numbers
= HCF × LCM.
(a) Both assertion (A) and reason (R) are true and reason(R) is the correct explanation of
assertion (A).
(b) Both assertion (A) and reason (R) are true and reason (R) is not the correct explanation
of assertion (A).
(c) Assertion (A) is true but reason (R) is false.
(d) Assertion (A) is false but reason (R) is true.
4

Y
Q20. Statement A (Assertion) : The value of sin   in not posible.
3
Statement R (Reason) : Hypotentuse is the largest side in any right angled triangle.

T
(a) Both assertion (A) and reason (R) are true and reason(R) is the correct explanation of

I
assertion (A).
(b) Both assertion (A) and reason (R) are true and reason (R) is not the correct explanation

d
N
of assertion (A).

o n
I
(c) Assertion (A) is true but reason (R) is false.
(d) Assertion (A) is false but reason (R) is true.
e y

N F SECTION - B

i n k B
Section B consists of 5 questions of 2 marks each.

h
I
Q21. Find the sum of all multiples of 7 lying between 100 and 1000.

Q22. T
In the given figure, ABCD is a trapezium in which AB || DC || EF. Show that
AE BF

ED FC
.

Q23. In the given figure, two circles touch each other at the point C. Prove that the common
tangent to the circles at C, bisects the common tangent at P and Q.

Q24. An arc of a circle of length 7 cm and the sector it bounds has an area 28 cm3. Find the
radius of the circle.
OR
The wheels of a car are of diameter 80 cm each. How many complete revolutions does each
wheel make in 10 minutes when the car is travelling at a speed of 66 km per hour ?
DEEPIKA MA’AM # 8743011101 : CLASS – X : SAMPLE PAPER – 1 : INFINITY ... Think beyond...
SAMPLE PAPER – 1 LEARN MATHEMATICS BY : DEEPIKA MA’AM – 54 –
Q25. In some buildings especially in industries, the roof is inclined. This inclination of roof is the
application of trigonometric functions. Here the roof of industry is inclined at angle  and 
with horizontal line as shown. Determine the value of sin(  ), if cos ec  2 and
cot   1, where both  and  are acute angles.

SECTION - C
Section C consists of 6 questions of 3 marks each.

T Y
Q26.

Q27.
Prove that 3  2 5 is irrational.

Solve for x :
2x

1

NI 3x  9
x  3 2x  3 (x  3)(2x  3)
 0; x  3,
3
2

o n d
I y
Q28. A cottage industry produces a certain number of pottery articles in a day. It was observed on

e
a particular day that the cost of production of each article (in rupees) was 3 more than twice

F B
the number of articles produced on that day. If the total cost of production on that day was

k
Rs. 90, find the number of articles produced and the cost of each article.

n
i
N
OR

h
I
Find the 31st term of an AP whose 11th term is 38 and the 16th term is 73.

Q29. If sec   x 
1
4x
T
, then prove that sec   tan  
1
2x
or 2x.

Q30. Prove that the line segment joining the mid-points of the adjacent sides of a quadrilateral
form a parallelogram.
OR
Prove that opposite sides of a quadrilateral circumscribing a circle subtend supplementary
angles at the centre of the circle.
Q31. A game has 8 triangles of which 6 are blue and rest are green, 12 rectangles of which 3 are
green and rest are blue, and 10 rhombuses of which 3 are blue and rest are green. One piece
is lost at random. Find the probability that it is
(i) a rectangle (ii) a triangle of green colour
(iii) a rhombus of blue colour.

SECTION - D
Section D consists of 4 questions 5 marks each.
Q32. If a line is drawn parallel to one side of a triangle, the other two sides are divided in the same
ratio, prove it.
Use the result to prove the following :
DEEPIKA MA’AM # 8743011101 : CLASS – X : SAMPLE PAPER – 1 : INFINITY ... Think beyond...
SAMPLE PAPER – 1 LEARN MATHEMATICS BY : DEEPIKA MA’AM – 55 –
AE BF
In the given figure, ABCD is a trapezium in which AB || DC || EF. Show that  .
ED FC

Q33. At t minutes past 2 p.m. the time needed by the minutes hand of a clock to show 3 p.m. was

t2

Y
found to be 3 minutes less than minutes. Find t.
4

T
OR

I
At present Asha’s age (in years) is 2 more than the square of her daughter Nisha’s age. When
Nisha grows to her mother’s present age, Asha’s age would be one year less than 10 times
the present age Nisha. Find the present age of btoh Asha and Nisha.

d
N n
Q34. A circus tent is in the shape of a cylinder surmounted by a conical top of the same diameter.

o
I
If their common diameter is 56m, the height of cylindrical part is 6m and the total height of

e y
the tent above the ground is 27m, find the area of canvas used in making the tent.

F B
OR

k
A solid is in the form of a cylinder with hemispherical ends. The total height of the solid is 20

n
cm and the diameter of the cylinder is 7 cm. Find the total volume of the solid.

i
N h
Q35. The marks of 80 students of class X in Mathematics test are given below. Find the mode of

I T
these marks obtained by the students in Mathematics test.
Marks Frequency
0  10 2
10  20 6
20  30 12
30  40 16
40  50 13
50  60 20
60  70 5
70  80 1
80  90 4
90  100 1
Total 80

SECTION - E
Case study based questions are compoulsory.
Q36. Two friends Raj and Anuj have to travel to Shimla via Chandigarh from Gurgaon. When they
reached the bus stand of Gurgaon, Raj got a call from his friend Ankit who was also on his
way to bus stand. Ankit requested Raj to buy two tickets to Chandigarh and 3 tickets to
Shimla also Anuj’s friend Kamla asked Anuj to buy 3 tickets to Chandigarh and 4 tickets to
Shimla. Raj purchased 2 tickets to Chandigarh and 3 tickets to Shimla for Rs. 3700, Anuj
DEEPIKA MA’AM # 8743011101 : CLASS – X : SAMPLE PAPER – 1 : INFINITY ... Think beyond...
SAMPLE PAPER – 1 LEARN MATHEMATICS BY : DEEPIKA MA’AM – 56 –

spent Rs. 5100 to buy 3 tickets to Chandigarh and 4 tickets to Shimla.


(i) If cost of one ticket to Chandigarh is Rs. x and cost of one ticket to Shimla is Rs. y then
represent the situation algebraically.
(ii) Find the cost of one ticket from Gurgaon to Chandigarh.
(iii) If Raj purchases 3 tickets to Chandigarh and 5 tickets to Shimla, how much amount he
will pay ?
OR
If Anuj spends Rs. 5600 to buy tickets find how many total number of tickets he pur-
chased ?

Y
Q37. Five ships are positioned in the Indian Ocean. Their positions were plotted on a graph paper
in reference to a rectangular coordinate axes.

I T d

F I N
An enemy ship is spotted at P(–5 6).

i n k B e y o n

N h
(i) What is the distance between P and E ?

I T
(ii) Find the coordinate of mid-point of BD.
(iii) Ship D is moved to a position which is mid-point of AE. Find the distance moved by D.
OR
We find a rock at new position G such that B, G and C are in a straight line and BG : GC
= 3 : 1 then fin dthe coordinates of G.

Q38. Group of friends playing with cards bearing numbers 5 to 50. All cards placed in a box and
are mixed thoroughly one friend withdrawns the card from box at random and then replace it.
Answer the questions based on above.
(i) What is the probability that the card withdrawn from the box bears a prime number less
than 10 ?
(ii) What is the probability that the card withdrawn from the box bears a number which is a
perfect square ?
(iii) What is the probability that the card withdrawn from the box bears a number which is
multiple of 7 between 40 and 50 ?
OR
Find the probabilit of drawing a card bearing number from 5 and 50.

DEEPIKA MA’AM # 8743011101 : CLASS – X : SAMPLE PAPER – 1 : INFINITY ... Think beyond...
SOLUTIONS SAMPLE PAPER – 1 LEARN MATHEMATICS BY : DEEPIKA MA’AM – 57 –

X - MATHEMATICS
SOLUTIONS : SAMPLE PAPER - 1
A-1. (c) 128 Hence, triangle is an isosceles right-
A-2. (b) x2 – 3x – m(m + 3) = 0 angled triangle.

 x2 + mx – (m + 3)x – m(m + 3) = 0 A-6. (c) In ABC and DEF

 x(x + m) – (m + 3)(x + m) = 0 AB BC
=
 (x + m)[x – (m + 3)] = 0 DE DF

Y
 x + m = 0 or x – (m + 3) = 0
 x = – m or x = m + 3

T
A-3. (c) The given polynomial is

I
f(x) = (x – 2)2 + 4 Also ABC ~ EDF
for zeroes, This is possible when D = D.

d
N
f(x) = 0 A-7. (c)

o n
I
 (x – 2)2 + 4 = 0 A-8. (d)

 (x – 2)2 = – 4
A-9. (d) 

e y
DE || BC

F B
 ABC = 70° (Corresponding s)
Which is not possible.

k
Using angle sum property of triangle
Hence the polynomial has no zeroes.

i n
N
ABC + BCA + BAC = 180°
A-4. (a) The given equations are 2x – 3y = 1

h
I
and 3x – 2y = 4  BCA = 180° – 70° – 50° = 60°

a1 2 b1 3 3
Here a = , b  2  2
3
T A-10. (a) DE || BC, if
AD AE

DB EC
2 2

1.28 0.64
a1 b1  =
 a  b 2.56 EC
2 2
 EC = 1.28 cm
The given pair of linear equations has
A-11. (c)
unique solution.
A-12. (a) Let radius of the circle be r units
A-5. (c) Let coordinates of vertices be A(4,
Circumference of the circle = 2r
0), B(–1, –1) and C(3, 5).
Area of the circle = r2
AB = (1  4)2  (1  0)2  36 A.T.Q
Circumference of the circle = Area
BC = 2 2
(3  1)  (5  1)  52 of the circle

2 2
 2r = r2
AC = (3  4)  (5  0)  26
 r = 2 units
 AB2 + AC2 = BC2 A-13. (b) Let radius of the sphere be a cm
and AB = AC  Surface area of sphere = 4a2
DEEPIKA MA’AM # 8743011101 : CLASS – X : SOLUTIONS SAMPLE PAPER – 1 : INFINITY ... Think beyond...
SOLUTIONS SAMPLE PAPER – 1 LEARN MATHEMATICS BY : DEEPIKA MA’AM – 58 –

 4 × a2 = 616  In ABC,

2 2 AB h 1
 4   a  616 tan =  
7 BC 3h 3

tan = tan 30°


616  7
 a2 =  49
  = 30°
22  4
A-19. (d) Assertion (A) is false but Reason (R)
 a = 7 cm
is true.
fi di A-20. (a) Both Assertion (A) and Reason (R)

Y
A-14. (c)  Mean = assumed mean + f
i are true and Reason (R) is the cor-
rect explanation of Assertion (A).

T
 x = assumed mean.
A-21. All multiples of 7 lying between 100 and

I
A-15. (c) Circumference of smaller wheel
1000 are 105, 112, 119,..., 994
= 30 cm
These numbers from an A.P.

d
Circumference of bigger wheel

N n
Here a = 105, d = 112 – 105 = 7
= 50 cm

o
I
Let a n = 994

y
Now, 15 × 50 = number of revolu-
tion × 30
e
F
 a + (n –1)d = 994
 number of revolutions = 25 

k B n = 128

n
A-16. (c) Sum of 100 observations

i
N
128
= 100 × 49 = 4900 Now, S128 = (105  994)

h
2

I T
Correct sum = 4900 – [40 + 20 + 50]
= 70336
+ [60 + 70 + 80] = 5000
A-22. Given : In trapezium ABCD,
5000 AB || DC || EF
 Correct mean = = 50
100 AE BF
To prove : 
A-17. (c) Number of lottery ticket = 2000 ED FC
Total number of prizes = 16
 Probability that Abhinav wins a
16 1
prize =  = 0.008
2000 125 Construction : Join AC, where point O
is intersection of AC and EF.
A-18. (b) Here AB is tower of height h m.
Proof : In ADC and AEO, EO || DC
Its shadow BC = 3h m
AE AO
 =
Let  be the angle of elevation AD AC

AD AC
 =
AE AO

ED CO
 = ...(i)
AE AO
DEEPIKA MA’AM # 8743011101 : CLASS – X : SOLUTIONS SAMPLE PAPER – 1 : INFINITY ... Think beyond...
SOLUTIONS SAMPLE PAPER – 1 LEARN MATHEMATICS BY : DEEPIKA MA’AM – 59 –

In CFO and CBA, FO || BA

CF CO
 =
BC AC
Now, length of an arc of a sector of angle
BC AC
 =
CF CO 
=  2r
360
BF AO
 =
CF CO 
 7 =  r

Y
180
CF CO
 = ...(ii)
BF AO 1260

T
 =
r

I
From (i) and (ii), we get

ED CF 
= Now, area of the sector =  r 2

d
AE BF 360


AE
ED
=
BF
CF

F I N
Hence proved.

A-23. Given : PQ and RC are common tangents


to the two circles.

i n

k B
28 =

e y o
1260
r
360

r = 8 cm
Radius of the circle 8 cm.
 r 2
n

N h
OR

I T
Given, diameter of the wheels of car =
80cm.
 Radius = 40 cm
To prove : RC bisects PQ or R bisects
PQ. Circumference of the wheel

Proof : PR and RC are tangents to a circle 22


= 2r = 2   40 cm
with centre A. 7
 PR = RC [ Length of tangents Speed of the car = 66 km/h
drawn from an external point R to a circle
Distance covere in 10 minutes
are equal] ...(i)
Similarly, RQ and RC are tangents to a 66  10
= = 11 km
circle with centre B. 60

 RQ = RC ...(ii) = 1100000 cm
From (i) and (ii), we get  Number of revolutions
PR = RQ
Total distance in 10 min utes
 R bisects PQ. Hence proved. = Circumference of the wheel
A-24. Length of are AB = 7 cm,
Let AOB =  1100000  7
= = 4375
2  22  40
DEEPIKA MA’AM # 8743011101 : CLASS – X : SOLUTIONS SAMPLE PAPER – 1 : INFINITY ... Think beyond...
SOLUTIONS SAMPLE PAPER – 1 LEARN MATHEMATICS BY : DEEPIKA MA’AM – 60 –

A-25. Given, cos ec  2 3q  p


  5
2q
1
 sin  =
2 So, our supposition is wrong.

  = 45° and cot = 1 Hence, 3  2 5 is irrational.

 tan = 1   = 45°
2x 1 3x  9
A-27.   0
 sin( + ) = sin (45° + 45°) x  3 2x  3 (x  3)(2x  3)
= sin 90° = 1

Y
OR 2x(2x  3)  x  3  3x  9
 =0
(x  3)(2x  3)
sin 6   cos6 

T
3 2 3 2
= (sin )  (cos )  4x2 + 10 + 6 = 0

I
= (sin 3   cos3 )(sin 3   cos3 )
 2x2 + 5x + 3 = 0
2 2
= (sin   cos )(sin   cos  

d
 (x + 1)(2x + 3) = 0

N n
sin  cos )  (sin   cos )

o
3

I
(sin 2   cos 2   sin  cos ) x = – 1 or x =

y

2

e
= (sin   cos )(1  sin  cos )

F B
(sin   cos )(1  sin  cos ) 3
When x = , given equation is not de-

k
2 2 2
= (sin   cos )(1  sin  cos )

in
N
(1  sin  cos ) fined.

h
I
= (sin 2   cos 2 )(1  sin 2  cos2 )  x = –1

A-26. Let us suppose that 3  2 5 is irrational. T A-28. Let total number of pottery articles pro-
duced in a particular day be x.
 3  2 5 can be written in the form 90
Cost of production per article = Rs.
p x
, where p and q are integers and q  0.
q
90
ATQ 2x + 3 =
x
p p
 3 2 5   3 2 5
q q  x(2x + 3) = 90
 2x2 + 3x = 90
3q  p 3q  p
 2 5   5  2x2 + 3x – 90 = 0
q 2q
 (2x+15)(x–6) = 0
3q  p  2x = – 15 or x – 6 = 0
Since p and q are integers, we get
2q
15
irrational, and so  x=  (rejected) or x = 6
5 is rational. 2

But this contradicts the fact that 5 is  Number of articles produced in a


particular day = 6
irrational.

DEEPIKA MA’AM # 8743011101 : CLASS – X : SOLUTIONS SAMPLE PAPER – 1 : INFINITY ... Think beyond...
SOLUTIONS SAMPLE PAPER – 1 LEARN MATHEMATICS BY : DEEPIKA MA’AM – 61 –

Cost of production per article = RHS


90  LHS = RHS
= = Rs. 15
6 A-30. Given : In a quadrilateral PQRS, A, B, C
and D are the mid-points of sides PQ, QR,
OR
RS and SP respectively.
Given a11 = 38 and a16 = 73
 a + 10d = 38
and a + 15d = 73

Y
 a + 15d – a – 10d = 73 – 38
 5d = 35 To prove : ABCD is a parallelogram.

T
 d=7
Construction : Join PR.

I
 a11 = a + 10 × 7 = 38 Proof : In PQR, A and B are mid-points
 a = 38 – 70 = – 32 of sides PQ and QR respectively.

d
N
a 31 = a + 30d  AB || PR (Using mid-point theorem)

n

o
...(i)

I
= – 32 + 30 × 7
= – 32 + 210 = 178
e y
In PSR, D and C are mid-pionts of sides

F
PS and SR respectively.

A-29. Given sec   x 


1
x
k B
 DC || PR (Using mid-point theorem)

n
...(ii)

i
N
squaring both sides, we get

h
From (i) and (ii), we get

I T
2 AB || DC
 1 
sec2 =  x  
 4x  Similarly, we have AD || BC
 In quadrilateral ABCD, AB || CD and
1 1 AD || BC.
 sec2 = x 2  
16x 2 2
 ABCD is a parallelogram, because
both pairs of opposite sides ofa quadri-
1 1
 tan2 = x 2  2
 lateral ABCD are parallel.
16x 2
OR
2 AB touches at P and BC, CD and DA
 1 
= x 
 4x  touch the circle at Q, R and S.
Construction : Join OA, OB, OC, OD
 1   1  and OP, OQ, OR, OS.
 tan  =  x   or   x  
 4x   4x 
Consider LHS = sec – tan
1 1
= x x
4x 4x

1  1  1  1 = 2 [OA bisects POS]


or x x   or 2x
4x  4x  2x Similarly, 4 = 3;
DEEPIKA MA’AM # 8743011101 : CLASS – X : SOLUTIONS SAMPLE PAPER – 1 : INFINITY ... Think beyond...
SOLUTIONS SAMPLE PAPER – 1 LEARN MATHEMATICS BY : DEEPIKA MA’AM – 62 –

5 = 6; Proof : BDE and CDE are on the same


8 = 7 base and between the same parallel BC
2[1 + 4 + 5 + 8] = 360° and DE, hence equal in area, i.e,

(1 + 8 + 4 + 5) = 180° ar(BDE) = ar(CDE) ...(i)

AOD + BOC = 180° 1


ar(ADE) 2 .AD.EL AD
Similarly AOB + COD = 180° Now, ar(BDE)  1 .BD.EL  BD
2
Hence, opposite sides of quadrilateral cir-
cumscribing a circle subtend supplemen- ...(ii)

Y
tary angles at the centre of a circle.
1
A-31. Total number of objects = 12+8+10 = 30 ar(ADE) 2 .AE.DP AE
Similarly, ar(CDE) 1 .EC.DP  EC

T
Number of blue triangles = 6 2

I
Number of green triangles = 8 – 6 = 2 ...(iii)
Number of green rectangles = 3

d
ar(ADE) ar(ADE)

N
Number of blue rectangles = 12 – 3 = 9 Also,

n
(BDE) ar(CDE)

yo
I
Number of blue rhombus = 3
[Using (i)]

e
Number of green rhombuses = 10 – 3 = 7

F B
(i) Probability that one piece lost is a AD AE
 = [From (ii) and (iii)]
BD EC
rectangle =
12 2

n k
hi
Second Part :

N
30 5
Join intersecting EF at G.

I
(ii) Probability that one piece lost is a

tiangle of green colour =


2
T
30 15
1
In DAB, EG || AB

(iii) Probability that one piece lost is a


3 1
rhombus of blue colour = 
30 10 AE BG
 = [Using B.P.T.) ...(i)
DE GD
A-32. First part :
Given : A triangle ABC In DBC, GF || DC

BG BF
 = ...(ii)
GD FC
From (i) and (ii)

AE BF
=
DE || BC, meeting AB at D and AC at E DE FC

AD AE
To Prove :  t2
DB EC A-33. ATQ (60 – t) = 3
4
Construction : Join BE, CD and draw
EL  AD.  240 – 4t = t2 – 12
DEEPIKA MA’AM # 8743011101 : CLASS – X : SOLUTIONS SAMPLE PAPER – 1 : INFINITY ... Think beyond...
SOLUTIONS SAMPLE PAPER – 1 LEARN MATHEMATICS BY : DEEPIKA MA’AM – 63 –

Curved surface area of conical part


 t2 + 4t – 252 = 0
= rl = (28)35
 t2 + 18t – 14t – 252 = 0
m2 = 980 m2
 (t + 18)(t – 14) = 0
Radius of cylindrical part = 28 m
 t = 14, – 18 [rejected]
Height of cylindrical part = 6m
 t = 14 minutes.
Curved surface area of cylindrical part
OR
= 2rh = 2(28)6
Let present age of Asha be x years
= 336 m2

Y
and present age of Nisha be ye years
Total curved surface area = 980 + 336
ATQ x = y2 + 2

T
Difference in ages = (x – y) years 1316  22
= 1316 m2 =

I
Mother’s age after (x – y) yers is 7
x + (x – y) = 10y – 1 = 4136 m2

d
N
 2x – y – 10y + 1 = 0  Area of canvas used = 4136 m2
 2(y2 + 2) – 11y + 1 = 0
o n
I
OR
 2y2 + 4 – 11y + 1 = 0
e y
F B
 2y2 – 11y + 5 = 0
 2y2 – 10y – y + 5 = 0

n k
i
N
 (y – 5)(2y – 1) = 0

h
I T
1
 y = 5 or y = (rejecting)
2 Diameter of cylinder = diameter of the
Neha’s present age = 5 years hemisphere

Asha’s present age = 52 + 2 = 27 years. = 7 cm

A-34. 7
 Radius of cylinder = cm
2
Total height of the solid = 20 cm

7 7
Height of the cylinder = 20    
2 2
Let l be the slant height of conical part of
tent. = 13 cm
Radius of conical part (r) = 28 m Volume of the solid = Volume of the
Height of conical part (h) = 21 m cylinder + 2 × vol. of one hemisphere
2 2 3 2 4 
Now, l= (28)2  (21)2 = r h  2  r  r  h  r 
3 3 

= 784  441 22 7 7  4 7
=   13    cm3
7 2 2 3 2
= 1225 = 35 m
DEEPIKA MA’AM # 8743011101 : CLASS – X : SOLUTIONS SAMPLE PAPER – 1 : INFINITY ... Think beyond...
SOLUTIONS SAMPLE PAPER – 1 LEARN MATHEMATICS BY : DEEPIKA MA’AM – 64 –

A-38. (i) Prime number from 5 to 10 are 5 and


22 7 7  14 
=     13   7 only.
7 2 2  3
 number of favourable cases = 2
= 680.167 cm3.
Total possible outcomes = 46
A-35.
P(prime number less than 10)
Marks Frequency
0  10 2 2 1
= 
10  20 6 46 23
20  30 12
(ii) Perfect squares from 5 to 50 are 9,

Y
30  40 16
40  50 13 16, 25, 36, 49
50  60 20 Number of favourable cases = 5

T
60  70 5
Total possible outcomes = 46

I
70  80 1
80  90 4 P(a perfect square number from 5 to
90  100 1

d
5

N
Total 80 50) =

n
46

o
Here, frequency of the class 50 – 60 is

I y
maximum. (iii) Multiple of 7 between 40 and 50 are
42 and 49
e
F B
 Modal class is 50 – 60
Number of favourable outcomes = 2

k
Also, l = 50, f0  13, f1  20, f 2  5,
Total possible outcomes = 46

in
N
h = 10
P(multiple of 7 between 40 and 50)

h
I T
 f1  f0  2 1
 Mode = l    h = 
 2f1  f0  f 2  46 23
OR
 20  13 
= 50     10
 2  20  13  5  2 1
P(from 5 and 50) = 
7 46 23
= 50   10
22
= 50 + 3.18 = 53.18
So, the mode marks are 53.18.

A-36. (i) 40 km
(ii) (5, 1)

50  15 
(iii) km or  6, 
2  4

A-37. (i) 2x + 3y = 3700, 3x + 4y = 5100


(ii) Rs. 500
(iii) Rs. 6000 OR 8

DEEPIKA MA’AM # 8743011101 : CLASS – X : SOLUTIONS SAMPLE PAPER – 1 : INFINITY ... Think beyond...
SAMPLE PAPER – 2 LEARN MATHEMATICS BY : DEEPIKA MA’AM – 65 –

X - MATHEMATICS
SAMPLE PAPER - 2
Time Allowed : 3 Hours] [Maximum Marks : 80
General Instructions :
1. This Question Paper has 5 Sections A - E.
2. Section A has 20 MCQs carrying 1 mark each.
3. Section B has 5 questions carrying 02 marks each.
4. Section C has 6 questions carrying 03 marks each.

Y
5. Section D has 4 questions carrying 05 marks each.

T
6. Section E has 3 case based integrated units of assessment (04 marks each)with sub-parts of the
values of 1,1 and 2 marks each respectively.

I
7. All Questions are compulsory. However an internal choice in 2 Qs. of 5 marks, 2 Qs of 3 marks
and 2 Questions of 2 marks has been provided. An internal choice has provided in the 2 marks

d
N
questions of Section E.

o n
I
8. Draw neat figures wherever required. Take  = 22/7 wherever required if not stated.

SECTION - A
e y
F B
Section A consists of 20 questions of 1 mark each.
Q1.
k
If a = (22 × 33 × 54) and b = (23 × 32 × 5) , then HCF (a, b) =

in
N
(a) 90 (b) 180 (c) 360 (d) 540

h
I
(x + 2)3 = 2x(x2 – 1) is

T
Q2.
(a) linear equation (b) not quadratic equation
(c) quadratic equation (d) not defined
Q3. In an AP, 18, 13, 8, 3,... S35 =
(a) 2345 (b) 2435 (c) –2345 (d) – 2435
Q4. x = a and y = b is the solution of the linear equation x – y = 2 and x + y = 4, then values of a
and b are
(a) 2, 1 (b) 3, 1 (c) 4, 6 (d) 1, 2
Q5. Three vertices of a parallelogram taken in order are (–1, –6), (2, –5) and (7, 2). The fourth
vertex is
(a) (1, 4) (b) (1, 1) (c) (4, 4) (d) (4, 1)

PX PY 1
Q6. In figure XY || QR,   , then
XQ YR 2

1
(a) XY  QR (b) XY = QR
3

1
(c) XY 2  QR 2 (d) XY = QR
2

DEEPIKA MA’AM # 8743011101 : CLASS – X : SAMPLE PAPER – 2 : INFINITY ... Think beyond...
SAMPLE PAPER – 2 LEARN MATHEMATICS BY : DEEPIKA MA’AM – 66 –

Q7. The coordinates of the point which is equidistant from the three vertices of the AOB as
shown in the figure is

(a) (x, y) (b) (y, x)


x y
(c)  , 

T Y y x
(d)  , 

I
2 2 2 2

Q8. If sin   3 cos , 0    90, then is equal to

d
N n
(a) 30° (b) 45° (c) 60° (d) 90°

o
I y
AB BC CA

e
Q9. If in two triangles ABC and PQR,   , then

F
QR PR PQ

(a) PQR ~ CAB (b) PQR ~ ABC


k B
(c) CBA ~ PQR (d) BCA ~ PQR

i n
N
Q10. The LCM of 2.5, 0.5 and 0.175 is
(a) 2.5 (b) 5
h (c) 7.5 (d) 17.5

I T
Q11. In figure PA and PB are tangents to a circle, PA = 9 cm and APB = 60°, then chord AB =

(a) 4 cm (b) 7 cm (c) 6 cm (d) 9 cm


Q12. The arc of a circle of radius 30 cm having length 19 cm, then angle subtended by this acr at
the centre O of the circle is
(a) 36.27° (b) 36° (c) 30.99° (d) 34°
Q13. If two solid hemispheres of the same base radius r are joined together along their bases, then
curved surface area circle is
(a) 4r2 (b) 6r2 (c) 3r2 (d) 8r2
Q14. While computing mean of grouped data, we assume that the frequencies are
(a) evenly distributed over all the classes
(b) centred at the class marks of the classes
(c) centred at the upper limits of the classes
(d) centred at the lower limits of the classes
DEEPIKA MA’AM # 8743011101 : CLASS – X : SAMPLE PAPER – 2 : INFINITY ... Think beyond...
SAMPLE PAPER – 2 LEARN MATHEMATICS BY : DEEPIKA MA’AM – 67 –

Q15. In making 1000 revolutions, a wheel covers 88 km, then the diameter of the wheel is
(a) 7 m (b) 14 m (c) 36 m (d) 28 m
Q16. A card is drawn from a well-shuffled deck of 52 playing cards. The probability that the card
will not be an ace is

1 1 12 3
(a) (b) (c) (d)
13 4 13 4
Q17. The probability of getting a red face card from a pack of cards is

3 1 1 1

Y
(a) (b) (c) (d)
26 13 52 4

T
Direction : In the question number 19 and 20, a statement of Assertion (A) is followed by a

I
statement of Reason (R). Choose the correct option.
Q19. Statement A (Assertion) : If in a ABC, a line DE || BC, intersects AB in D and AC in E,

d
N n
AB AC
then  .

o
I
AD AE

e y
Statement R (Reason) : If a line a drawn parallel to one side of a triangle intersecting the

F B
other two sides, then the other two sides are divided in the same ratio.

n k
(a) Both assertion (A) and reason (R) are true and reason(R) is the correct explanation of

i
N h
assertion (A).

I T
(b) Both assertion (A) and reason (R) are true and reason (R) is not the correct explanation
of assertion (A).
(c) Assertion (A) is true but reason (R) is false.
(d) Assertion (A) is false but reason (R) is true.

3
Q20. Statement A (Assertion) : In a right-angled triangle, if tan   , the greatest side of the
4
triangle is 5 units.
Statement R (Reason) : (greatest side)2 i.e. (hypotenuse)2 = (perpendicular)2 + (base)2
(a) Both assertion (A) and reason (R) are true and reason(R) is the correct explanation of
assertion (A).
(b) Both assertion (A) and reason (R) are true and reason (R) is not the correct explanation
of assertion (A).
(c) Assertion (A) is true but reason (R) is false.
(d) Assertion (A) is false but reason (R) is true.

DEEPIKA MA’AM # 8743011101 : CLASS – X : SAMPLE PAPER – 2 : INFINITY ... Think beyond...
SAMPLE PAPER – 2 LEARN MATHEMATICS BY : DEEPIKA MA’AM – 68 –

SECTION - B
Section B consists of 5 questions of 2 marks each.
Q21. Aftab tells is daughter, “Seven years ago, I was seven times as old as you were then. Also,
three years from now, I shall be three times as old as you will be.” Represent this situation
algebraically.
Q22. In the given figure, QR is a tangent at Q. P is centre of the circle and PR || AQ, where AQ is
a chord through A, an end point of the diameter AB. Prove that BR is tangent at B.

T Y
Q23.
Q24.

NI
If a hexagon ABCDEF circumscribes a circle prove that AB + CD + EF = BC + DE + FA.
If 5th term of an AP is zero, show that 33rd term is two times is 9th term.

o n d
I y
OR

e
F
Along a road lies an odd number of stones of weight 10 kg each, placed at intervals of 10

B
metres. These stones have to assembled around the middle stone. Nirvah, a stone loader can

k
n
carry only one stone of 10 kg at a time. He started the job with one of the end stones by

i
N h
carrying them in succession. In carrying all the stones, he covered a distance of 3 km. Find

I T
the number of stones.

cos  cos 
Q25. If  m and  n, prove that (m2  n 2 ) cos2   n 2 .
cos  sin 
OR

cos 2 
Solve the equation for  :  3.
cot 2   cos 2 

SECTION - C
Section C consists of 6 questions of 3 marks each.

Q26. Prove that p  q is irrational, where p, q are primes.


Q27. Solve the following system of equations graphically :
x + 2y = 4, 4x + 3y = 10

DEEPIKA MA’AM # 8743011101 : CLASS – X : SAMPLE PAPER – 2 : INFINITY ... Think beyond...
SAMPLE PAPER – 2 LEARN MATHEMATICS BY : DEEPIKA MA’AM – 69 –

15 2 15 7
Q28. Solve for x and y :   1 and   10 (x  y  0, x  y  0)
xy xy xy xy
OR
Determine by drawing graph, whether the following pair of linear equations has infinite
number of solutions or not : y = 5 nd y + 3 = 0.

cos ec 2  sec 2  3
Q29. If cot   7 , show that 2 2

cos ec   sec  4

Q30.
BC.

T Y
A quadrilateral ABCD is a drawn to circumscribes a circle. Prove that : AB + CD = AD +

NI o n d

F I OR

n k B e y
In the given figure, a circle is inscribed in a quadrilateral ABCD in which B = 90°. If AD
= 23 cm, AB = 29 cm and DS = 5 cm, find the radius (r) of the circle.

Q31.
I N T h
Light house is a tower with a bright light at the top. Light house serve as a navigational aid
and to warn boats or ships about dangerous area.
Study the diagram and answer the question based on it.

If one ship is exactly behind the other on the same side of the light-house, find the distance
between the two ships. (Use 3  1.73)

DEEPIKA MA’AM # 8743011101 : CLASS – X : SAMPLE PAPER – 2 : INFINITY ... Think beyond...
SAMPLE PAPER – 2 LEARN MATHEMATICS BY : DEEPIKA MA’AM – 70 –
SECTION - D
Section D consists of 4 questions 5 marks each.
Q32. The sum of three numbers of an AL is 3 and the product of the first and the third number is
–35. Find the three numbers.
OR
Shalini gets pocket money from her father every day. Out of the pocket money, she saves Rs.
30 on the first day and on each succeeding day, she increases her saving by 500 paise. At the
end of every month, Shalini purchases some biscuits packs, toffees and nuts from the amount

Y
that she saved and distribute these items to the needy children in her school.

T
(i) Find the amount saved by Shalini on 10th day.

I
(ii) Find the total amount saved by Shalini in 30 days.
Q33. In the given figure, M is mid-point of the side CD of a rectangle ABCD. BM when joined

d
N
meets AC at L and AD produced at E. Prove that EL = 2BL.

o n

F I i n k B e y

N
Q34. Find the number of bricks, each measuring 25 cm × 12.5 cm × 7.5 cm, required to construct

h
I
a wall 24m long, 20 m high and 0.5 m thick while the cement and sand mixture occupies
1
20
th ofthe volume of the wall. T
OR
Irrigation canals are used to move water from a source (whether it is a stream, reservoir or
holding tank). A farmer connects a pipe of internal diameter 20 cm from a canal into a
cylindrical tank in his field, which is 10m in diameter and 2 m deep. If water flows through
the pipe at the rate of 6 km/h, in how much time will the tank be filled ?
Q35. The marks obtained by 100 students in a mathematics test consisting of 100 marks are given
in the following table :
Marks obtained 0  14 14  28 28  42 42  56 56  70
No. of students 8 20 28 18 26
Find the mean marks obtained by the students.

SECTION - E
Case study based questions are compoulsory.
Q36. We can determine whether a quadrilateral placed on coordinate plane is a parallelogram or
not. In coordinate geometry, distance formula and mid point formula are enough to show
DEEPIKA MA’AM # 8743011101 : CLASS – X : SAMPLE PAPER – 2 : INFINITY ... Think beyond...
SAMPLE PAPER – 2 LEARN MATHEMATICS BY : DEEPIKA MA’AM – 71 –

that quadrilateral placed on coordinate axes is a parallelogram or not. If vertices of triangle


are given then using distance formula we can find length of sides of triangle, e.g., ABCD is a
quadrilateral place on coordinates axes as shown.

T Y
NI
(i) If A(2, 3), B(4, 6), C(7, 4) and D(a, b) are the vertices of a quadrilateral, such that

d
diagonals AC and BD intersects each other at O. If O is mid point of AC and BD then

o n
I
find the value of a and b.

e y
(ii) Three vertices of a parallelogram taken in order are (0, 3), (0, 0) and (5, 0), then find the

F B
fourth vertex.

n k
(iii) If P(5, 2), Q(2, – 2) and R(2, y) are vertices of right-angled triangle where Q = 90°

i
N
then find y.

h
I T OR
Three vertices of rectangle AOBC are A(0, 3), O(0, 0), B(5, 0), then find the length of
diagonals AB and OC.
Q37. India is competitive manufacturing location due to the low cost of manpower and strong
technical and enginnering capabilities contributing to higher quality production runs. The
production of TV set in a factory increases uniformly by a fixed number every year. It
produced 16000 sets in 6th year and 22600 in 9th year.

Based on the above information, answer the following questions :

DEEPIKA MA’AM # 8743011101 : CLASS – X : SAMPLE PAPER – 2 : INFINITY ... Think beyond...
SAMPLE PAPER – 2 LEARN MATHEMATICS BY : DEEPIKA MA’AM – 72 –

(i) Find the production in first year.


(ii) In which year, the production will be 29200 ?
(iii) Find the difference of the production during 7th year and 4th year.
OR
Find the difference between 12th year and first year.
Q38. Point A is the position of jet fighter flying in the sky. The angle of elevation of point A from
ground is shown. After 15 seconds, the jet figher moves in direction AP and reaches at point

Y
P. The angle of elevation of point P on the ground is shown (Assume that figher is flying at
the constant height above the ground).

I T d

F I N i n k B
Based on the above information, answer the following questions :
e y o n
(i) What is the distance of AP, if jet is flying with speed 720 km/h in 15 seconds ?

N
(ii) If the jet is flying at the speed of 360 km/h then find the distance covered in 15 seconds.

h
I T
(iii) If the jet in flying at a speed of 720 km/h then find the constant height at which it is
flying.
OR
If the jet is flying at the speed of 360 km/h then find the constant height at which it is
flying.

DEEPIKA MA’AM # 8743011101 : CLASS – X : SAMPLE PAPER – 2 : INFINITY ... Think beyond...
SAMPLE PAPER – 2 LEARN MATHEMATICS BY : DEEPIKA MA’AM – 73 –

XII - MATHEMATICS
SOLUTIONS : SAMPLE PAPER - 2
A-1. (b) HCF (a, b) = 22 × 32 × 5 = 180
PX PY 1
Also   (Given)
A-2. (b) We have (x + 2)3 = 2x(x2 – 1) XQ YR 2
 x3+ 8 + 3(x)(2)(x + 2) = 2x3 – 2x
PX PY 1
 x3 – 6x2 – 14x – 8 = 0  PQ  PR  3
It is not a quadratic equation.

Y
A-3. (c) The given AP is 18, 13, 8, 3,... XY 1 1
 QR  3  XY = QR
Here a = 18, d = 13 – 18 = – 5 3

T
35 A-7. (a)  AOB is a right triangle

I
S35  [2  18  34( 5)] = –2345
2  Mid-point of AB is equidistant
from A, O and B.

d
A-4. (b) We have x – y = 2 and x + y = 4

N
Mid-piont of AB
Also x = a and y = b

o n
I y
 Equaton become  0  2x 2y  0 
,

e
=  = (x, y)
 2 2 

F
a–b=3 ...(i)
and a + b = 4 ...(ii_

k B
(c) sin   3 cos  
sin 
 3

n
On adding (j) and (ii), we get a = 3, A-8.

i
cos 

N
b = 1.

h
I T
A-5. (d) Let fourth vertex be (x, y)  tan   3  tan 60   = 60°
Mid-points of diagonals
AB BC CA
A-9. (a)   
 1  7 6  2   2  x 5  y  QR PR PQ
 2 , 2  =  2 , 2 
    Then CAB PQR
 x=4y=1 A-10. (d)
A-6. (a) In PQR A-11. (d) PA and PB are tangent to a circle
XY || QR Also PA = PB
 PXY ~ PQR (AA similarity) (Equal tangents from common exter-
nal point)
 PAB = PBA
(Angles opposite to equal side
of triangle are equal)

PX PY XY
  
PQ PR QR

[As PXY ~ PQR] ...(i)


In APB

DEEPIKA MA’AM # 8743011101 : CLASS – X : SOLUTIONS SAMPLE PAPER – 2 : INFINITY ... Think beyond...
SAMPLE PAPER – 2 LEARN MATHEMATICS BY : DEEPIKA MA’AM – 74 –

PAB + PBA + APB = 180° A-18. (b) In ABC, B = 90°


 PAB + PAB + 60° = 180°  tan A = 1 – tan 45°
 PAB = 60°  A = 45°
 APB is an equilateral triangle  2 sin A cos A = 2 sin 45° cos 45°
 PA = PB = AB 1 1
(All sides are equal) = 2  1
2 2
 AB = 9 cm
A-19. (a) Both Assertion (A) and Reason (R)
A-12. (a) Right of circle = 30 cm
are true and Reason (R) is the cor-

Y
r rect explanation of Assertion (A).
Length of an arc of a circle =
180

T
A-20. (a) Both Assertion (A) and Reason (R)
are true and Reason (R) is the cor-

I
 is the angle subtended by arc at
rect explanation of Assertion (A).
the centre of circle.
A-21. Let the present age of A flab be x years

d
N
22 30   and present age of his daughter be y years.

n
 19 = 

o
7 180

I
According to question,
 36.27° = 

e y
x – 7 = 7(y – 7)

F B
A-13. (a) 4r2  x – 7y = – 42 ...(i)
A-14. (b) centred at the class marks of the

n k
and x + 3 = 3(y + 3)

i
N
classes.  x – 3y = 6 ...(ii)

h
I
A-15. (d) Let radius of wheel be 4 m Thus , the algebraic representation is

T
 Distance of travelled during one
revolution = 2r
given by (i) and (ii).
A-22. Given : QR is tangent at Q to a circle
Distance travelled during 1000 havin centre at P and chord AQ || PR.
revolutons
= 1000 (2r)
 88 × 1000 = 1000 (2r)

88  7
 r=  14
2  22
To Prove : BR is tangent at B.
Diameter = 2r = 2 × 14 m Proof : We have AQ || PR
= 28 m  1 = 4 (Correspoinding angles)
A-16. (c) Total number of cards = 52 ...(i)
Number of ace = 4 and 2 = 3 (Alternative interior
angles) ...(ii)
48 12
P(not be an ace) =  Also 1 = 2 ...(iii)
52 13
( PA = PQ,radii of the same circle)
3 From (i), (ii) and (iii), we get
A-17. (a)
26
3 = 4 ...(iv)
DEEPIKA MA’AM # 8743011101 : CLASS – X : SOLUTIONS SAMPLE PAPER – 2 : INFINITY ... Think beyond...
SAMPLE PAPER – 2 LEARN MATHEMATICS BY : DEEPIKA MA’AM – 75 –

In PQR and PBR, = AL + BH + CH + DJ + EJ + FL


PR = PR (Common)  (AG + BG) + (CI + DI) + (EK + FK)
PQ = PB (Radii of the same = (BH + CH)+(JD + EJ)+(FL + AL)
circle)  AB + CD + EF = BC + DE + FA
3 =4 (From iv) Hence proved.
PQR PBR (SAS congruence rule) A-24. Let the term of AP be a and the common
 PBR = PQR (CPCT) difference be d.
Now, PQR = 90° [QR is tangent and A.T.Q. a5 = 0

Y
PQ is radius]
 a + 4d = 0  a = – 4d ...(i)
 PBR = 90°

T
Now, a 33 = a + 32d
 BR is tangent at B. Hence Proved.

I
A-23. Given : ABCDEF hexagon circumscribe  a 33 = – 4d + 32d [using (i)]
a circle and touches at G, H, I, J, K, L
 a 33 = 28d ...(ii)
To prove :
d
N n
Also a19 = a + 18d

o
AB + CD + EF = BC + DE + FA

I y
Proof : Hexagon ABCDEF touches a

e
 a19 = – 4d + 18d ...[using (i)]

F
circle at G, H, I, J, K, L. So, from the
external point, tangents drawn on the
circle are equal in length.

k B a19 = 14d

n
On multiplying with 2 on both sides, we

i
N
If A is external point and AG and AL are get

h
I T
tangents, so
 2  a19 = 2 × 14d = 28d ...(iii)
From (ii) and (iii)

a 33 = 2 × a19 Hence proved.


OR
AG = AL ...(i)
Similarly for B,
BG = BH ...(ii)
Let there are (2n + 1) stones. The middle
Similarly for C, stone is at B. The middle stone is at B.
CI = CH ...(iii) Let n stones are on one side of B and n
Similarly for D, stones on other side of B.
DI = DJ ...(iv) Let man started from A.
Similarly for E, Distance covered from A to B
EK = EJ ...(v) = 10 × nm = 10n metres
and similarly for F, Distance covered to carry IInd stone
FK = FL ...(vi) = 2 × (n – 1) × 10 metres
Adding (i), (ii), (iii), (iv), (v), (vi), we get Distance covered to carry IIIrd stone
AG + BG + CI + DI + EK + FK = 2 × (n – 2) × 10 metres
DEEPIKA MA’AM # 8743011101 : CLASS – X : SOLUTIONS SAMPLE PAPER – 2 : INFINITY ... Think beyond...
SAMPLE PAPER – 2 LEARN MATHEMATICS BY : DEEPIKA MA’AM – 76 –

and so on.
 cos 2  cos2  2
 Total distance covered to carry n =  2
 2 
 cos 
stones from this side of B.  cos  sin  
= 10n + 2 × 2(n – 1) × 10 + 2(n – 2) + 10
+ .... + 2 × 10  cos 2  sin 2   cos2  cos2   2
=   cos 
= 10[n +2(n – 1) + 2(n – 2) + 2... + 2]  cos 2  sin 2  
= 10 {n + 2[(n – 1) + (n – 2) + ... + 1]}

n 1 cos 2 (sin 2   cos 2 )


  =

Y
= 10 n  2   [(n  1)  1 sin 2 
 2 

T
= 10[n + (n – 1)n] 2
cos2 
 cos  
n2 10n2   n 2 = RHS

I
= 10[n + – n] = = 2 
sin   sin  
Now, distance covered to collect n stones

d
from other side of B will be 10n metres  LHS = RHS Hence Proved

N n
more than this distance as the person has OR

o
I
to move from B to C to pick the stone at
other end and come back. cos 2 

e y 3

F
2 2

B
 Distance covered to collect n stones cot   cos 
from other side = 10n2 + 10n
k
in
cos 2 

N
Total distance covered  3

h
cos 2  2

I
= 10n2 + 10n2 + 10n  cos 

= 20n2 + 10n T sin 2 

cos 2  sin 2 
 20n 2  10n  3000  3
cos 2 (1  sin 2 )
 2n 2  25n  24n  300  0

 n(2n + 25) – 12(2n + 25) = 0 sin 2 


 2
 3  tan 2   3
 (n – 12)(2n + 25) = 0 cos 
 n – 12 = 0 or 2n + 25 = 0  tan   3  60    60
25 A-26. Let p be rational so that it can be writ-
 n = 12 or n =  (rejecting)
2
a
 Total number of stones ten in the form of
b
= 2n + 1 = 2 × 12 + 1 = 25 a
p (where a and b are coprime)
cos  cos  b
A-25. We have  m and n
cos  sin  a2
Squaring both sides, p =
b2
2
Consider LHS = (m  n ) cos  2 2
a2 has a factor p. pb2 = a2 ...(i)
so, a also has a factor p.
DEEPIKA MA’AM # 8743011101 : CLASS – X : SOLUTIONS SAMPLE PAPER – 2 : INFINITY ... Think beyond...
SAMPLE PAPER – 2 LEARN MATHEMATICS BY : DEEPIKA MA’AM – 77 –

so, a = pc; a2 = p2 c2 1 1
Now, =
Put the value of a2 in equation (i) xy 5
pb2 = p2 c2; b2 = pc2  x+y= 5 ...(iv)
b2 has a factor p.  b has a factor p and B= 1

But a and b are common factor p. But as 1


stated earlier a, b are coprimes.  =1
xy
So, our supposition is wrong, p must  x–y= 1 ...(v)
be an irrational number. (where p is a Adding (iv) and (v), we get

Y
prime number) Putting x = 3 in (iv), we get y = 2
We can prove q is also an irrational  x = 3, y = 2

T
OR
number (where q is a prime number).

I
Sum of two irrational number is irratio-
nal if both are prime numbers.

d
N
So, p  q is irrational number..

o n
I
A-27. The solution table for x + 2y = 4

x 0 4 2 2
e y
F B
y 2 0 1 3
The solution table for 4x + 3y = 10 is

n k
i
N
x 1 4 2 2.5

h
Given equation are y = 5 and y + 3 = 0

I
y 2 2 6 0

T x 0 1 2
Table for y = 5 is y 5 5 5

Table for y + 3 = 0  y = 3 is

x 0 1 2
y 3 3 3
 Graph represent two parallel lines.
 Given pair of linear equations has no
common solution.

1 1 A-29. Given, cot   7


A-28. Let  A and B
xy xy
cos ec 2  sec 2 
 Given equation becomes LHS =
cos ec 2  sec 2 
15A – 2B = 1 ...(i)
Dividing the numerator and denominator
15A + 7B = 10 ...(ii)
by sec2,
Subtracting (i) from (ii), we get
B= 1 cot 2   1 ( 7) 2  1 7 1
 
1 cot 2   1 ( 7 )2  1 7 1
Putting B = 1 in (i), we get A =
5
6 3
=  = RHS
8 4
DEEPIKA MA’AM # 8743011101 : CLASS – X : SOLUTIONS SAMPLE PAPER – 2 : INFINITY ... Think beyond...
SAMPLE PAPER – 2 LEARN MATHEMATICS BY : DEEPIKA MA’AM – 78 –
A-30. Given : A quadrilateral ABCD circum-  r = 11 cm
scribes a circle with centre O. Hence, the radius (r) of the circle = 11cm.
A-31. Here AB be the lighthouse and C and D
be the two ships.

To prove : AB + CD = AD + BC
Proof : Here, AP = AS ...(i)

Y
(Lengths of tangents drawn from an ex-
ternal point to a circle are equal)  XAD = 30° and XAC = 45°

T
Similarly, BP = BQ ...(ii)  AX || BD
CR = CQ ...(iii)

I
 ADB = 30° and ACB = 45°
and DR = DS ...(iv) (Alternate interior angles)
Adding (i), (ii), (iii) and (iv), we get In right-angled ABC,

d
N
AP + BP + CR + DR = AS + BQ +
CQ + DS
AB

o
= tan 45°
n
I
BC

y
 (AP + BP) + (CR + DR)
70
e
F
= (AS + DS) + (BQ + CQ)

B
 =1
 AB + CD = AD + BC BC
Hence proved.

n

k BC = 70 m

i
N
OR In right-angled ABD,

h
I
Given : Quadrilateral ABCD circum-

T
AB
scribed a circle. = tan 30°
BD

70 1
 =
BD 3

 BD = 70 3m

B = 90°, AD = 23 cm, AB = 29 cm, DS  Distance between the two ships,


= 5 cm. CD = BD – BC
To find : Radius of circle = 70 3  70
OQ AB and OP BC
= 70( 3  1)
(Radius is perpendicular to the tangent)
OQ = OP (Radii of a circle) = 70(1.73 – 1) = 70 × 0.73
 OPBQ is a square. = 51.1 m
 BQ = BP = OP = r cm A-32. Let the three numbers of an AP are a – d,
a and a + d
Now, RD = DS  RD = 5 cm
According to question,
 AR = AD – RD
a–d+a+d=3
= 23 – 5 = 18cm
 3a = 3
Also, AR = AQ  AQ = 18 cm
 a= 1 ...(i)
Now, AB = AQ + BQ
Also (a – d)(a + d) = – 35
 29 = 18 + r
DEEPIKA MA’AM # 8743011101 : CLASS – X : SOLUTIONS SAMPLE PAPER – 2 : INFINITY ... Think beyond...
SAMPLE PAPER – 2 LEARN MATHEMATICS BY : DEEPIKA MA’AM – 79 –

 (1 – d)(1 + d) = – 35[using (i)]  EDM  BCM


 1 – d2 = – 35 (ASA Congurnece rule)
 d2 = 36  ED = BC (CPCT) ...(i)
d = 6 or – 6 Also, AD = BC ...(ii)

when a = 1 and d = 6, the required three (opposite sides of rectangles)
numbers are 1 – 6, 1, 1 + 6; i.e, –5, 1, 7 Adding (i) and (ii), we get
When a = 1 and d = – 6, the required num- ED + AD = 2BC
bers are {1 – (–6)}, 1, {1 + (–6)}; i.e, 7,  AE = 2BC ...(iii)
1, – 5. In ALE and CLB,

Y
OR 7 = 8
Money saved on 1st day = Rs. 30 (Vertically opposite angels)

T
Money saved on IInd day = Rs. 35 1 = 2

I
Money saved on IIIrd day = Rs. 40 and (Alternate interior angels)
so on.
 ALE ~ CLB (AA similarity)
Amount of money saved on successive
AE EL
d
N
days is an AP with a = 30 and d = 5.

n
 =

o
(i) Money saved on 10th day, BC BL

I y
2BC EL

e
a10 = a + 9d = 30 + 9 × 5
 = [Using (iii)]

F
BC BL

B
= Rs. 75
 EL = 2BL Hence Proved

k
(ii) Money saved in 30 days

n
A-34. Dimensions of brick are l = 25 cm

i
N
30
S30 = [2 × 30 + (30 – 1)×5]

h
2 25

I
= m


n
T 
 Sn  2 [2a  (n  1)d 

100

b = 12.5 m =
12.5
100
m
= 15(60 + 145)
= 15 × 205 = Rs. 3075 7.5
h = 7.5 cm = m
A-33. Given : In a rectangle ABCD, M is mid- 100
point of CD. BM intersects AC at L and Volume of one brick = l × b × h
meets AD on producing at E.
25 12.5 7.5 3
To Prove : EL = 2BL =   m
100 100 100
Proof : In EDM and BCM.
Volume of the wall = L × B × H
= 24 × 20 × 0.5 m3
= 240 m3
Volume of occupied by bricks
= (240 – 12) m3
5 = 6
= 228 m3
(vertically opposite angles)
3 =4  Number of bricks required
(Altrenate interiror angles) Vol. occupied by bricks
= Vol. of one brick
DM = CM
( M is mid-point of CD)
DEEPIKA MA’AM # 8743011101 : CLASS – X : SOLUTIONS SAMPLE PAPER – 2 : INFINITY ... Think beyond...
SAMPLE PAPER – 2 LEARN MATHEMATICS BY : DEEPIKA MA’AM – 80 –

228  34 
= = 35     14
25 12.5 75
   100 
100 100 100
= 35 + 4.76
228 100 100 100 = 39.76
= = 9728
25  12.5  7.5 So, the mean marks obtained is 39.76.
OR A-36. (i) Here O is mid piont of AC and BD
Radius of cylindrical tank = r = 5m

Depth of cylindrical tank = h = 2m

 Volume of cylindrical tank = r2h


= ×5×5×2 27 a4

T Y
I
 =
= 50 m3 2 2
 9= a+4  a=5

d
Radius of the pipe = 10cm = 0.1m

N n
3 4 6 b
and =

o
Rate of flow of water = 6 km/h

I
2 2
= 6000 m/h 

e y
7= 6+b  b=1

F B
Volume of water that flows in 1 hour (ii) As we know that diagonals of a par-

=  × 0.1 × 0.1 × 6000 m3


n k allelogram bisect each other.

i
N h
= 60 m3

A-35.
I
 Time required to fill the tank

=
50 5


6
 hours
60 6

 60 = 50 minutes
T
Now,


0 x
2

0 y
=
05

x= 5
2

3 0
and =
xi  A
2 2
Marks Obt. No. of students Class mark ui  fi ui
(fi ) (xi ) h  y= 3
0  14 8 7 2 16
14  28 20 21 1 20  Fourth vertex is (5, 3).
28  42 28 35  A 0 0
42  56 18 49 1 18 (iii) Using pythagoras theorem in right-
56  70 26 63 2 52
fi  100 fi ui  34 angled PQR, we get

Using step-deviation method, we have


fi u i
x = A h
f i

(PR)2 = (PQ)2 + (QR)2


DEEPIKA MA’AM # 8743011101 : CLASS – X : SOLUTIONS SAMPLE PAPER – 2 : INFINITY ... Think beyond...
SAMPLE PAPER – 2 LEARN MATHEMATICS BY : DEEPIKA MA’AM – 81 –

 (5 – 2)2 + (2 – y)2 = (5 – 2)2 + (iii) a 7 = a + 6d


(2 + 2)2 + (2 – 2)2 +(y + 2)2 a 4 = a + 3d

 9 + 4 + y2 – 4y = 9+16 + y2 + 4y + 2 a 7  a 4 = a + 6d – a – 3d
= 3d = 3 × 2200 = 6600
 – 4y = 16 + 4y
OR
 y = –2 a12  a = a + 11d – a = 11d
OR = 11 × 2200
= 24200

Y
As we know that diagonals of rect- A-38. (i) Distance AP = Speed × time
angle are equal. 720  1000

T
= × 15
3600

I
= 3000 m
(ii) Distance AP = Speed × time
360  1000
d
N
 15m

n
=
3600

o
I
 AB = OC = 1500 m

(5  0)2  (0  3)3
e y
(iii) Let H be the constant height at which

F
 AB =

= 25  9  34
k B the jet is flying

n
In ABQ

i
N
Hence AB = OC = 34 units

h AQ

I T
A-37. (i) a 6 = 16000 tan 60° =
BQ
a 9 = 22600
H
 BQ =
a + 5d = 16000 ...(i) 3
a + 8d = 22600 ...(ii) In PBD
PD
From (i) and (ii), we get tan 30° =
BD
d = 2200 1 H
 =
 a + 5(2200) = 16000 3 BQ  QD

a = 16000 – 11000 = 5000 1 H


 = H  3000 (QD =AP)
(ii) a n = 29200 3 3

 29200 = a + (n – 1)d  H  1
  3  3000  3  H
29200 = 5000 + (n – 1)2200  

H 3000
24200  =H
 = n–1 3 3
2200
 n = 12 3  3000
H=  1500 3 m
In 12th year the production of the 2 3
company will be 29200. OR
DEEPIKA MA’AM # 8743011101 : CLASS – X : SOLUTIONS SAMPLE PAPER – 2 : INFINITY ... Think beyond...
SAMPLE PAPER – 2 LEARN MATHEMATICS BY : DEEPIKA MA’AM – 82 –

AQ
In ABQ = tan 60° =
BQ
AQ
 BQ =
3
PD
Now in PBD, tan 30° =
BD
1 AQ
= (AQ = PD)
3 BQ  QD

Y
BQ  QD
AQ =
3

T
AQ QD

I
= 
3 3 3
QD

d
2AQ

N
 =

n
3 3

o
I
3 1500
 AQ =
2 3
 750 3m

e y

N F h i n k B

I T

DEEPIKA MA’AM # 8743011101 : CLASS – X : SOLUTIONS SAMPLE PAPER – 2 : INFINITY ... Think beyond...
SAMPLE PAPER – 3 LEARN MATHEMATICS BY : DEEPIKA MA’AM – 83 –

X - MATHEMATICS
SAMPLE PAPER - 3
Time Allowed : 3 Hours] [Maximum Marks : 80
General Instructions :
1. This Question Paper has 5 Sections A - E.
2. Section A has 20 MCQs carrying 1 mark each.
3. Section B has 5 questions carrying 02 marks each.
4. Section C has 6 questions carrying 03 marks each.

Y
5. Section D has 4 questions carrying 05 marks each.

T
6. Section E has 3 case based integrated units of assessment (04 marks each)with sub-parts of the
values of 1,1 and 2 marks each respectively.

I
7. All Questions are compulsory. However an internal choice in 2 Qs. of 5 marks, 2 Qs of 3 marks
and 2 Questions of 2 marks has been provided. An internal choice has provided in the 2 marks

d
N n
questions of Section E.

o
I
8. Draw neat figures wherever required. Take  = 22/7 wherever required if not stated.

SECTION - A
e y
F B
Section A consists of 20 questions of 1 mark each.

Q1.
n k
If two positive integers a and b are written as a  x 3 y 2 and b  xy3 , where x, y are prime

i
N h
numbers, then HCF (a, b) is

I T
(a) xy (b) xy2 (c) x3y3 (d) x2y2

Q2. If (1 – p) is a root of the equation x2 + px + 1 – p = 0, then its roots are


(a) 0, 1 (b) –1, 1 (c) 0, –1 (d) –1, 2
Q3. If p(x) = ax2 + bx + c and a + c = b, then one of the zero is

b c c b
(a) (b) (c) (d)
a a a a

Q4. The HCF of 2472, 1284 and a third number N is 12. If their LCM is 23 × 32 × 5 × 103 × 107,
then the number N is :
(a) 22 × 32 × 7 (b) 22 × 33 × 103 (c) 22 × 32 × 5 (d) 24 × 32 × 11

Q5. If three points (0, 0), (0, 3) and (3, k) form an equilateral triangle, then k =

(a) 2 (b) –3 (c)  3 (d)  2

Q6. Number of tangens to a circle which are parallel to a secant is


(a) 1 (b) 2 (c) 3 (d) Infinite
Q7. (cosec A – sin A) (sec A – cos A) – cos A (tan A + cot A) =
(a) 2 (b) –2 (c) 1 (d) –1
DEEPIKA MA’AM # 8743011101 : CLASS – X : SAMPLE PAPER – 3 : INFINITY ... Think beyond...
SAMPLE PAPER – 3 LEARN MATHEMATICS BY : DEEPIKA MA’AM – 84 –

Q8. If 3 sin   cos   0, 0    90, then  


(a) 30° (b) 45° (c) 90° (d) 60°
Q9. In given figure, AD = 3 cm, AE = 5 cm, BD = 4 cm, CE = 2 cm, BF = 2.5 cm, then

Y
(a) DE || BC (b) DF || AC (c) EF || AB (d) none of these
Q10. If ABC ~ EDF and ABC is not similar to DEF, then which of the following is not true?

T
(a) BC.EF = AC.FD (b) AB.EF = AC.DE

I
(c) BC.DE = AB.EF (d) BC.DE = AB.FD

d
Q11. AOBC is a rectangle whose three vertices are A(0, 3), O(0, 0) and B(5, 0). The length of its

N n
diagonal is

o
I y
(a) 5 units (b) 3 units (c) 34 units (d) 4 units

e
F B
Q12. Three numbers are in an AP, having sum 24. Its middle term is

k
(a) 6 (b) 8 (c) 3 (d) 2
Q13.

i n
Which term of the AP : 22, 19, 16, ... is its first negative term ?

N h
(a) 9 (b) 8 (c) 10 (d) 11

Q14.

Q15.

Q16.
I
(a) s4 (b) 5
T
If f1  11, f1x1  2p  52 and the mean of any distribution is 6, find the value of p.
(c) 6 (d) 7
A solid cube is cut into 27 small cubes of equal volume, then the ratio of the surface area of
the given cube and that of one small cube is
(a) 9 : 1 (b) 1 : 9 (c) 1 : 1
If the mode of a data is 18 and the mean is 24, then median is
(d) 3 : 3

(a) 10 (b) 15 (c) 22 (d) 24


Q17. For an even E, P(E) + P(E)  q, then
(a) a  q  1 (b) 0  q  1 (c) 0 < q < 1 (d) none of these

Q18. If sin  and cos  are the roots of the equation ax2 – bx + c = 0, then a, b, c satisfy the
relation
(a) b2 – a2 = 2ac (b) a2 – b2 = 2ac (c) a2 + b2 = c2 (d) a2 + b2 – 2ac
Direction : In the question number 19 and 20, a statement of Assertion (A) is followed by a
statement of Reason (R). Choose the correct option.
Q19. Statement A (Assertion) : Discriminant of the quadratic equation 3x2 + 4x – 5 = 0 is 76.
Statement R (Reason) : D = b2 + 4ac
DEEPIKA MA’AM # 8743011101 : CLASS – X : SAMPLE PAPER – 3 : INFINITY ... Think beyond...
SAMPLE PAPER – 3 LEARN MATHEMATICS BY : DEEPIKA MA’AM – 85 –

(a) Both assertion (A) and reason (R) are true and reason(R) is the correct explanation of
assertion (A).
(b) Both assertion (A) and reason (R) are true and reason (R) is not the correct explanation
of assertion (A).
(c) Assertion (A) is true but reason (R) is false.
(d) Assertion (A) is false but reason (R) is true.
Q20. Statement A (Assertion) : The two tangents are drawn to a circle from an external point,
then they subtend equal angles at the centre.

Y
Statement R (Reason) : A parallelogram circumscribing a circle is a rhombus.
(a) Both assertion (A) and reason (R) are true and reason(R) is the correct explanation of

T
assertion (A).

I
(b) Both assertion (A) and reason (R) are true and reason (R) is not the correct explanation
of assertion (A).

d
(c) Assertion (A) is true but reason (R) is false.

N n
(d) Assertion (A) is false but reason (R) is true.

o
Q21.

F I SECTION - B

B
Section B consists of 5 questions of 2 marks each.

i n k
e
Which term of an Arthmetic Progression : 2, 7, 12, 17, ..., is 137 ?
y

N h
RQ RS

I
Q22. In the given figure, P = RTS. Show that : RPQ ~ RTS and  .

T RP RT

Q23. Find the distance between two parallel tangents of a circle of radius 6 cm.
Q24. The circumference of the edge of a hemispherical bowl is 132 cm. Find the capacity of the
bowl. (Use  = 22/7).
OR
Find the area of a sector of an angle A (in degree) of a circle with radius R.

1 sec 2   tan 2 
Q25. If cos   , find .
2 7  2sec  cos ec
OR

1
Prove that (cosec A – sin A) (sec A – cos A) =
tan A  cot A

DEEPIKA MA’AM # 8743011101 : CLASS – X : SAMPLE PAPER – 3 : INFINITY ... Think beyond...
SAMPLE PAPER – 3 LEARN MATHEMATICS BY : DEEPIKA MA’AM – 86 –

SECTION - C
Section C consists of 6 questions of 3 marks each.

Q26. Prove that 15  17 3 be an irrational number..


Q27. Solve for x and y :
1 1 3 1 1 1
  ;    ; 3x  y  0, 3x  y  0
3x  y 3x  y 4 2(3x  y) 2(3x  y) 8

Q28. Find the zeros of the polynomial 4 3x 2  4 3x  3 3. Also, verify the relationship be-
tween the zeroes and the coefficients.

Y
OR

T
4 5 3
Solve for x : 3 ; x  0,

I
x 2x  3 2
3
Q29. If sin (A + 2B) = and cos(A + 4B) = 0, A > B and A  4B  90, then find A and B.
2
d
N n
Q30. In the given figure, PT is a tangent and PAB is a secant to

o
I y
a circle with centre O. ON is perpendicular to the chord
AB. Prove that :
e
F B
(i) PA.PB = PN2 – AN2

(ii) PN2 – AN2 = OP2 – OT2


n k
i
N h
OR

I T
In the given figure, O is the centre of the circle and TP is the tangent to the circle from an
external point T. If PBT = 30°, prove that BA : AT = 2 : 1.

Q31. A boy standing on a horizontal plane find a kite flying at a distance of 150m from him at an
angle of elevation of 30°. A girl standing on the roof of 30m high building finds the angle of
elevation of the same kite to be 45°. Both boy and girl are on the opposite side of the kite.
Find the distance of the kite from the girl.

SECTION - D
Section D consists of 4 questions 5 marks each.
Q32. A train travels 360 km at a uniform speed. If the speed had been 5 km/h more, it would have
taken 1 hour less for the same journey. Find the speed of the train.
OR

1
The difference of two numbers is 5 and the difference of their reciprocals is . Find the
10
numbers.
DEEPIKA MA’AM # 8743011101 : CLASS – X : SAMPLE PAPER – 3 : INFINITY ... Think beyond...
SAMPLE PAPER – 3 LEARN MATHEMATICS BY : DEEPIKA MA’AM – 87 –

Q33. Diagonals AC and BD of a trapezium ABCD with AB || DC intersect each other at the point
OA OB
O. Using a similarlity criterion for two triangles, show that  .
OC OD
Q34. A solid consisting of a right circular cone of height 120 cm and radius 60 cm standing on a
hemisphere of radius 60 cm is placed upight in a right circular cylinder full of water such that
it touches the bottom. How many litres of water is left in the cylinder if the radius of the
cylinder is 60 cm and its height is 180 cm.
OR
The cost of fencing a circular field at the rate of Rs. 24 per m is Rs. 5280. The field is to be

Y
ploughed at the rate of Rs.0.50 per m2. Find the cost of ploughing the field.
Q35. The students of Class X of a school decided to donate their pocket money to purchase

T
mineral water bottles for the people using contaminated water in a nearby village. They
packed the mineral water bottles in different boxes. These boxes contained varying number

I
of mineral water bottles. The following table shows the distribution of mineral water bottles
according to the number of boxes :

d
N n
No. of mineral water bottles Number of boxes

o
I
50  52 20

y
53  55 120
56  58 105
e
F B
59  61 125

k
62  64 30

i n
N
Find the mean number of mineral water bottles kept in a packing box.

h
I T
SECTION - E
Case study based questions are compoulsory.
Q36. Our country can be a manufacturing hub due to cheap labour cost and very high number of
skilled technical man powers, which can contribute to cheaper and higher production. The
manufacturing of mobile phone sets production unit increase by a fixed number every year.
If manufactures 4,20,000 sets in the 5th year and 6,00,000 sets in 8th year.

(i) Find the production in the first year.


(ii) Find the common difference
(iii) What will be the total production in the first 4 years ?
OR
What will be the total production in first 5 years ?
DEEPIKA MA’AM # 8743011101 : CLASS – X : SAMPLE PAPER – 3 : INFINITY ... Think beyond...
SAMPLE PAPER – 3 LEARN MATHEMATICS BY : DEEPIKA MA’AM – 88 –

Q37. A building is made by keeping the lower window of a building at a particular height above
the ground and upper window is constructed at some height vertically above the lower
window. Position of both windows are shown in diagram.
Both windows are designed and constructed in order to have proper Sunlight.

T Y
NI o n d

F I i n k B e y

N
At certain instant, the angle of elevation of balloon from these windows are shown. Balloon

h
I
is flying at constant height H above the ground.

T
(i) Find the length AR (in terms of H)
(ii) Find the height H.
(iii) Find the distance of balloon from the lower window.
OR
Find the distance of balloon from the upper window.
Q38. Rajiv decided to put a frame on a scenery which is quadrilat-
eral in shape as shwon. He placed this scenery on coordinate
axes such that one vertex coincides with origin O and one arm
OA coincides with x-axis and another arm OC coincides with
y-axis. Here OA = 5 units and OC = 3 units.
(i) Find the length of diagonal OB.
(ii) Find the value of ABC.
(iii) Find the perimeter of OABC.
(iv) Find the coordinates of mid-point of OB and OC.

DEEPIKA MA’AM # 8743011101 : CLASS – X : SAMPLE PAPER – 3 : INFINITY ... Think beyond...
SAMPLE PAPER – 3 LEARN MATHEMATICS BY : DEEPIKA MA’AM – 89 –

XII - MATHEMATICS
SOLUTIONS : SAMPLE PAPER - 3
A-1. (b) a=x×x×x×y×y
 k 3
and b = xy × y × y
A-6. (b) Here PBA is a secand. We can draw
 HCF (a, b) = x × y × y = x × y2 = xy2
only two tangents which are parallel
A-2. (c) Let  and  be the roots of the to secant PBA.
equation

Y
x 2  px  1  p  0
Let  = 1 – p (given)

T
p

I
 +=
1
A-7. (c) (cosecA – sinA)(secA – cosA)(tanA
1–p+=–p

d
+ cot A)

N n
   1

o
 sin1A  sin A  cos1 A  cos A 

I
=

y
Putting   1 in x2 + px + 1 – p =

e
F
we get sin A  cos A

B
cos A sin A
   1 1  0
Roots of equation are 0 and –1
n k cos2 A sin 2 A 1

i
N
= . .
(c) We have p(x) = ax2 + bx + c

h
A-3. sin A cos A sin A.cos A

I T
Putting x = –1, we get
p(–1) a – b + c = (a + c) – b cos 2 A sin 2 A
= 1
= b – b = 0 (As a + c = b) sin 2 A cos 2 A

 –1 is zero of p(x) A-8. (d) 3 sin   cos 


Let other zero be  .
tan   3
c
 ( 1)  (Product of rots) tan   tan 60   60
a
BF AE
c A-9. (c)  
 FC EC
a
 EF || AB
A-4. (c) 2472 = 23 × 3 × 103
A-10. (c)  ABC ~ EDF
1284 = 22 × 3 × 107
AB BC AC
 LCM = 23 × 32 × 5 × 103 × 107 Then,  
ED DF EF
 N = 22 × 32 × 5 = 180
 AB.DF = ED.BC
2 2
A-5. (c) (3  0)  ( 3  0) = or AB.EF = AC.BC
(3  0)2  (k  0) 2 or BC.EF = DF.AC

 3  k2  k   3  BC.DE  AB.EF

DEEPIKA MA’AM # 8743011101 : CLASS – X : SOLUTIONS SAMPLE PAPER – 3 : INFINITY ... Think beyond...
SAMPLE PAPER – 3 LEARN MATHEMATICS BY : DEEPIKA MA’AM – 90 –

A-11. (c) Surface area of one small cube

1 2
=6× x sq units
9

Surface area of solid cube


 Surface area of one small cube

AB = (5  0)2  (0  3)2
6x 2 9
= 2

= 25  9  34 units 6  19 x 1

Y
A-12. (b) Let three terms in AP are  Required ratio = 9 : 1

T
a + d, a , a – d A-16. (c)
 a + d + a + a – d = 24

I
A-17. (d)  P(E) + P(E) = 1
 a=8
 q=1
 Middel term = a = 8

d
N n
A-13. (a) Here a= 22, d = 19 – 22 = – 3 A-18. (a)  sin  and cos  are the roots.

o
I y
Let a n be its first negative term  b 

e
sin   cos     
 a 

F B
 an  0
 a + (n – 1)d < 0

n k and sin   cos  


c

i
N
 22 + (n – 1)(–3) < 0 a
 –3n < – 25
h
I T
2
2 b
25  (sin   cos )   
 n> a
3
 9th term is the first negative of b2
2 2
term of the given AP.  sin   cos   2 sin   cos  
a2
A-14. (d)
A-15. (a) Let the sideof a solid cube be x units 2
c b
Volume of a solid cube = x3 cubic  1  2   
 a  a2
units.
This solid is cut into 27 small cubes  a 2  2ac  b2  b 2  a 2  2ac
of equal volume. A-19. (c)
1 3 A-20. (b) is correct option.
Volume of one small cube = x
27 A-21. Here, a = 2, d = 7 – 2 = 5
cubic units Let a n = 137
1  a + (n + 1)d = 137  n = 28
 Side of one small cube = x units
3 Hence, 28th term is 137.
Now, surface area of a solid cube A-22. Given : P = RTS, where T and S are
= 6 × x2 sq units points on the side RQ and RP respectively.

DEEPIKA MA’AM # 8743011101 : CLASS – X : SOLUTIONS SAMPLE PAPER – 3 : INFINITY ... Think beyond...
SAMPLE PAPER – 3 LEARN MATHEMATICS BY : DEEPIKA MA’AM – 91 –

OR
We have angle of sector = A
and radius of a sector = R

R 2 A
 Area of a sector =
360
To Prove : RPQ ~ RTS

RQ RS 1
and = A-25. We have cos =
RP RT 2

Y
As we know that
Proof : In RPQ ~ RTS
P = RTS (Given) sin = 1  cos 2 

T
and R = R (Common)
1 3

I
 RPQ ~ RTS (AA similarity) = 1 
4 2
RQ RP Now,

d
 =

N
RS RT

o
1
n sin 2 

I
(Sides of similar triangles are in the same 2 2 
sec   tan  2 2
ratio)

e
7  2sec cos ec
y
= cos  cos 
2 1

F
7 .

RQ
RP
=
RS
RT
Hence Proved.

k B cos  sin 

n
(2)2  ( 3) 2 7 3

i
N
A-23. Distance between two parallel tangents = 
2

h 7  2 2 7 3 8

I
of a circle is equal to the diameter of the

T
3
circle.
OR
 Distance between two parallel tan-
LHS = (cosec A – sin A)(sec A – cos A)
gents of the given circle = 2 × radius
= 2 × 6 = 12 cm.  1  1 
=  sin A   cos A 
A-24. Let the radius of hemispherical bowl  sin A  cos A 

= r cm  1  sin 2 A  1  cos 2 A 
Circumference of edge of hemispherical =  sin A   cos A 
  
bowl = 132 cm
 2r = 132 cos 2 A sin 2 A
= . = sin A cosA
22 sin A cos A
 2  r =132 Taking RHS
7
1 1
132  7 RHS = 
 r= = 21 cm tan A  cot A sin A  cos A
2  22 cos A sin A
 Capacity of hemispherical bowl cos A sin A
= = cosA sinA
2 3 2 22 cos2 A  sin 2 A
= r =   21 21 21
3 3 7 LHS = RHS
= 19404 cm3
DEEPIKA MA’AM # 8743011101 : CLASS – X : SOLUTIONS SAMPLE PAPER – 3 : INFINITY ... Think beyond...
SAMPLE PAPER – 3 LEARN MATHEMATICS BY : DEEPIKA MA’AM – 92 –

a  4A + 4B = 3 ...(i)
A-26. Let 3 , where a and b are coprime
b 1 1 1
and A B 
integers and b  0 . 2 2 8
1
a2  A–B= 
Squaring both sides, we get 3 = 2
. 4
b
 4A – 4B = –1 ...(ii)
Multiplying with b on both sides, we get
Adding (i) and (ii), we get
a2 1
3b = A=
b 4

Y
LHS = 3 × b = Integer
1
Putting A = in (i), we get

T
2
Integer 4
a
RHS = =
Integer

I
b 1
= Rational number 4   + 4B = 3
 4
 LHS  RHS

d
 4B = 2

N
 Our supposition is wrong.
1

o n
I
 3 is irrational  B=

y
2
Let 15  17 3 be a rational number..
e
F
1

B
When A=
a 4
 15  17 3  ,
b

n k 1 1

i
N
where a and b are coprime integers and  =
3x  y 4

h
I
b0

T
 3x + y = 4 ...(iii)
a
 17 3 =  15 1
b When B=
2
a  15b
3 = 1 1
17b  =
3x  y 2
a  15b  3x – y = 2 ...(iv)
is rational number..
17b Adding (iii) and (iv), we get
But 3 irrational. x = 1 and y = 1
Hence x = 1, y = 1
a  15b
 3  A-28. Let p(x)  4 3x 2  4 3x  3 3
17b
 Our supposition is wrong. For zeroes of p(x), put p(x) = 0
2
 15  17 3 is an irrational number..  4 3x  4 3x  3 3 = 0
Hence Proved.  3(4x 2  4x  3)  0
1 1  4x 2  4x  3  0
A-27. Let  A and B
3x  y 3x  y  [4x 2  6x  2x  3]  0
 Given equations becomes  [2x(2x + 3) – 1(2x + 3)] = 0
 (2x + 3)(2x – 1) = 0
3
A+B= 3 1
4  x= ,
2 2
DEEPIKA MA’AM # 8743011101 : CLASS – X : SOLUTIONS SAMPLE PAPER – 3 : INFINITY ... Think beyond...
SAMPLE PAPER – 3 LEARN MATHEMATICS BY : DEEPIKA MA’AM – 93 –

3 1 A-30. Given : PT is a tangent and PAB is a se-


Thus, the zeroes of p(x) are and cant to the circle with centre O. ON is
2 2
perpendicular to the chord AB.
Here, a  4 3, b  4 3, c  3 3 To prove :
3 1 (i) PA.PB = PN2 – AN2
Also, sum of zeroes =  =–1
2 2 (ii) PN2 – AN2 = OP2 – OT2
Construction : Join OP
(Coefficient of x)
= Proof :
Coefficient of x 2

Y
3 1 3
Product of zeroes =  
2 2 4

T
Constant term
=

I
Coefficient of x 2
Hence verified. (i) Consider
PA.PB = (PN – AN)(PN + BN)

d
OR

N n
= (PN – AN)(PN + AN)
4 5 3

o
I
3 ; x  0, ( AN = BN, as perpendicular from

y
x 2x  3 2 of a circle to a chord bisects the chord)

e
 PA.PB = PN2 – AN2

F
4  3x 5

B
 
x 2x  3 (ii) In right-angled ONA,
 (4 – 3x)(2x + 3) = 5x

n k OA2 = PN2 + AN2

i
N
 8x + 12 – 6x2 – 9x = 5x (using Pythagoras theorem) ...(i)

h
I
 x2 + x – 2 = 0 In right-angled ONA,
 (x + 2)(x – 1) = 0
 x+2= 0
T OA2 = ON2 + AN2
(using Pythagoras theorem) ...(ii)
or x–1= 0 Subtracting (ii) from (i), we get
 x= –2 OP2 – OA2 = PN2 – AN2
or x= 1  OP2 – OT2 = PN2 – AN2
So, the solutions of the given equation ( OA = OT, radii of the same circle)
are x = – 2 and 1.
Hence Proved.
3 OR
A-29. sin(A + 2B) =
2 Given : TP is tangent to the circle having
So, sin(A + 2B) = sin 60° centre O, PBT = 30°
Hence To Prove : BA : AT = 2 : 1
A + 2B = 60° ...(i) Proof :
Also, we have
cos(A + 4B) = 0
= cos 90°
 A + 4B = 90° ...(ii)
Subtrancting (ii) from (i), we have
B = 15° We have BPA = 90°
Put B = 15° in eq. (i), we have A = 30°. (Angle in a semicircle)
In BPA,

DEEPIKA MA’AM # 8743011101 : CLASS – X : SOLUTIONS SAMPLE PAPER – 3 : INFINITY ... Think beyond...
SAMPLE PAPER – 3 LEARN MATHEMATICS BY : DEEPIKA MA’AM – 94 –
ABP + BPA + PAB = 180° AK
(Angle sum property of triangle) = sin 45°
GK
 30° + 90° + PAB = 180° 45 1
 PAB = 60°  =
GK 2
Also POA = 2PBA
 POA = 2 × 30° = 60°  GK = 45 2 m
= PAB = 63.63 m
 OP = AP ...(i) A-32. Let the speed of the train be x km/h
(Sides opposite to equal angles are equal) Distance travelled = 360 km
In OPT, OPT = 90° 360

Y
 POT = 60° and PTO = 30°  Time taken = hours
x 5
(Angle sum property of a triangle)
According to equation,

T
Also, APT + ATP = PAO
360 360

I
(Exterior angle property) 1
=
 APT + 30° = 60° x x 5
 APT = 30° = ATP

d
360 360
 1

N
AP = AT ...(ii) 

n
 x x 5

o
(Sides opposite to equal angles are equal)

I
360(x  5)  360x

y
From (i) and (ii), we get 1

e
 x(x  5)
AT = OP

F B
= radius of the circle  1800 = x2 + 5x
and AB = 2r

n k
 x2 + 5x – 1800 = 0

i
N
 AB = 2AT
 x2 + 45x – 40x – 1800 = 0

h
I
AB  (x + 45)(x – 40) = 0

T
 =2
AT  x = – 45 or x = 40
 AB : AT = 2 : 1 Hence Proved Rejecting x = – 45
A-31. Let boy be at B, girl be at G and kite be at  Speed of the train = 40 km/h
K. OR
 KBC = 30° and AGK = 45° Let the two numbers be x and x – 5
According to question,
1 1 1
 
x  5 x 10
sin ce x15  x1 
x x 5 1
 (x  5)x  10
In right-angled BCK,
 (x – 5)x = 50
KC  x2 – 5x – 50 = 0
= sin 30°
KB  (x – 10)(x + 5) = 0
KC 1  x = 10 or x = – 5
 = When x = 10, then x – 5 = 10 – 5 = 5
KB 2
 KC = 75 m When x = –5, then x – 5 = – 5 – 5 = –10
Now, AC = GD = 30 m Thus, the required numbers are either 10
AK = KC – AC and 5 or –5 and – 10.

= 75 m – 30 m = 45 m A-33. Given : Diagonals AC and BD intersect
at O.
In right-angled KAG,
DEEPIKA MA’AM # 8743011101 : CLASS – X : SOLUTIONS SAMPLE PAPER – 3 : INFINITY ... Think beyond...
SAMPLE PAPER – 3 LEARN MATHEMATICS BY : DEEPIKA MA’AM – 95 –

Now, Vol. of cylinder = r 2 h

=  (60) 2 180  648000 cm3


Vol. of water left in the cylinder
= Vol. of cylinder – Vol. of solid
AB || DC
= 648000 cm3  288000  cm3
OA OB
To prove :  3
OC OD = 360000 cm
Proof : In AOB and COD 22 3
= 360000  cm

Y
1 = 2 7
3 = 4 [Alternate angle]
= 1131428.57 cm3
 AOB ~ COD [Alternate angle]

T
1131428.57
OA OB = l = 1131.42 l

I
 = 1000
OC OD
OR
[Corresponding sides of similar triangles]

d
Rs. 24, is the cost for fencing 1m of cir-

N
A-34. Radius of cone = 60 cm

n
cular field.

o
Height of cone = 120 cm

I
Rs. 5280, is the cost for fencing
1
e y
 5280 = 220 m of circular field

F
=

B
24

k
Circumference of the field = 220 m

n
 2r = 220

i
N h
22

I
 2  r =220

T 
7

r=
220  7
44
= 35m

1 2  Area of the field = r2 = (35)2


Volume of cone = r h = 1225 cm2
3
1 Cost of ploughing = Rs. 0.50 per m2
2
=  (60)  120 Total cost of ploughing the field
3
= Rs. 1225 × 0.50
= 14400 cm3.
Radius of hemipshere = 60 cm 1225  22 1
= Rs. = Rs. 175 × 11
1
 Volume of hemisphere 7 2
= Rs. 1925
2 3 2
= r   (60)3
3 3
= 144000 cm3
 Volume of solid
=Vol. of cone + Vol. of hemisphere
= 144000 cm3  144000 cm3

= 288000  cm3

DEEPIKA MA’AM # 8743011101 : CLASS – X : SOLUTIONS SAMPLE PAPER – 3 : INFINITY ... Think beyond...
SAMPLE PAPER – 3 LEARN MATHEMATICS BY : DEEPIKA MA’AM – 96 –
A-35. Let A = 57, h = 3
xi  A
No. of mineral No. of boxes Class marks u i  fi u i
h
water bottles (fi ) (x i )
49.5  52.5 20 51 2 40
52.5  55.5 120 54 1 120
55.5  58.5 105 57  A 0 0
58.5  61.5 125 60 1 125
61.5  64.5 30 63 2 60
Total n  400 fi u i  25

Y
Here A = 57, h = 3, n = 400 and = 300000

T
fi u i  25 a 4 = 180000 + 180000

I
By step-deviation method = 360000
1 Total production of first four years

d
Mean, x = A + h × n × fi u i

N n
= a1  a 2  a 3  a 4

o
1

I
= 57  3   25 = 180000 + 240000
400

e y
F
+ 300000 + 360000

B
75
= 57 

k
400 = 1080000 sets

n
= 57 + 0.1875

i
N
OR

h
= 57.1875  57.19 (app.)

I
The production in first 5 years
A-36. (i)


a 5 = a + (n – 1)d
420000 = a + 4d ...(i)
T =
5
2
[2 × 180000+4×60000]

a 8 = a + (n – 1)d = 15,00,000 sets


 600000 = a + 7d ...(ii) A-37. (i) Legnth AR = length BQ
Subtracting (i) from (ii), we get H2
 length AR =  3  m ...(i)
 d = 60000  

Now putting the value of d in eqn.


(i), we get

 a = 180000
Production in first year = 180000
(ii) Common difference = 60000
(iii) a1 = 180000 Also in ARS,
a 2 = 180000 + 60000 SR
tan 30° =
= 240000 AR

a 3 = 180000 + 120000
DEEPIKA MA’AM # 8743011101 : CLASS – X : SOLUTIONS SAMPLE PAPER – 3 : INFINITY ... Think beyond...
SAMPLE PAPER – 3 LEARN MATHEMATICS BY : DEEPIKA MA’AM – 97 –

1 H 8
= AB = (5  5)2  (3  0)2
3 AR
AR = 3(H  8)m ...(ii)
= 0  (3)2  3 units
(ii) equating (i) and ii), we get = OC

H2 BC = (5  0)2  (3  3)2


= 3(H  8)
3
= 25  5
 H = 11 m
Perimeter OABC
SQ

Y
(iii) I n SQB, sin 60° =
SB = OA + AB + BC + OC
2 = 5 + 3 + 5 + 3 = 16 units

T
 SB = (11  2) 
3

I
OR
18  3 Coordinates of O and B are (0, 0)
=  6 3m

d
3

N
and (5, 3).

n
Hence distance of the balloon from

o
I
lower window is 6 3 m . 50 5

y
Mid-point of OB = = and
2 2

e
OR

F B
In SRA, 03 3
=

k
SR 2 2

n
sin 30° =

i
N
AS Coordinates A and C are (5, 0) and

h
I
 AS = (11 – 8) × 2 = 6 m (0, 3)

upper window is 6 m.
T
Hence, distance of the balloon from
Mid-point of AC =
50
2
and
03
2

A-38. (i) Coordinates of O are (0, 0) and co- 5 3 5 3


= and =  , 
2 2 2 2
ordinates of B are (5, 3).
Coordinate of mid-point of AC and
Length of diagonal OB
5 3
= (5  0)2  (3  0)2 OB are  ,  .
2 2
= 25  9  34 units
(ii) In quadrilateral OABE
O + A + B + C = 360°

 90° + 90° + B + 90° = 360°


B = 360° – 270°
= 90°
(iii) Perimeter of OABC
OA = 5 units
DEEPIKA MA’AM # 8743011101 : CLASS – X : SOLUTIONS SAMPLE PAPER – 3 : INFINITY ... Think beyond...
SAMPLE PAPER – 3 LEARN MATHEMATICS BY : DEEPIKA MA’AM – 98 –
— Notes —
_________________________________________________________________________________________________________

_________________________________________________________________________________________________________

_________________________________________________________________________________________________________

_________________________________________________________________________________________________________

_________________________________________________________________________________________________________

Y
_________________________________________________________________________________________________________

_________________________________________________________________________________________________________

I T
_________________________________________________________________________________________________________

_________________________________________________________________________________________________________

d
N n
_________________________________________________________________________________________________________

o
I y
_________________________________________________________________________________________________________

e
F B
_________________________________________________________________________________________________________

k
_________________________________________________________________________________________________________

n
i
N h
_________________________________________________________________________________________________________

I T
_________________________________________________________________________________________________________

_________________________________________________________________________________________________________

_________________________________________________________________________________________________________

_________________________________________________________________________________________________________

_________________________________________________________________________________________________________

_________________________________________________________________________________________________________

_________________________________________________________________________________________________________

_________________________________________________________________________________________________________

_________________________________________________________________________________________________________

_________________________________________________________________________________________________________

_________________________________________________________________________________________________________

_________________________________________________________________________________________________________

_________________________________________________________________________________________________________

DEEPIKA MA’AM # 8743011101 : CLASS – X : SOLUTIONS SAMPLE PAPER – 3 : INFINITY ... Think beyond...
SAMPLE PAPER – 4 LEARN MATHEMATICS BY : DEEPIKA MA’AM – 99 –

X - MATHEMATICS
SAMPLE PAPER - 4
Time Allowed : 3 Hours] [Maximum Marks : 80
General Instructions :
1. This Question Paper has 5 Sections A - E.
2. Section A has 20 MCQs carrying 1 mark each.
3. Section B has 5 questions carrying 02 marks each.
4. Section C has 6 questions carrying 03 marks each.

Y
5. Section D has 4 questions carrying 05 marks each.

T
6. Section E has 3 case based integrated units of assessment (04 marks each)with sub-parts of the
values of 1,1 and 2 marks each respectively.

I
7. All Questions are compulsory. However an internal choice in 2 Qs. of 5 marks, 2 Qs of 3 marks
and 2 Questions of 2 marks has been provided. An internal choice has provided in the 2 marks

d
N
questions of Section E.

o n
I
8. Draw neat figures wherever required. Take  = 22/7 wherever required if not stated.

SECTION - A
e y
F B
Section A consists of 20 questions of 1 mark each.
Q1. The ratio between LCM and HCF of 5, 15, 20 is
k
in
N
(a) 9 : 1 (b) 4 : 3 (c) 11 : 1 (d) 12 : 1

h
I
If (1 – p) is a root of the equation x2 + px + 1 – p = 0, then roots are

T
Q2.
(a) 0, 1 (b) –1, 1 (c) 0, –1 (d) –1, 2
Q3. The HCF of 2472, 1284 and a third number N is 12. If their LCM is 23 × 32 × 5 × 103 × 107,
then the number N is :
(a) 22 × 32 × 7 (b) 22 × 33 × 103 (c) 22 × 32 × 5 (d) 24 × 32 × 11
1 1
Q4. The roots of the equation x   5 are
x 5
1
(a) 5, (b) 5, – 5 (c) –5, – 5 (d) 2, –2
5
Q5. The distance of the point (, ) from y-axis is
(a)  units (b) |  | units (c)  units (d) |  | units
Q6. The height of mountains is found out using the idea of indirect measurements which is based
on the
(a) principle of congruent figures (b) principle of similarity of figures
(c) principle of equality of figures (d) none of these

Q7. If tan   cot   4, tan 4   cot 4  


(a) 196 (b) 194 (c) 192 (d) 190

DEEPIKA MA’AM # 8743011101 : CLASS – X : SAMPLE PAPER – 4 : INFINITY ... Think beyond...
SAMPLE PAPER – 4 LEARN MATHEMATICS BY : DEEPIKA MA’AM – 100 –

1  cot 2 A
Q8. 
1  tan 2 A
(a) tan2A (b) cot2A (c) cosec2A – 1 (d) 1 – sin2A
PQ QR PR
Q9. In PQR and MNS,   , then symbolically we write it as
NS MS MN
(a) PQR ~ MNS (b) PQR ~ SMP (c) QRP ~ NSM (d) QRP~ SMN
Q10. If in two right triangles, hypotenuse and one side of one triangle are proportional to the
hypotenuse and one side of the other triangle, then the two triangles are similar. This may be
referred to as the

Y
(a) AAA similarity criterion (b) ASSS similarity criterion

T
(c) SAS similarity criterion (d) RHS similarity criterion

I
Q11. If tangents PA and PB from a point P to a circle with centre O are inclined to each other at
angle of 80°, then POA is equal to

d
(a) 50° (b) 60° (c) 70° (d) 80°

N n
Q12. The area of the square inscribed in circle of diameter p is

yo
Q13.
(a) p2 cm 2 (b)
p2

F
2

Icm 2

i
(c)

n k
p 2
2
cm

B e (d)
p2
2
cm 2

Two cylindrical cans have equal base areas. If one of the can is 15 cm high and other is 20 cm

N
high, find the ratio of their volumes.

h
I
(a) 2 : 3 (b) 3 : 4 (c) 4 : 3 (d) 3 : 2
Q14. The median from the table is
Value 7 8 9 10 11 12 13
T
Frequency 2 1 4 5 6 1 3
(a) 11 (b) 10 (c) 12 (d) 11.5
Q15. If the circumference of a circle is 352 metres, then its area in square metres is
(a) 5986 (b) 6589 (c) 7952 (d) 9856
Q16. If the difference of mode and median of data is 26, then the difference of median and mean is
(a) 13 (b) 26 (c) 8 (d) 32

Q17. If two towers of heights h1 and h 2 subtend angles of 60° and 30° respectively at the mid-

point of the line joining their feet, then h1 : h 2 =


(a) 1 : 2 (b) 1 : 3 (c) 2 : 1 (d) 3 : 1
Q18. In the figure, there are two concentric circles with centre O
and radii 5 cm and 3 cm. PA and PB are tangents to these
circles from an external point P. If PA = 12 cm, then length of
PB (in cm) is

(a) 10 (b) 4 10 (c) 12 (d) 178

DEEPIKA MA’AM # 8743011101 : CLASS – X : SAMPLE PAPER – 4 : INFINITY ... Think beyond...
SAMPLE PAPER – 4 LEARN MATHEMATICS BY : DEEPIKA MA’AM – 101 –
Direction : In the question number 19 and 20, a statement of Assertion (A) is followed by a
statement of Reason (R). Choose the correct option.
Q19. Statement A (Assertion) : If one zero of the polynomial p(x) = (k2 + 4)x2 +9x + 4k is the
reciprocal of the other zero then k = 2.
Statement R (Reason) : If (x – a) is a factor of the polynomial p(x), then a is zero of p(x).
(a) Both assertion (A) and reason (R) are true and reason(R) is the correct explanation of
assertion (A).
(b) Both assertion (A) and reason (R) are true and reason (R) is not the correct explanation

Y
of assertion (A).
(c) Assertion (A) is true but reason (R) is false.

T
(d) Assertion (A) is false but reason (R) is true.

I
Q20. Statement A (Assertion) : The point (0, 6) lies on y-axis.
Statement R (Reason) : The x co-ordinate on the point on y-axis.

d
(a) Both assertion (A) and reason (R) are true and reason(R) is the correct explanation of

N n
assertion (A).

o
I y
(b) Both assertion (A) and reason (R) are true and reason (R) is not the correct explanation

e
F
of assertion (A).
(c) Assertion (A) is true but reason (R) is false.

k B
n
(d) Assertion (A) is false but reason (R) is true.

i
Q21.

Q22.
I N T h
SECTION - B
Section B consists of 5 questions of 2 marks each.
Find the number of solutions of the following pair of linear equations :
3x – 3y = 5, 7x – 2y = 2
In figure, AB || DE and BD || EF. Prove that DC2 = CF × AC.

Q23. In the given figure, O is the centre of circle. AB is a chord and


the tangent AC at A makes an angle iof 60° with AB. Find
AOB.
Q24. A chord of circle of a radius 28 cm subtends a right angle at the centre. What is the area of
the minor sector ?
OR
The length of the minute hand of a clock is 14 cm. Find the area swept by the minute hand
in 5 minutes.

DEEPIKA MA’AM # 8743011101 : CLASS – X : SAMPLE PAPER – 4 : INFINITY ... Think beyond...
SAMPLE PAPER – 4 LEARN MATHEMATICS BY : DEEPIKA MA’AM – 102 –
2 tan 
Q25. For  = 30°, verify that : sin 2  .
1  tan 2 
OR
23 23
3 3 x y
If x  p cos  and y  q sin , prove that     1.
p q

SECTION - C
Section C consists of 6 questions of 3 marks each.

Y
Q26. Prove that 5 is an irrational.

T
143
Q27. The difference of an integer and its reciprocal is . Find the integer..

I
12
OR

d
Find the positive value of k, for which the equation x2 + kx + 64 = 0 and x2 – 8x + k = 0 will

N n
both have real roots.

o
I y
Q28. If  and  are zeroes of the quadratic polynomial 4x2 + 4x + 1, then form a quadratic

e
F
polynomial whose zeroes are 2 and 2.

Q29. If tan(A  B)  3 and tan(A  B) 


1
k B
; 0° < A + B  90; A > B, find A and B.
3

i n
N
Q30. The lengths of tangents drawn from an external point (point outside the circle) to a circle are

h
I T
equal. Prove it.
OR
ABC is an isosceles triangle, in which AB = AC, circumscribed about a circle. Show that BC
is bisected at the point of contact.
Q31. Radhika, a good student has ability to save her pocket money into her own piggy bank.
Saving money is a skill that will be useful at all stages in person’s life.
Radhika’s piggy bank contains hundred 50p coins, fifty Rs. 1 coins, twenty Rs. 2 coins and ten
Rs. 5 coins. One day she decided to take out money from her piggy bank. If it is equally likely
that one of the coins will fall out when the piggy bank is turned upside down, find the probabil-
ity that the coin (i) will be a 50p coin (ii) will not be a Rs. 5 coin (iii) will be Rs. 2 coin.

SECTION - D
Section D consists of 4 questions 5 marks each.

2 11 2
Q32. Find the zeroes of the quadratic polynomial 7y  y  and verify the relationship be-
3 3
tween the zeroes and the coefficients.
OR
If the zeroes of x2 – px + 6 are in the ratio 2 : 3, find p.
DEEPIKA MA’AM # 8743011101 : CLASS – X : SAMPLE PAPER – 4 : INFINITY ... Think beyond...
SAMPLE PAPER – 4 LEARN MATHEMATICS BY : DEEPIKA MA’AM – 103 –
Q33. Prove that the lengths of tangents drawn from an external point to a circle are equal.
Q34. A cylindrical vessel with internal radius 5 cm and height of 10.5 cm is full of water. A solid
cone of base radius 3.5 cm and height 6 cm is completely in water. Find the volume of
(i) water displaced out of the cylindrical vessel
(ii) water left in the cylindrical vessel.
OR
A sector of a circle of radius 12 cm has the angle 120°. It is rolled up so that two bounding
radii are joined together to form a cone. Find the volume of the cone.
Q35. The median of the distribution given below is 14.4. Find the values of x and y, if the sum of

Y
frequency is 20.

T
Class interval Frequency
06 4

I
6  12 x
12  18 5

d
18  24 y

N n
24  30 1

o
Q36.

F I SECTION - E

B
Case study based questions are compoulsory.

i n k
e y
SOM, a firm organised an athletic meet. They made a rectangular grid on their ground.

I N T h

Points P(1, 2) and Q(7, 8) were marked for disc throw competition. Disc were made to
throw from point P towards point Q.
(i) Find the PM.
(ii) Find PN.
(iii) Find the ratio in which M divides PQ.
OR
Find PM : QN.
DEEPIKA MA’AM # 8743011101 : CLASS – X : SAMPLE PAPER – 4 : INFINITY ... Think beyond...
SAMPLE PAPER – 4 LEARN MATHEMATICS BY : DEEPIKA MA’AM – 104 –

Q37. The farmers in the field make a heap of wheat in the field in the form of a cone. The base
diameter of heap formed in the field is 24 m and height of heap formed is 3.5 m.

Answer the questions based on above.

Y
(i) What will be the slant height of heap formed in the field ?

T
(ii) How much canvas cloth is required to just cover the heap ?
(iii) Find the volume of heap of wheat ?

I
OR

d
Farmer packed the wheat into bags. If volume of each bag of wheat is 0.48 m3, then

N n
two many bags of wheat can be made ?

o
I y
Q39. A boy is standing on the top of light house. He observed boat P and boat Q are approaching

e
to light house from opposite directions. He finds that angle depression of boat P is 45° and

F B
angle of depression of boat Q is 30°. He also knows that height of the light house is 100 m.

n k
i
I N T h

Based on the above information, answer the following questions :


(i) Find ACD.
(ii) Find the length of CD.
(iii) Find the length of BD.
OR
Find the length of AC.

DEEPIKA MA’AM # 8743011101 : CLASS – X : SAMPLE PAPER – 4 : INFINITY ... Think beyond...
SOLUTIONS SAMPLE PAPER – 4 LEARN MATHEMATICS BY : DEEPIKA MA’AM – 105 –

XII - MATHEMATICS
SOLUTIONS : SAMPLE PAPER - 4
A-1. (d) 5, 15 = 5 × 3, 20 = 2 × 2 × 5 Squaring both sbides
LCM (5, 15, 20) = 5 × 3 × 2 × 2 = 60
tan 4   cot 4   2  196
HCF (5, 15, 20) = 5
tan 4   cot 4   194
LCM 60 12
Ratio =   = 12 : 1
HCF 5 1 1  cot 2 A cos ec 2 A
A-8. (b) 
A-2. (c) (1 – p) is a root 1  tan 2 A sec2 A

Y
 (1 – p)2 + p(1 – p) + 1 – p = 0
cos 2 A
 (1 – p)[1 – p + p + 1] = 0 =  cot 2 A

T
2
 (1 – p) (2) = 0 sin A

I
 p=1 A-9. (d) QRP ~ SMN
x2 + x = 0 A-10. (d) RHS similarity criterion

d
A-11. (a) In OAP and OBP
One root = 0 and another root = –1

N n
 roots are 0 and –1

o
I y
A-3. (c) 2472 = 23 × 3 × 103

e
F
1284 = 22 × 3 × 107
 LCM = 33 × 32 × 5 × 103 × 107
k B
N = 22 × 32 × 5 = 180

i n OA = OB [Radii]

N

h
A-4. (a) We have PA = PB [Length of tangent

I T
from an external point are equal]
1 1
x 5 OP = OP [Common]
x 5
 OAP OBP
x 2  1 26 [SSS congruence rule]

x 5  POA = POB = ½ AOB
 AOB + APB = 180°
5x 2  5  26x AOB + 80° = 180°

5x 2  26x  5  0  AOB = 180° – 80° = 100°

5x 2  25x  5  0 1
 100  50
 POA =
5x(x – 5) – 1(x – 5) = 0 2
(5x – 1)(x – 5) = 0 A-12. (b)

1
 x= and x = 5
5
A-5. (b) Distance = |  | units
A-6. (b) Suare is inscribed in a circle of di-
A-7. (b) We have tan   cot   4 ameter p cm are so dash angle of
square = 90°
Squaring both sides,
BAD = BCD = 90°
tan 2   cot 2   14 There are angle in semicircle.
DEEPIKA MA’AM # 8743011101 : CLASS – X : SOLUTIONS SAMPLE PAPER – 4 : INFINITY ... Think beyond...
SOLUTIONS SAMPLE PAPER – 4 LEARN MATHEMATICS BY : DEEPIKA MA’AM – 106 –
 diameter of circle is equal to di- h1
agonal of square.  tan 60  3
h
In BCD
 h1  3x ...(i)
DB2 = DC2 + BC2
DB2 = 2BC2 (BC = DC) h2 1
 tan 30 
x 3
DR 2 p 2
BC2 =  cm 2 1
2 2
 h2  x
A-13. (b) Let the base area of first cylinder is 3
r2.

Y
h1 3x 3
 Base area of second cylinder is  
h2 1
also r2 x 1

T
3
h1  15cm, h 2  20cm

I
 h1 : h 2  3:1
2
r h1 15 A-19. (a) Both Assertion (A) and Reason (R)
Ratio of volumes = 2

20 are true and Reason (R) is the cor-

d
r h 2

N n
3 rect explanation of Assertion (A).

o
=

I
4 A-20. (a) Both Assertion (A) and Reason (R)
Volume of first cylinder : Volume of

e y
are true and Reason (R) is the cor-
rect explanation of Assertion (A).

F B
second cylinder = 3 : 4
A-14. (b) A-21. Given equations are

n k
3x – 3y = 5; 7x – 2y = 2

i
N
Value 7 8 9 10 11 12 13
Here, a1  3, b1  3, c1  5

h
f 2 1 4 5 6 1 3

I T
c.f . 2 3 7 12 18 19 22 a  7, b  2, c  2
2 2 2

n a1 3 b1 3
n = 22,   11, so, median is 10.  , 
2 a 2 7 b 2 2
Median = mean of 11th and 12th
observations = 10 a 1 b1
 a b
2 2
176
A-15. (d)  2r = 352  r =  Given pair of linear equations has a

unique solution.
 Area = r2. A-22. Given : ABC in which DE || AB and
 176  176 176  176  7 BD || EF.
= =
  22 To Prove : DC2 = CF × AC
= 9856 m2 Proof : In ABC,
A-16. (a) DE || AB
A-17. (d) CD CE
 = ...(i)
AC BC
(Baisc Proportionallity Theorem)
Again in CDB, EF || BD
CF CE
 = ...(ii)
CD CB
DEEPIKA MA’AM # 8743011101 : CLASS – X : SOLUTIONS SAMPLE PAPER – 4 : INFINITY ... Think beyond...
SOLUTIONS SAMPLE PAPER – 4 LEARN MATHEMATICS BY : DEEPIKA MA’AM – 107 –
From (i) and (ii), we get 360
Angle swept in 5 minutes = 5
CD CF 60
=
AC CD = 30°
 CD2 = CF × AC Area swept in 5 minutes
Hence Proved. 22 14 14  30
= 
A-23. AC is tangent and OA is radius 7 360
OA  AC
11 2  14 11 14
[ Tangent to a circle is perpendicular = 
6 3
to radius through point of contact]

Y
154 2
= cm
3

T
A-25. LHS = sin 2 = sin(2 × 30°) = sin 60°

I
3
= ...(i)
2
 OAC = 90°
d
N n
 OAB + BAC = 90° 2 tan  2 tan 3

o
RHS = 

I
1  tan  1  tan 2 30

y
 OAB + 60° = 90°
( Given BAC = 60°)
e
F B
1 2
 OAB = 30° 2
3  3
In AOB,

n k =  1 
2 3 1

i
N
OA = OB (Radii of the same 1   3
 3

h
circle)

I T
 OAB = OBA (Angle opposite
2 3 3
to equal sides of triangle are equal) =   ...(ii)
3 4 2
 OBA = 30°
In AOB, From (i) and (ii), LHS = RHS.
 AOB + OBA + OAB = 180° Hence Proved.
(Angle sum property of triangle) OR
 AOB + 30° + 30° + 30° = 180° 3
We have x = p cos 
 AOB = 180° – 60° = 120°
3
A-24. Area of the sector of angle  and y = q sin 

 3
 r 2 as x = p cos 
=
360
x 3
90 22  = cos 
  (28) 2 p
=
360 7
13
1 22 x
=   28  28  cos =  
4 7 p
= 616 cm2 Similarly
OR 13
y
Length of minute hand of clock = 14 cm sin =  
q
DEEPIKA MA’AM # 8743011101 : CLASS – X : SOLUTIONS SAMPLE PAPER – 4 : INFINITY ... Think beyond...
SOLUTIONS SAMPLE PAPER – 4 LEARN MATHEMATICS BY : DEEPIKA MA’AM – 108 –

as we know sin 2   cos 2   1 1


Rejecting x =  , because x is an inte-
12
23 23
x  y ger.
So,     1
p q  x = 12
Hence proved.  The required integer is 12.
OR
a
A-26. Let 5 is a rational number and 5  b , If the equation x2 + kx + 64 = 0 has real
where a and b are coprime and b  0 . roots, then D  0.

 k 2  4  1 64  0

Y
2
a
Now, ( 5)2 =   2
b  k  256

T
 5b2 = a2 ...(i)  k 2  (16) 2

I
 5 is a factor of a2  k  16 [ k  0] ...(i)
 a is also divisible by 5.

d
If the equation x2 – 8k + k = 0 has real

N n
Let a =5c, where c is some integer roots, then D  0

o
I
Substituting a = 5c in (i), we get

y
 64 – 4k  0  4k  64
5b = (5c)2  5b2 = 25c2

e
k  16 ...(ii)

F B
 b2 = 5c2
From (i) and (ii), we get
 5 is a factor of b2

n k k = 16

i
N
 5 is a factor of b. A-28. Let p(x) = 4x2 + 4x + 1
 5 is a common factor of a and b
h
I T
 ,  are zeroes of p(x)
This contradicts the fact that a and b are
coprime so, our assumption is wrong. b
    = sum of zeroes =
Hence, 5 is irrational. a
A-27. Let the integer be x and its reciprocal be 4
    1 ...(i)
1 4
.
x c
According to question, Also   = Product of Zeroes =
a
1 143 1
x =   = ...(ii)
x 12 
4
Now a quadratic polynomial whose ze-
x 2  1 143
 = roes are 2 and 2
x 12
x2–(Sum of zeroes)x + Product of zeroes
 12x2 – 12 = 143x
2
 12x2 – 143x – 12 = 0 = x  (2  2)x  2  2
2
 12x2 – 144x + x – 12 = 0 = x  2(  )x  4
 12x(x – 12) + 1(x – 12) = 0 2 1
= x  2  (1)x  4 
 (x – 12)(12x + 1) = 0 4
1 [Using eqn. (i) and (ii)]
 x = 12 or x =  = x2 + 2x + 1
12
DEEPIKA MA’AM # 8743011101 : CLASS – X : SOLUTIONS SAMPLE PAPER – 4 : INFINITY ... Think beyond...
SOLUTIONS SAMPLE PAPER – 4 LEARN MATHEMATICS BY : DEEPIKA MA’AM – 109 –
AB = AC (Given) ...(i)
A-29. tan(A + B) = 3
AF = AE
 tan (A + B) = tan 60°
(Tangent from an external point A to a
 A + B = 60° ...(i) circle are equal) ...(ii)
1 Subtracting (ii) from (i), we get
tan(A + B) =
3 AB – AF = AC – AE
 tan(A – B) = tan 30°  BF = CE ...(iii)
 A – B = 30° ...(ii) Now, BF = BD
Adding (i) and (ii), we get (Tangents from an external point B to a
2A = 90°

Y
circle are equal)
90 Also, CE = CD
 A= = 45°

T
2 (Tangents from an external point C to a
From (i), circle are equal)

I
45° + B = 60°  BD = CD
 B = 60° – 45° = 15°  BC is bisected at the point of con-
tact. Hence Proved.
d
N
Hence, A = 45°, B = 15°
A-30. Given : A circle C(O, r) is a point out- A-31. Total number of coins

o n
I y
side the circle and PA and PB are tan- = 100 + 50 + 20 + 10 = 180

e
gents to a circle. (i) Number of 50p coins = 100

F B
 Probability of getting a 50p coins

n k 100 5

i
N
=
180 9

h
I
(ii) Number of Rs. 5 coins = 10
To Prove : PA = PB
Construction : Join OA, OB and OP
T Number of coins other than a Rs. 5
coin = 180 – 10 = 170
Proof : In OAP and OBP,  Probability of not getting a Rs. 5
OAP = OBP = 90° 170 17
coin = 
(Radius is perpendicular to the tangent at 180 18
the point of contact) (iii) Number of Rs. 2 coin = 20
OA = OB (Radii of the same  Probability of getting Rs. 2 coin
circle)
20 1
OP = OP (Common) = 
180 9
 OAP  OBP (RHS congruence
rule) 11 2
A-32. Here p(y) = 7y 2  y 
 PA = PB [CPCT] 3 3
Hence proved. For zeroes of p(y), p(y) = 0
OR
2 11 2
Given : In an isosceles  7y  y   0
3 3
ANC, AB = AC, circum-
scribed a circle.  21y 2  11y  2  0
To prove :
BD = DC  21y 2  14y  3y  2  0
Proof : Here,  7y(3y  2)  1(3y  2)  0
DEEPIKA MA’AM # 8743011101 : CLASS – X : SOLUTIONS SAMPLE PAPER – 4 : INFINITY ... Think beyond...
SOLUTIONS SAMPLE PAPER – 4 LEARN MATHEMATICS BY : DEEPIKA MA’AM – 110 –

 (7y  1)(3y  2)  0 when m = –1, eq. (i) becomes


5×–1= p
1 2
 y= ,  p= –5
7 3
 p= ±5
1 2 A-33. Given : In circle, O is the centre. P is an
 zeroes are and
7 3 external point and PA and PB are the tan-
gents drawn.
11 2
Also a = 7, b = , c To prove : PA = PB
3 3
Construction : Join OA, OB and OP
1 2 3  14 11

Y
Sum of zeroes =   =
7 3 21 21

T
b  (11/ 3) 11
Also, = 
a 7 21

I
b Proof : Since PA and PB are the tangents
 Sum of zeroes = and OA and OB are the radii of a circle.

d
a

N
 OA PA and OB PB
1 2 2

o n
[Tangent to a circle makes angle 90° with

I
and product of zeroes =  =

y
7 3 21 the radius at the point of contact]

e
F
 OAP = OBP = 90°

B
2
c 2
3
Also =  Now, in OAP and OBP,

k
a 7 21
OA = OB (Radii)

i n
N
c OP = OP (Common)

h
 Produt of zeroes =

I
a OAP = OBP (Each 90°)
OR
p(x) = x2 – px + 6 T  OAP  OBP
[By RHS congruence rule]
Let zeroes aer 2m and 3m PA = PB [By CPCT]
Hence Proved.
b
Sub of zeroes =  A-34. Height of cylinder = h = 10.5 cm
a
Radius of cylinder = r = 5 cm
( p)
 2m + 3m =
1
 5m = p ...(i)
c
Product of zeroes =
a
6
 2m×3m =
1  Volume of cylinder = r2h
 6m2 = 6
22 105
m2
=1 =  5 5 cm3
 7 10
 m= ±1 = 825 cm3
When m = 1, eq. (i) becomes Now, radius of base of cone = R = 3.5cm
5×1= p Height of cone = H = 6cm
 p= 5
DEEPIKA MA’AM # 8743011101 : CLASS – X : SOLUTIONS SAMPLE PAPER – 4 : INFINITY ... Think beyond...
SOLUTIONS SAMPLE PAPER – 4 LEARN MATHEMATICS BY : DEEPIKA MA’AM – 111 –

1 2 = 189.61 cm3.
 Volume of cone = R H
3 A-35.
1 22 35 35 Class Interval f cf
=    6 06 4 4
3 7 10 10
6  12 x 4x
= 77 cm3 12  18 5 9x
(i) Volume of water displaced out of the 18  24 y 9xy
cylinderical vessel = Volume of cone 24  30 1 10  x  y
= 77 cm3. Total 10  x  y
(ii) Volume of water left in cylindrical Here n = 20

Y
vessle n
= Vol. of cylinder – Vol. of cone  = 10, Median =14.4

T
2
= 825 – 77 = 748 cm3  Median class = 12 – 18

I
OR Here l = 12, cf = 4 + x
 r f = 5, h = 6
Length of the arc =
d
N n
180  n  cf 

o
2

I
120  Median = l   f   h
=
180
  12

e y 

F B
= Circumference of the base of the cone  10  (4  x) 
 14.4 = 12 +   6

k
Let the radius of cone be R cm  5 

i n
N
120  x = 4, y = 6 (As x + y = 10)

h
 2   R =   12

I
180 A-36. (i) 8 units

 R=
2 12

3 2
= 4 cm T (ii) 4 2 units
(iii) 1 : 2 OR 1 : 1
A-37. (i) Diameter of base of heap = 24 m
24
Radius of base of heap = m
2
Now, R = 4 cm, l = 12 cm
= 12 m
 h2 = l2 – R2 = 122 – 42 Height of heap = 3.5 m
= 144 – 16 Let l be the slant height of heap
 h2 = 128
 l= r2  h2
 h= 128  8 2 cm
 Volume of the cone = (12)2  (3.5) 2
1 =
=   R 2  h 144  12.25
3
= 156.25
1 22 2
=   (4)  8  2 l= 156.25 = 12.5m
3 7
(ii) Canvas cloth required to cover the
1 22
=   16  8  1.414 cm3 heap = rl
3 7
DEEPIKA MA’AM # 8743011101 : CLASS – X : SOLUTIONS SAMPLE PAPER – 4 : INFINITY ... Think beyond...
SOLUTIONS SAMPLE PAPER – 4 LEARN MATHEMATICS BY : DEEPIKA MA’AM – 112 –

22
=  12  12.5 = 471.42 m2.
7
(iii) Volume of heap of wheat
1 2 1 22
= r h    12  12  3.5
3 3 7
= 22 × 4 × 12 × 0.5 = 528 m3
OR
Volume of one bag = 0.48 m3

Y
Number of bags required
528

T
=  = 1100
0.48

I
A-38. (i) ACD =CAX (Alternate angles)
 ACD = 45°

d
(ii) In right-angled ADC,

N o n
AD

I
tan 45° =

y
CD

e
F
 CD = AD = 100 m
(ii) In right-angled ADB,

k B
n
AD

i
N
tan 30° =
DB

h
I
[ ABD = BAY]
 BD = AD cot 30°
= 100  3 m
T
OR
In ADC,
AD
sin 45° =
AC

 AC = AD × 2  100 2 m

 1 
sin 45  
 2

DEEPIKA MA’AM # 8743011101 : CLASS – X : SOLUTIONS SAMPLE PAPER – 4 : INFINITY ... Think beyond...
SAMPLE PAPER – 5 LEARN MATHEMATICS BY : DEEPIKA MA’AM – 113 –

X - MATHEMATICS
SAMPLE PAPER - 5
Time Allowed : 3 Hours] [Maximum Marks : 80
General Instructions :
1. This Question Paper has 5 Sections A - E.
2. Section A has 20 MCQs carrying 1 mark each.
3. Section B has 5 questions carrying 02 marks each.
4. Section C has 6 questions carrying 03 marks each.

Y
5. Section D has 4 questions carrying 05 marks each.

T
6. Section E has 3 case based integrated units of assessment (04 marks each)with sub-parts of the
values of 1,1 and 2 marks each respectively.

I
7. All Questions are compulsory. However an internal choice in 2 Qs. of 5 marks, 2 Qs of 3 marks
and 2 Questions of 2 marks has been provided. An internal choice has provided in the 2 marks

d
N
questions of Section E.

o n
I
8. Draw neat figures wherever required. Take  = 22/7 wherever required if not stated.

SECTION - A
e y
F B
Section A consists of 20 questions of 1 mark each.
Q1.

n k
4 bells toll together at 9.00 am. They toll after 7, 9, 11 and 12 seconds respectively. How

i
N
many times will they toll together again in the next 3 hours ?

h
I
(a) 3 (b) 4 (c) 5 (d) 6

Q2. T
A quadratic polynomial whose zeroes are
3
5
1
and  are ________.
2

(a) 10x2 – x – 3 (b) 10x2 + x – 3 (c) 102 – x + 3 (d) none of these


Q3. If the zeroes of the quadratic polynomial x2 + (a + 1)x + b are 2 and – 3, then
(a) a = –7, b = –1 (b) a = 5, b = 1 (c) a = 2, b = – 6 (d) a = 0, b = – 6
Q4. Three runners running around a circular track, can complete one revolution in 2, 3 and 4 hrs
respectively. They will meet again at the starting point after
(a) 8 hrs (b) 6 hrs (c) 12 hrs (d) 18 hrs
Q5. If A and B are the points (–3, 4) and (2, 1) respectively, then the coordinates of the point on
AB produced such that AC = 2BC are
(a) (2, 4) (b) (3, 7) (c) (7, – 2) (d) none of these
Q6. What is the largest number that divides each one of 1152 and 1664 exactly ?
(a) 32 (b) 64 (c) 128 (d) 256
Q7. In right triangle, B = 90°, AB = 24 cm, BC = 7 cm, then cos C =

7 24 25 7
(a) (b) (c) (d)
24 25 24 25
DEEPIKA MA’AM # 8743011101 : CLASS – X : SAMPLE PAPER – 5 : INFINITY ... Think beyond...
SAMPLE PAPER – 5 LEARN MATHEMATICS BY : DEEPIKA MA’AM – 114 –

Q8. In ABC, C = 90°, then tan A + tan B =


b2 a2 c2
(a) (b) a + b (c) (d)
ac bc ab
Q9. If quadrilateral ABCD and PQRS are similar, then x =

(a) 4 cm (b) 5 cm (c) 6 cm (d) 7 cm

Y
Q10. Distance between two parallel tangents is 14cm, then the radius of circle is
(a) 6 cm (b) 7 cm (c) 12 cm (d) 14 cm

T
Q11. sin 45° – cos 45° is equal to

I
(a) 2 cos  (b) 0 (c) 2 sin  (d) 1

d
Q12. The length of the minute hand a wall clock is 7 cm, then how much area does it sweep in 20

N n
minutes ?

o
I
(a) 51 cm2 (b) 49.33 cm2 (c) 51.33 cm2 (d) 52 cm2

e y
The curved surface area of a cylinder of height 14 cm is 88 cm2, then diameter of the cylinder

F
Q13.
is
(a) 8.5 cm (b) 1 cm
k B
(c) 1.5 cm (d) 2 cm

i n
N
Q14. The relationship between mean, median and mode for a moderately skewed distribution is

h
I T
(a) mean = median – 2 mean (b) mode = 3 median – 2 mean
(c) mode = 2 median – 3 mean (d) mode = median – mean
Q15. In figure, at each corner of square side 100 cm, a quadrant of radius 14 cm is formed, then
area of shaded region is

(a) 9834 cm2 (b) 9348 cm2 (c) 9384 cm2 (d) 9884 cm2
Q16. The mean age of combined group of men and women is 30 years. If the mean of the age of
men and women are respectively 32 and 27, then the percentage of women in the group is
(a) 30 (b) 20 (c) 50 (d) 40

Q17. Radius of circumcircle of a triangle ABC is 5 10 units. If point P is equidistant from A(1,
3), B(–3, 5) and C(5, –1), then AP =
(a) 5 units (b) 5 5 units (c) 25 units (d) 5 10 units
Q18. If sin   cos   0, then the value of (sin 4   cos4 ) is
(a) 1 (b) 3/4 (c) 1/2 (d) 1/4
DEEPIKA MA’AM # 8743011101 : CLASS – X : SAMPLE PAPER – 5 : INFINITY ... Think beyond...
SAMPLE PAPER – 5 LEARN MATHEMATICS BY : DEEPIKA MA’AM – 115 –
Direction : In the question number 19 and 20, a statement of Assertion (A) is followed by a
statement of Reason (R). Choose the correct option.
Q19. Statement A (Assertion) : Pair of linear equations : 9x + 3y + 12 = 0, 8x + 6y + 24 = 0 have
infinitely many solutions.

Statement R (Reason) : Pair of linear equations a1x  b1y  c1  0 and a 2 x  b 2 y  c2  0

a1 b1 c1
have infinitely many solutions, if a  b  c .
2 2 2

(a) Both assertion (A) and reason (R) are true and reason(R) is the correct explanation of

Y
assertion (A).
(b) Both assertion (A) and reason (R) are true and reason (R) is not the correct explanation

T
of assertion (A).

I
(c) Assertion (A) is true but reason (R) is false.
(d) Assertion (A) is false but reason (R) is true.

d
N n
Q20. Statement A (Assertion) : PA and PB are two tangents to a circle with centre O, such that

o
I
AOB = 110°, then APB = 90°.

e y
Statement R (Reason) : The length of two tangents drawn from an external point are equal.

F B
(a) Both assertion (A) and reason (R) are true and reason(R) is the correct explanation of

k
assertion (A).

i n
N
(b) Both assertion (A) and reason (R) are true and reason (R) is not the correct explanation
of assertion (A).
h
I T
(c) Assertion (A) is true but reason (R) is false.
(d) Assertion (A) is false but reason (R) is true.

SECTION - B
Section B consists of 5 questions of 2 marks each.
Q21. 5 books and 7 pens together cost Rs. 79, whereas 7 books and 5 pens together cost Rs. 77.
Represent this situation in the form of linear equation in two variables.
Q22. Amandeep draws two right-angled triangle ABC and AMP
right-angled at B and M respectively, as shown in figure.
Prove that :
(i) ABC ~ AMP

CA BC
(ii) 
PA MP
Q23. In the given figure, O is the centre of the circle. The radius of
the circle is 3.1 cm and PA is a tangent drawn to the circle
from point P. If OP = x cm and AP = 6.2 cm, then find the
value of x.
OR
DEEPIKA MA’AM # 8743011101 : CLASS – X : SAMPLE PAPER – 5 : INFINITY ... Think beyond...
SAMPLE PAPER – 5 LEARN MATHEMATICS BY : DEEPIKA MA’AM – 116 –
AB is a tangent drawn from a point A to a circle with centre O and BOC is a diameter of the
circle such that AOC = 110°. Find OAB.
Q24. Find the area of the quadrant of a circle whose circumference is 44 cm.

1 2 2
Q25. If tan   , then evaluate cos ec   sec  .
3 cos ec 2  sec 2 

OR

1 1
If sin (A – B) = and cos (A + B) = , find A and B.

Y
2 2

T
SECTION - C

I
Section C consists of 6 questions of 3 marks each.
Q26. Manju and Manish participate in a cycle race, organised for National integration. Manju

d
N
takes 18 minutes to complete one round, while Manish takes 12 minutes for the same. Sup-

o n
pose they both start at the same time and go in the same direction. After how many minutes,

I y
will they meet again at the starting point ?

e
F B
x 2 x 6 2
Q27. Solve for x :   6 , (x  4, 8)

k
x  4 x 8 3

i n
N
Q28. Abhishek is planning a journey by ship to Andaman. Andaman trip in itself is an advaenture.

h
There are three port in India from where you can sail to Andaman : Kolkata, Chennai and

I T
Vishakhapatnam. Abhishek did not know the length of journey so he took the help of an
expert who helped him by solving a simple mathematical situation related to ships.
The ship covered a certain distance at a uniform speed. If the speed would have been 6 km/h
faster, it would have taken 4 hours less than the scheduled time. And if the speed of ship were
slower by 6 km/hr, it would have taken 6 hours more than the scheduled time. Find the
length of the journey.
OR
Two pipes running together can fill a cistern in 6 minutes. If one pipe takes 5 minutes more
than the other to fil the cistern, find the time in which eace pipe would fill the cistern.
Q29. From a window (120 metres high above the ground) of a house in a street, the angles of
elevation and depression of the top and the foot of another house on opposite side of street
are 60° and 45° respectively. Show that the height of the opposite house is 120(1  3)
metres.
Q30. In the given figure, PAQ is a tangent to the circle with centre
O at a point A. If OBA = 45°, find the value of BAQ and
ACB.
OR
The incircle of ABC touches the sides BC, CA and AB at D, E and F respectively. Show
DEEPIKA MA’AM # 8743011101 : CLASS – X : SAMPLE PAPER – 5 : INFINITY ... Think beyond...
SAMPLE PAPER – 5 LEARN MATHEMATICS BY : DEEPIKA MA’AM – 117 –

1
that AF + BD + CE + AE + BF + CD = (perimeter of ABC).
2
Q31. Cards marked with numbers 4 to 99 are placed in a box and mixed thoroughly. One card is
drawn from this box. Find the probability that the number on the card is :
(i) a perfect square
(ii) a multiple of 7
(iii) a prime mumber less than 30

Y
SECTION - D

T
Section D consists of 4 questions 5 marks each.
Q32. Determine graphically the coordinates o the vertices of triangle formed by the equation

I
2x – 3y + 6 =0 and 2x + 3y – 18 = 0; and the y-axis.Also, find the area of this triangle.
OR

d
N n
Eight times a two-digit number is equal to three times the number obtained by reversing the

o
I
order of the digits. If the difference between the digits of the number is 5, find the number.
Q33.
e y
The diagonals of a quadrilateral ABCD intersect each other at the point O such that

F B
AO CO

k
 . Show that ABCD is a trapezium.
BO DO

i n
N
Q34. A chord PQ of a circle of radius 10 cm subtends an angle of 60° at the centre of circle. Find

h
I T
the area of major and minor segments of the circle.
OR
An umbrella has 8 ribs which are equally speed (see figure). Assuming umbrella to be a flat
circle of radius 45 cm, find the area between the two consecutive ribs of the umbrella.

Q35. Find mean, median and mode of the following data :


Classes Frequency
0  20 6
20  40 8
40  60 10
60  80 12
80  100 6
100  120 5
120  140 3

DEEPIKA MA’AM # 8743011101 : CLASS – X : SAMPLE PAPER – 5 : INFINITY ... Think beyond...
SAMPLE PAPER – 5 LEARN MATHEMATICS BY : DEEPIKA MA’AM – 118 –

SECTION - E
Case study based questions are compoulsory.
Q36. The houses of four friends are located by point A, B, P and Q shown in figure.

If coordinates of A and B with respect to coordinate axes are known and P and Q trisect the
AB. Then answer the following questions based on it
(i) Find the coordinates of P.

Y
(ii) Find the coordinates of Q.
(iii) Find the distance PQ.

T
OR

I
Find the distance AB.
Q37. Deepak and Sanju works together in a bank in Delhi. Hometown of both of them is Rampur

d
in Uttar Pradesh which is at a distance of 300 km from Delhi. To reach Rampur from Delhi

N n
they travel partly by train and partly by bus. This Diwali they travelled separately to Rampur.

o
I y
Deepak travels 60 km by train and remaining by bus and taken 4 hrs. Sanju travels 100 km
by train and remaining by bus and takes 4 hrs. 10 minuts.
e
F B
(i) If speed of train is x km/h and speed of bus is y km/h then write algebraic representation
of the situation.

n k
i
N
(ii) Find the speed of the bus.

h
I
(iii) If speed of the train 90 km/h and speed of the bus is 60 km/h then find time taken by

T
Deepak to travel 60 km by train and 240 km by bus.
OR
If speed of the train is 120 km/h and speed of bus is 60 km/h then find time taken by
Sanju to travel 120 km by train and 180 km by bus.
Q38. Akshat appears for a multiple choice questions test with four choices one of which is right.
He either guesses or copies or known the answer to a question. Total number of questions in
the test is 50.
He knows the answer to 50% of the questions, he guesses the answer of 15 questions and
copies the answer of remaining questions.
(i) What is the probability that he knows the answer of a question ?
(ii) What is probability that Akshat guesses the answer of a question ?
(iii) What is the probability that Akshat copies the answer of a question ?
OR
What is the probability that Akshat does not copy the answer of a question ?

DEEPIKA MA’AM # 8743011101 : CLASS – X : SAMPLE PAPER – 5 : INFINITY ... Think beyond...
SOLUTIONS SAMPLE PAPER – 5 LEARN MATHEMATICS BY : DEEPIKA MA’AM – 119 –

XII - MATHEMATICS
SOLUTIONS : SAMPLE PAPER - 5
A-1. (c) LCM of 7, 8, 11, 12 = 1848
 Bells will toll together after
every 1848 sec.
 In next 3 hrs, number of times
the bells will toll number
BC 7
3  3600 cos C = =
= = 5.84 AC (24) 2  (7) 2

Y
1848
 5 times 7
=

T
A-2. (a) Zeroes of quadraticpolynomial are 25
3 1

I
and  A-8. (d) In ABC, C = 90°
5 2
 quadratic polynomial

d
N n
= k[x2 – (Sum of zeroes) product

o
I
of zeroes]
 2  3  1   3  1 
e y
F
BC
= k  x   5   2   x  5    2 
B
     tanA =
 AC

 2 x 3
n k AC

i
N
= kx    tan B =
10 10  BC

h

I T
k BC AC
= [10x 2  x  3]  tan A + tan B = 
10 AC BC
where k is any constant.
BC2  AC2
A-3. (d) x2
+ (a + 1)x + b =
AC  BC
 x = 2 is a zero
and x = – 3 is another zero AB2
tanA + tanB =
 (2)2 + (a + 1)2 + b = 0
AC  BC

and (–3)2 + (a + 1)2 + b = 0 c2 c2


= 
 4 + 2a + 2 + b = 0 b  a ab
and 9 – 3a – 3 + b = 0 A-9. (c) Quadrilateral ABCD and quadrilat-
 2a + b = – 6 ...(i) eral PQRS are similar
and –3a + b = – 6 ...(ii)
PS PQ
Solving (i) and (ii), we get a = 0 and   (Similarly criteria)
AD AB
b=–6
A-4. (c) x 8

A-5. (c) 15 20
A-6. (c 8
x=  15  6cm
A-7. (d) In ABC, B = 90° 20
A-10. (b) Distance between two parallel tan-
DEEPIKA MA’AM # 8743011101 : CLASS – X : SOLUTIONS SAMPLE PAPER – 5 : INFINITY ... Think beyond...
SOLUTIONS SAMPLE PAPER – 5 LEARN MATHEMATICS BY : DEEPIKA MA’AM – 120 –
gents is always equal to the diameter Angle of shaded region
of circle. = area of square – (ara of four
quadrants)
= 10000 cm2 – 616 cm2
= 9384 cm2
A-16. (d) Let no. of men be x, and women be
y.
Here tangents l1 and l2 are parallel Total age of the group = 30(x + y)
and AB is diameter of circle. Total age of men = 32x years

Y
Hence, AB = 14 cm Total age of women = 27y years
1  30(x + y) = 32x + 27y

T
 Radius = AB  7 cm  30x + 30y = 32x + 27y
2

I
A-11. (b) 3
 x= y
A-12. (c) Angle subtended by minute hand in 2
20 minutes
d
N n
y
 % of women = 100

o
360

I
=  20  120 xy
60
y
e y
F
 Area swept in 20 minutes

B
 3 yy
 100  40%
2

k
22 7  7  120
= 

n
7 360 A-17. (d)

i
N
A-18. (c) sin   cos   0

h
= 51.33 cm2

I T
A-13. (d) Height of cylinder = 14 cm  (sin   cos ) 2  0
Radius of cylinder - r
 Curved surface area = 2rh  (sin 2   cos2   2sin  cos )  0

22  2sin  cos   1
88 = 2   r  14
7 2 2 1
Diameter = 2r  sin  cos  
4
88  7 4 4
= = 2 cm sin 4   cos 4  = sin   cos  
22  14
A-14. (b) Mode = 3 median – 2 mean 2sin 2  cos 2   2sin 2  cos 2 
A-15. (c) Radius of quadrant = 14 cm 2 2 2 2 2
= (sin   cos )  2sin  cos 
(14)2  90 1 1 1
Area of quadrant = = (1)2  2   1  
360 4 2 2
A-19. (a) Both Assertion (A) and Reason (R)
22 14  14  90
=  are true and Reason (R) is the cor-
7 360 rect explanation of Assertion (A).
= 154 cm2 A-20. (d) Assertion (A) is false but reason (R)
Area of four quadrants = 4(154) is true.
= 616 cm2 As per information given in question
we have figure given below :
DEEPIKA MA’AM # 8743011101 : CLASS – X : SOLUTIONS SAMPLE PAPER – 5 : INFINITY ... Think beyond...
SOLUTIONS SAMPLE PAPER – 5 LEARN MATHEMATICS BY : DEEPIKA MA’AM – 121 –
A-23. In right-angeld OAP,

Radius is perpendicular to the tan-


gent at point of conctant. OP2 = OA2 + AP2
Thus, OA AL and OB PB (Using pythagoras theorem)
In quadrilateral, OAPB, we have x2 = (3.1)2 + (6.2)2

Y
OAP + APB + PBO + AOB x2 = 9.61 + 38.44
= 360° x2 = 48.05

T
90° + APB + 90° + 110° = 360° x = 6.93 cm

I
APB = 70°
OR
Assertion (A) is false but reason (R)
AOB + AOC = 180° (linear pair)
is true.

d
N n
A-21. Let the cost of 1 book be Rs. x and the

o
I
cost of 1 pen be Rs. y.
According to question,

e y
F
5x + 7y = 79 ...(i)
and 7x + 5y = 77 ...(ii)

k B
n
A-22. Given : In ABC, B = 90° and in  AOB = 180° –AOC

i
N
AMP, M = 90°

h
= 180° – 110° = 70°

I T
In AOB,
OBA + OAB + AOB = 180°
 90° + OAB + 70° = 180°
OAB = 180° – 160° = 20°
A-24. Circumference of the circle = 44 cm
To Prove : (i) ABC ~ AMP
 2r = 44 cm
CA BC
(ii)  44  7
PA MP  r=  7 cm
2  22
Proof :
 Area of the quadrant of a circle
(i) In ABC and AMP,
ABC = AMP (Each 90°) 1 2 1 22 77
= r    7  7  cm2
BAC = MAP (Common) 4 2 7 2
 ABC ~ AMP (AA similarlity) 1
(ii) As ABC ~ AMP, A-25. Given tan =
3
AC BC 1
 = As we know that tan 30° =
AP MP 3
(Ratios of the corresponding sides of   = 30°
similar triangles
cos ec2   sec 2 
Putting  = 30° in , we
CA
=
BC
Hence Proved. cos ec 2   sec 2 

PA MP get
DEEPIKA MA’AM # 8743011101 : CLASS – X : SOLUTIONS SAMPLE PAPER – 5 : INFINITY ... Think beyond...
SOLUTIONS SAMPLE PAPER – 5 LEARN MATHEMATICS BY : DEEPIKA MA’AM – 122 –

2 90  820 45  205
2  2  = 
(2)    14 7
cos ec 2 30  sec 2 30  3
=
cosec2 30  sec 2 30  2 
2
45  205 45  205
(2) 2    Hence, x= ,
 3 7 7
A-28. Let the usual speed of ship be x km/h and
4 the usual time be y hours.
4
3  8 1
=  Distance covered = xy km
4 16 2
4 Case I :
3
OR When speed = (x + 6) km

Y
1 then time taken = (y – 4) hour
sin(A – B) =  sin 30

T
2 Now, distance covere = xy
 A – B = 30° ...(i)  (x + 6)(y – 4) = xy

I
1  2x – 3y = – 12 ...(i)
cos(A + B) = = cos 60° Case II :
2

d
N
When speed = (x – 6) km/h

n
 A + B = 60° ...(ii)

o
then time taken = (y + 6) hour

I
Adding (i) and (ii), we get
2A =90°  A = 45°

e y
Now distance covered = xy

F
 (x – 6)(y + 6) = xy

B
Putting the value in (i), we get
x–y=6 ...(ii)

k
45° – B = 30°
Solving (i) and (ii), we get

n
 B = 15°

i
N
x = 30, y = 24

h
A-26. Factors of 12 = 2 × 2 × 3 = 22 × 3

I
 Distance covered = xy = 30 × 24
Factor of 18 = 2 × 3 × 3 = 2 ×
LCM (12, 18) = 22 ×32 = 36
32
T = 720 km
 The length of the journey = 720 km
 After 36 mintues, they will meet OR
again at the staring point.
Let the time taken by first pipe to fill the
x 2 x 6 2 cistern be x minutes
A-27.  6
x  4 x 8 3
1
 In 1 minute, it can fill of cistern.
(x  2)(x  8)  (x  6)(x  4) 20 x
 
(x  4)(x  8) 3 Time taken by second pipe to fill the cister
 14x2 – 180x + 520 = 0 = (x + 5) minutes

 7x2 – 90x + 260 = 0 1


 In 1 minute, it fill of cistern.
x5
Here, a = 7, b = – 90, c = 260
According to question
 Discriminate,
D = b2 – 4ac 1 1 1
 
x x 5 6
= (–90)2 – 4 × 7 × 260
= 820  x 2– 7x – 30 = 0
Using quadratic formula, we have  (x – 10)(x + 3) = 0
 x = 10, –3
 b  D 90  820
x=   x = 10 [x = –3 is rejected]
2a 27
DEEPIKA MA’AM # 8743011101 : CLASS – X : SOLUTIONS SAMPLE PAPER – 5 : INFINITY ... Think beyond...
SOLUTIONS SAMPLE PAPER – 5 LEARN MATHEMATICS BY : DEEPIKA MA’AM – 123 –
 Time taken by first pipe = 10 minute We have OA = OB
Time taken by second pipe = 15 minutes (Radii of the same circle)
A-29. Let bet he window and CE be the oppo-  3 = 1
site house. (Angle opposite to equal
Now, CD = AB = 120 m ...(i) sides of a triangle are equal)
(Opposite sides of a rectangle)  3 = 45° ( 1=45°, given)
AB Also, in OAB
In right-angled ABC, tan 45° =
BC 1 + 2 + 3 = 180°
(Angle sum property of a triangle)

Y
 45° + 2 + 45° = 180°
 2 = 180° – 90° = 90°

T
1
Now 4 = 2 = 45°

I
2
(Degree measure theorem)

d
 ACB = 45°

N n
120 Now, BAQ = OAQ – 3

o
I
 1=
BC

y
= 90° – 45° = 45°

e
 BC = 120 m ...(ii) [OA AQ]

F B
Now, AD = BC OR

k
(Opposite sides of a rectangle) Given : Sides AB, BC and CA of ABC,

i n touches the incircle at D, E and F respec-

N
ADS = 120 m [From (ii)]...(iii)

h
In right-angled ADE, tively.

I T
To prove :
DE
tan 60° = AF + BD + CD = AE + BF + CD
AD
1
DE = (perimeter of ABC)
 3 = [From (iii)] 2
120
Proof : Since lengths of the tangents
 DE = 120 3 m drawn from an external point to a circle
 Height of the opposite house are equal
CE = CD + DE Therefore,
AF = AE ...(i)
= 120 m + 120 3 m
BD = BF ...(ii)
= 120(1  3)m CE = CD ...(iii)
A-30. Given : PAQ is a tangent to th circle with
centre O at a piont A and OBA = 45°.
To find : BAQ and ACB

Adding (i), (ii) and (iii), we get


AF + BD + CE = AE + BF + CD
Now,
DEEPIKA MA’AM # 8743011101 : CLASS – X : SOLUTIONS SAMPLE PAPER – 5 : INFINITY ... Think beyond...
SOLUTIONS SAMPLE PAPER – 5 LEARN MATHEMATICS BY : DEEPIKA MA’AM – 124 –
perimeter of ABC = AB + BC + CA
 Perimeter of ABC
= (AF + FB) + (BD + CD) + (EC + AE)
= (AF + AE) + (BD + BF) + (EC + CD)
= 2(AF + BD + CE)
 AF + BD + CE
1
= (perimeter of ABC) Coordinates of the vertices of a triangle
2
are A(0, 2), B(3, 4) and C(0, 6).
So, AF + BD + CE = AE + BF + CD

Y
1
1  Area of ABC = base × height
= (perimeter of ABC) 2
2

T
Hence proved. 1 1
= × AC × BD = × 4 × 3

I
2 2
A-31. Total number of cards = 96
= 6 units
Number of ways to draw one card = 96

d
OR
(i) Let A be the event of number on the

N n
card is a perfect square. Let the digit at unit’s place be x and the

o
I
digt at ten’s place be y.

y
Perfect squares are 4, 9, 16, 25, 36,

e
49, 64, 81  Required number = 10y + x

F B
Outcomes favourable to A = 8 When the digts are reversed, the number
becomes 10x + y
 P(A) =
8

1

n k
According to question,

i
N
96 12
8(10y + x) = 3(10x + y)
(ii) Let B be the event of number on the
h
I T
 80y + 8x = 30x + 3y
card is a multiple of 7.
 77y – 22x = 0  7y – 2x = 0 ...(i)
Multiples of 7 are 7, 14, 21, 28, 35,
42, 49, 56, 63, 79, 77, 84, 91, 98 Also x – y = 5 (keeing x > y) ...(ii)
Outcomes favourable to B = 14 Multiplying (ii) by 2 and adding to (i), we
get
14 7 y= 2
 P(B) = 
96 48
Putting y = 2 in (ii), we get
(iii) Let C be the event of number on the
x–2= 5
card is a prime number less than 30.
 x= 7
Prime numbers are 5, 7, 11, 13, 17,
19, 23, 29.  Required number
Outcomes favourable to C = 8 10y + x = 10 × 2 + 7 = 27
A-33. Given : A quadrilateral ABCD, whose
8 1 diagonals intersect at O.
 P(C) = 
96 12
A-32. The solution table for 2x – 3y + 6 = 0 is
x 0 3 3
y 2 0 4
The solution table for 2x + 3y – 18 = 0 is
x 0 9 6 AO CO AO BO
and = or 
y 6 0 2 BO DO OC DO

DEEPIKA MA’AM # 8743011101 : CLASS – X : SOLUTIONS SAMPLE PAPER – 5 : INFINITY ... Think beyond...
SOLUTIONS SAMPLE PAPER – 5 LEARN MATHEMATICS BY : DEEPIKA MA’AM – 125 –

To prove : ABCD is a trapezium = 43.3 cm2


Construction : Draw EO || AB  Area of minor segment ACBD
Proof : In ABC, OE || AB = Aea of sector OACB – Area of
triangle OAB
AO BE
 = [By B.P.T.] ...(i) = (52.38 – 43.30) cm2
OC EC
But given that = 9.08 cm2

AO BO Areaw of circle = r2


= ...(ii)
OC DO 22
=  (10) 2
From equation (i) and (ii) 7
BO
DO
=
BE
EC
OE || DC
=

T
7

Y
22  100 2

= 314.28 cm2
cm

I

[By converse of B.P.T.]  Area of major segment ADBE
OE || AB and OE || DC  AB || DC

d
= Area circle – Areaof minor segment

N n
 ABCD is a trapezium. = (314.28 – 9.08) cm2

o
I
A-34.
= 305.20 cm2
OR
e y
F B
Radius of the circle 45 cm

n k
Number of ribs = 8

i
N
Angle between two consecutive ribs

h
I
central angle of the circle

Radius of the circle 10 cm


Central angle subtended by chord AB
T = number of the sec tors (ribs)

360
= = 45°
= 60° 8
Area of minor sector OACB Area between two consecutive ribs
= Area of one sector of circle
r 2 
=
360 r 2 
=
22 (10) 2  60 360
= 
7 360 22 45  45 45
= 
22 10  10 7 360
= 
7 6 11  45  9  5 2
= cm
1100 2 74
= cm = 52.38 cm2.
21
22275
Area of equilateral triangle OAB formed = cm2
28
by radii and chord

3 2 3
= a =  (10)2
4 4

1.732
=  100
4
DEEPIKA MA’AM # 8743011101 : CLASS – X : SOLUTIONS SAMPLE PAPER – 5 : INFINITY ... Think beyond...
SOLUTIONS SAMPLE PAPER – 5 LEARN MATHEMATICS BY : DEEPIKA MA’AM – 126 –
A-35. A-36. (i) As P divides AB in the ratio 1 : 2.
Classes Frequency Cumulative  coordinates of P are
frequency
0  20 6 6 1( 2)  2(4)
x-coordinate =
20  40 8 14 1 2
40  60 10 24  Median
60  80 12 36 Class 2  8 6
=  2
80  100 6 42 3 3
100  120 5 47
120  140 3 50 1( 3)  2( 2)
y-coordinate =
n  50 1 2

Y
n 3  2 5
 = 25 = 
2 3 3

T
Median class = (60 – 80)
 5 

I
l = 60, f = 12, c.f. = 24, h = 20 Coordinate of P are  2, 
 3 
n  c.f . (ii) Coordinate of Q are as Q divides AB

d
Median = l  2 h

N
f in the ratio 2 : 1

o n
I
25  24 x-coordinate y-coordinate

y
= 60   20

e
12 2( 3)  1( 1)

F
=

B
1 5 180  5 1 2
= 60  

k
3 3 6  1 7

n
= 

i
N
185 3 3
= = 61.6
3
h
I T
 7 
Modal class = (60 – 80) as its frequency Coordinate of Q are  0, 
 3 
is 12
(iii)
h = 20, l = 60, f1  12, f0  10, f 2  6
Distance PQ
 f1  f0 
Mode = l    h 2
 2f1  f0  f 2   7 5 
= (0  2) 2    
 3 3
12  10
= 60   20
2  12  10  6 2
2  2 
= (2)   
2  3 
= 60   20 = 65
8
Now, Mode = 3 Median – 2 Mean 4
= 4
65 = 3(61.6) – 2Mean 9
2 Mean = 184.8 – 65
40
2 Mean = 119.8 =
9
119.8
 Mean = = 59.9 1
2 = 40 units
3
 Mean = 59.9; Median = 61.6,
OR
Mode = 65

DEEPIKA MA’AM # 8743011101 : CLASS – X : SOLUTIONS SAMPLE PAPER – 5 : INFINITY ... Think beyond...
SOLUTIONS SAMPLE PAPER – 5 LEARN MATHEMATICS BY : DEEPIKA MA’AM – 127 –

Distance AB

= (2  4)2  (3  1)2

= (6)2  (2) 2

= 36  4
= 40 units

60 240
A-37. (i)  4

Y
x y

100 200 1

T
 4
x y 6

I
(ii) 80 km/h
14
(iii) hours OR 4 hours
d
N n
3

o
I
1
A-38. (i)
2
e y
F B
3

k
(ii)
10

i n
N
1 4
(iii) OR
h
I T
5 5

DEEPIKA MA’AM # 8743011101 : CLASS – X : SOLUTIONS SAMPLE PAPER – 5 : INFINITY ... Think beyond...
SOLUTIONS SAMPLE PAPER – 5 LEARN MATHEMATICS BY : DEEPIKA MA’AM – 128 –

— Notes —
_________________________________________________________________________________________________________

_________________________________________________________________________________________________________

_________________________________________________________________________________________________________

_________________________________________________________________________________________________________

_________________________________________________________________________________________________________

Y
_________________________________________________________________________________________________________

_________________________________________________________________________________________________________

I T
_________________________________________________________________________________________________________

_________________________________________________________________________________________________________

d
N n
_________________________________________________________________________________________________________

o
I y
_________________________________________________________________________________________________________

e
F B
_________________________________________________________________________________________________________

k
_________________________________________________________________________________________________________

n
i
N h
_________________________________________________________________________________________________________

I T
_________________________________________________________________________________________________________

_________________________________________________________________________________________________________

_________________________________________________________________________________________________________

_________________________________________________________________________________________________________

_________________________________________________________________________________________________________

_________________________________________________________________________________________________________

_________________________________________________________________________________________________________

_________________________________________________________________________________________________________

_________________________________________________________________________________________________________

_________________________________________________________________________________________________________

_________________________________________________________________________________________________________

_________________________________________________________________________________________________________

_________________________________________________________________________________________________________
DEEPIKA MA’AM # 8743011101 : CLASS – X : SOLUTIONS SAMPLE PAPER – 5 : INFINITY ... Think beyond...
PRACTICE SAMPLE PAPER – 1 LEARN MATHEMATICS BY : DEEPIKA MA’AM – 129 –

X - MATHEMATICS
PRACTICE SAMPLE PAPER - 1
Time Allowed : 3 Hours] [Maximum Marks : 80
General Instructions :
1. This Question Paper has 5 Sections A - E.
2. Section A has 20 MCQs carrying 1 mark each.
3. Section B has 5 questions carrying 02 marks each.
4. Section C has 6 questions carrying 03 marks each.

Y
5. Section D has 4 questions carrying 05 marks each.

T
6. Section E has 3 case based integrated units of assessment (04 marks each)with sub-parts of the
values of 1,1 and 2 marks each respectively.

I
7. All Questions are compulsory. However an internal choice in 2 Qs. of 5 marks, 2 Qs of 3 marks
and 2 Questions of 2 marks has been provided. An internal choice has provided in the 2 marks

d
N
questions of Section E.

o n
I
8. Draw neat figures wherever required. Take  = 22/7 wherever required if not stated.

SECTION - A
e y
F B
Section A consists of 20 questions of 1 mark each.
Q1.

n k
There are 567 boys and 448 girls in a school that are to be divided into equal sections of

i
N
either boys or girls alone. The total number of sections thus formed are

h
I
(a) 22 (b) 16 (c) 36 (d) 21

Q2. T  13 
A circle drawwn with origin as the centre through  , 0  . The point which does not lie in
2 
the interior of the circle is

 3   7
(a)   ,1 (b)  2, 
 4   3

 1  5
(c)  5,   (d)  6, 
 2  2

5
Q3. One zoer of the quadratic polynomialk 2x2 – 8x – m is , then unique solution, then p 
2

4 4
(a) –3 (b) 4 (c) (d)
3 3
Q4. If system of equations 4x + 6y – 21 and px – 2y = 15 has unique solution, then p 

4 4
(a) –3 (b) 4 (c) (d)
3 3

DEEPIKA MA’AM # 8743011101 : CLASS – X : PRACTICE SAMPLE PAPER – 1 : INFINITY ... Think beyond...
PRACTICE SAMPLE PAPER – 1 LEARN MATHEMATICS BY : DEEPIKA MA’AM – 130 –

Q5. The triangle whose vertices are (0, 0), (2.7, 0) and (0, 4.9) is a/an
(a) equilateral triangle (b) right-angled triangle
(c) isoscels triangle (d) obtuse-angled triangle
Q6. If a and b are two coprime numbers, then a3 and b3 are
(a) coprime (b) not coprime (c) even (d) odd
Q7. Two natural numbers whose differencie is 10 and the least common multile is 420, are
(a) 80 and 70 (b) 70 and 60 (c) 20 and 10 (d) 90 and 80

Y
Q8. If sin x + cosec x = 2, then sin19 x  cos ec 20 x 
(a) 219 (b) 220 (c) 2 (d) 239

T
Q9. In figure, which of the following is correct ?

NI o n d
Q10.
(a) DE || BC

F I
(b) DF || AC
B
(c) EF || AB

k
e y (d) None of these
The tops of two poles of heights 20 m and 14 m are connected by a wire. If the wire makes

i n
N
an angle of 30° with the horizontal, then the length of the wire is

h
I
(a) 8 m (b) 10 m (c) 12 m (d) 14 m
Q11.
then AC = T
In figure, AB and AC are tangents to a circle with centre O and radius 8 cm. If OA = 17 cm,

(a) 15 cm (b) 9 cm (c) 25 cm (d) 353 cm


Q12. In the given figure, ABC is a triangle right-angled at A, with AB = 21 cm and AC = 42 cm.
With the vertices A, B and C as centres, arcs are drawn each of radius 10 cm, then the area
of the shaded region is (Use  = 3.14)

(a) 284 cm2 (b) 144 cm2 (c) 441 cm2 (d) 157 cm2

DEEPIKA MA’AM # 8743011101 : CLASS – X : PRACTICE SAMPLE PAPER – 1 : INFINITY ... Think beyond...
PRACTICE SAMPLE PAPER – 1 LEARN MATHEMATICS BY : DEEPIKA MA’AM – 131 –

Q13. How many bags of grain can be stored in the cuboid granary 12 m × 6 m × 5 m, if each bag
occupies a space of 0.48 m3 ?
(a) 750 (b) 705 (c) 600 (d) 640
Q14. A set of numbers consists of four 5’s, six 7’s, ten 9’s, eleven 12’s, three 13’s two 14’s. The
mode of this set of numbers is ________.
(a) 10 (b) 11 (c) 12 (d) 13
Q15. A wheel makes 1000 revolutions in covering a distance of 0.88 km. The radius of the wheel
is

Y
(a) 7 cm (b) 14 cm (c) 21 cm (d) 28 cm
Q16. The value of k for which the system of equations x + y – 4 = 0 and 2x + ky = 3, has no

T
solution, is

I
(a) – 2 (b)  2 (c) 3 (d) 2
Q17. A man is known to speak truth 3 out of 4 times. He throws a die and a number other than six
comes up. Find the probability that he reports it is a six.
d
Q18.
(a)
3
4

If cosec A – cot A =
(b)

4
5
1
4

F I N
, then cosec A =

i n
(c)
1
2

k B e y o (d) 1 n

N h
47 59 51 41

I
(a) (b) (c) (d)
40 40
T 40 40
Direction : In the question number 19 and 20, a statement of Assertion (A) is followed by a
statement of Reason (R). Choose the correct option.

3
Q19. Statement A (Assertion) : In a right angled triangle, if tan   , the greatest side of the
4
triangle is 5 units.
Statement R (Reason) : (greatest side)2 = (hypotenuse)2 = (perpendicular)2 + (base)2
(a) Both assertion (A) and reason (R) are true and reason(R) is the correct explanation of
assertion (A).
(b) Both assertion (A) and reason (R) are true and reason (R) is not the correct explanation
of assertion (A).
(c) Assertion (A) is true but reason (R) is false.
(d) Assertion (A) is false but reason (R) is true.
Q20. Statement A (Assertion) : The arithmetic mean of the following given distribution table is
20.
x 7 10 13 16 19
y 2 6 8 10 12

DEEPIKA MA’AM # 8743011101 : CLASS – X : PRACTICE SAMPLE PAPER – 1 : INFINITY ... Think beyond...
PRACTICE SAMPLE PAPER – 1 LEARN MATHEMATICS BY : DEEPIKA MA’AM – 132 –

 f1  f0 
Statement R (Reason) : Mean = l    h
 2f1  f0  f 2 
(a) Both assertion (A) and reason (R) are true and reason(R) is the correct explanation of
assertion (A).
(b) Both assertion (A) and reason (R) are true and reason (R) is not the correct explanation
of assertion (A).
(c) Assertion (A) is true but reason (R) is false.
(d) Assertion (A) is false but reason (R) is true.

SECTION - B
Section B consists of 5 questions of 2 marks each.

T Y
I
Q21. For which values of p, does the pair of equations given below has unique solution ?
4x + py + 8 = 0, 2x + 2y + 2 = 0

d
N n
QR QT

o
Q22. In the given figure, , 1 = 2. Show that PQR ~ TQR.

I
QS PR

e y

N F h i n k B
Q23.

I T
From a point P, the length of the tangent to a circle is 8 cm and distance of P from the centre
of the circle is 17 cm. Find the radius of the circle.
OR
Two concentric circles of radii x and y are given, where x > y. Find the chord of the larger
circle which touches the smaller circle.
Q24. Find the area of a sector of a circle, when the radius of the circle is 21 cm and angle of the
sector is 60°.
1
Q25. Evaluate : cos 2 30  sin 2 45  tan 2 60  cos 90
3
OR

a a sin   b cos 
If tan   , find .
b a sin   b cos 

SECTION - C
Section C consists of 6 questions of 3 marks each.
Q26. Explain why 15 × 7 + 7 is a composite number ?

DEEPIKA MA’AM # 8743011101 : CLASS – X : PRACTICE SAMPLE PAPER – 1 : INFINITY ... Think beyond...
PRACTICE SAMPLE PAPER – 1 LEARN MATHEMATICS BY : DEEPIKA MA’AM – 133 –
Q27. Find the zeroes of the quadratic polynomial x2 – 2x – 8, and verify the relationship between
the zeroes and its coefficients.
Q28. One year ago, a man was 8 times as old as his son. At present, his age is equal to the square
of his son’s age in years. Find their present ages.
OR
A fast train takes 3 hours less than a slow train for a journey of 600 km. If the speed of the
slow train is 10 km/h less than that of the fast train, find the speeds of the two trains.
Q29. The angle of elevation of the top of a tower from a point on the ground is 45°. On walking

Y
30 metres towards the tower, the anle of elevation becomes 60°. Find the height of the
tower and the original distance from the foot of the tower. (Use 3  1.73 )

T
Q30. In the given figure, XP and XQ are tangents from a point X to the circle with centre OI and

I
ARB is a tangent at point R. Prove that XA + AR = XB + BR.

F I N i n k B e y o n

N
OR

h
I T
Prove that the tangent to a circle is perpendicular to the radius through the point of contact.
Q31. Cards as shown are placed in the box. All even numbered cards and prime numbered cards
are shaded. Then all the cards are mixed thoroughly. A card is drawn at random from the box.
Find the probability that the number on the card is
(i) an even number
(ii) a prime number
(iii) divisible by 4

SECTION - D
Section D consists of 4 questions 5 marks each.
Q32. The first and the last terms of an AP are 6 and 348 respectively. If the common difference is
9, how many terms are there and what is their sum ?
OR
If 9th term of an AP is zero, prove that its 29th term is double of its 19th term.
DEEPIKA MA’AM # 8743011101 : CLASS – X : PRACTICE SAMPLE PAPER – 1 : INFINITY ... Think beyond...
PRACTICE SAMPLE PAPER – 1 LEARN MATHEMATICS BY : DEEPIKA MA’AM – 134 –

Q33. In the given figure the radius of incircle of ABC of area 84 cm2 is 4cm and the lengths of
the segments AP and BP into which side AB is divided by the contact P are 6 cm and 8 cm.
Find the length of the sides AC and BC.

Y
Q34. A sphere of diameter 12 cm is dropped in a right circular cylindrical vessel partly filled with
water. If the sphere is completely submerged in water, the water level in the cylindrical

T
5

I
vessel rises by 3 cm. Find the diameter of the cylindrical vessel.
9
OR

d
N n
A solid is in the form of a right circular cylinder mounted on a solid hemisphere of radius

o
I
14cm. The radius of the base of the cylindrical part is 14cm and the vertical height of the
complete solid is 28 cm. Find the volume of the solid.
e y
F B
Q35. The following data gives the distribution of total monthly mobile phones expenditure of 200

k
families of a locality. Find the modal monthly mobile phones expenditure of the families.

i n
N
Mobile Phones Expenditure (in Rs.) Number of families

h
1000  1500 24

I T
1500  2000 40
2000  2500 31
2500  3000 27
3000  3500 32
3500  4000 16
4000  4500 22
4500  5000 8
SECTION - E
Case study based questions are compoulsory.
Q36. Students of class X of a secondary school in Delhi have been allotted a rectangular plot of
land for their maths integrated project. They made a rectangular grid on this plot as shown in
the figure.

Four
DEEPIKA students
MA’AM take their: CLASS
# 8743011101 positions
– X :atPRACTICE
A, B, C SAMPLE
and D as shown
PAPER – 1in the grid.... Think beyond...
: INFINITY
PRACTICE SAMPLE PAPER – 1 LEARN MATHEMATICS BY : DEEPIKA MA’AM – 135 –

Refer to the sketch


(i) Find the distance between B and D.
(ii) Find the mid-point of AC.
(iii) Find the distance of C from the x-axis.
OR
Find the coordinates of the point which divides the line segment BD in the ratio 1 : 2
internally.

Y
Q37. Rampal deposits some money in bank and gets an increment on it every year.

I T d

F I N i n k B e y o n

I N T h
For example, Rampal deposits some money Rs. p in bank and get an interest of Rs. 1 on it
every year then this is represented as
p, p + 1, p + 2I, p + 3I...
The sequence p, p + 1, p + 2I, p + 3I... form an AP, with 1st term p and common difference
I. Using the AP apply formula and its application.
Answer the questions based on above
(i) If 4p + 8, 2p2 + 3p + 6 and 3p2 + 4p + 4 form three consecutive terms of an AP, then find
the value of p.
(ii) Find the 12th term of AP, 10.0, 10.5, 11.0, 11.5
OR
If the 17th term of an AP exceeds its 10th term by 14, then find the common difference.

1 1  x 1  2x
(iii) Find the common difference of an AP. , , ?
x x x

Q38. A building is made by keeping the lower window of a building at a particular height above
the ground and upper window is constructed at some height vertically above the lower
window. Position of both windows are shown in diagram.
DEEPIKA MA’AM # 8743011101 : CLASS – X : PRACTICE SAMPLE PAPER – 1 : INFINITY ... Think beyond...
PRACTICE SAMPLE PAPER – 1 LEARN MATHEMATICS BY : DEEPIKA MA’AM – 136 –

Both windows are designed and constructed in order to have proper Sun light.

T Y
NI d
At certain instant, the angle of elevation of balloon from these windows are shown. Balloon

o n
I
is flying at centre height H above the ground.
(i) Find the length AR (in terms of H)
e y
F B
(ii) What will be the value of Height H.

k
(iii) Find the distance between the balloon and the upper window.

n
i
N
OR

h
I T
Find the distance between the balloon and the lower window.

DEEPIKA MA’AM # 8743011101 : CLASS – X : PRACTICE SAMPLE PAPER – 1 : INFINITY ... Think beyond...
Infinity Provides coaching for :
III - V ( ALL SUBJECTS )
VI - X ( MATHS & SCIENCE )
XI - XII ( COMMERCE & SCIENCE STREAM )

INFINITY SECRET TO SUCCESS


1.INFINITY faculty members are highly qualified and dedicated to
provide quality education with a difference .
2. Periodic weekly / Fortnightly / Monthly test and individual attention .
3. Regular Parents Teacher Meeting for providing feedback of
performance the student in class.
4. Progress Report Card & recording performance of student in all tests
conducted at the academy .
5.Self Prepared study material of each topic , which are designed in
accordance with updated CBSE syllabus.
6. Highly interactive and modernized methodology and teaching.

.....Glimpse of our results.....


Akshita -95 Puneet Sahool-95 Taapti - 93 Piyush Nag -92
Hamsa Fahim-93
Dev Samaj modern
Cambridge Public School blubells international Cambridge School SNP Mata Gujri school
school nehru nagar
Srinivaspuri school

khushi Agarwal -96


Amity International School

Shriansh - 95
Akansha-92 Naman Garg-95 Nishant -95 BHART.VIDYA BHAVAN'S
Kamakshi-95 National Public School GLT Saraswati school
Mayoor School MEHTA VID K G MARG

Apeejay School Noida

Tanishi -97
Mahasweta -95 Aditiya Dubey-96 Simran Kapoor-95
Cambridge School bluebells international
Ishaan Bansal - 95
Dev Samaj School
Amity International PV
Ayush -94 Mayoor school Noida
Srinivaspuri school
Cambridge Public School
Srinivaspuri

INFINITY THINK BEYOND ..........


B- 104 , New Friends Colony , Near Mata Mandir, New Delhi - 110025
8743011101,8130061101

You might also like